Tài liệu, học tập, trắc nghiệm, tiếng anh, văn bản, biểu...

208
VnDoc - Tải tài liệu, văn bản pháp luật, biểu mẫu miễn phí A/ LISTENING: (2pts) Listen to two students talking about what they will do if they become the president of the 3Rs club. Put the words or a number in each blank to complete the table: (2pts) B/ USE OF ENGLISH: I/ Find the word which has a different sound in the underlined part: (1.0 pt) 1. A. da d B. ha t C. pla nt D. ha ppy 2. A. do ctor B. o pening C. go ld D. po stcard 3. A. earth B. farth er C. both D. marath on 4. A. ca ve B. fa mous C. la te D. cha nnel II/ Choose the best answer by circling A, B, C or D: (1.0 pt) 1. My father ..................to the zoo last week. A. go B. will go C. went D. goes 2. If we cut down more forests, there..............more floods. A. are B. have been C. were D. will be 3.Sydney Opera House is in ………………………… A. England B. Australia C. Singapore D. France . TRƯỜNG THCS BÌNH CHÂU ---------------------------- Họ và tên: ...................................................... Lớp: 6 ......... ĐỀ KIỂM TRA HỌC KÌ II NĂM HỌC 2014 - 2015 MÔN: TIẾNG ANH - LỚP 6 Thời gian làm bài: 60 phút Name Mi Nam Class (1)…………………………………… (5)………………………. Idea 1 Put (2)…………… bin in every classroom Encourage student to use the(6)…….. Idea 2 Buy energy(3)…………………............... Have a(7)…………………… group Idea 3 Have(4)………………...................fairs. Organize(8)…………………….fairs .

Transcript of Tài liệu, học tập, trắc nghiệm, tiếng anh, văn bản, biểu...

Page 1: Tài liệu, học tập, trắc nghiệm, tiếng anh, văn bản, biểu ...s1.vndoc.com/.../06/40-de-thi-hoc-ky...6-co-dap-an.pdf · "/ 0. 1 2 %$ 3 ( 0 4 5 6 2 3 ( 5789 578: 2

VnDoc - Tải tài liệu, văn bản pháp luật, biểu mẫu miễn phí

A/ LISTENING: (2pts)

Listen to two students talking about what they will do if they become the president of the 3Rsclub. Put the words or a number in each blank to complete the table: (2pts)

B/ USE OF ENGLISH:I/ Find the word which has a different sound in the underlined part: (1.0 pt)

1. A. dad B. hat C. plant D. happy2. A. doctor B. opening C. gold D. postcard

3. A. earth B. farther C. both D. marathon4. A. cave B. famous C. late D. channel

II/ Choose the best answer by circling A, B, C or D: (1.0 pt)1. My father ..................to the zoo last week.

A. go B. will go C. went D. goes

2. If we cut down more forests, there..............more floods.

A. are B. have been C. were D. will be

3.Sydney Opera House is in …………………………

A. England B. Australia C. Singapore D. France

.

TRƯỜNG THCS BÌNH CHÂU----------------------------

Họ và tên: ......................................................

Lớp: 6 .........

ĐỀ KIỂM TRA HỌC KÌ IINĂM HỌC 2014 - 2015

MÔN: TIẾNG ANH - LỚP 6

Thời gian làm bài: 60 phút

Name Mi Nam

Class (1)…………………………………… (5)……………………….

Idea 1 Put (2)…………… bin in every classroom Encourage student to use the(6)……..

Idea 2 Buy energy(3)…………………............... Have a(7)…………………… group

Idea 3 Have(4)………………...................fairs. Organize( 8)…………………….fairs .

Page 2: Tài liệu, học tập, trắc nghiệm, tiếng anh, văn bản, biểu ...s1.vndoc.com/.../06/40-de-thi-hoc-ky...6-co-dap-an.pdf · "/ 0. 1 2 %$ 3 ( 0 4 5 6 2 3 ( 5789 578: 2

VnDoc - Tải tài liệu, văn bản pháp luật, biểu mẫu miễn phí

4. It is getting light. Shall I turn ............the light to save electricity.

A. off B. up C. on D. in

C/ READING (2,0pts)

I. Choose A, B, C or D to complete the following passage: (1.0 pt)AIR QUALITY

People need to breath. If they don’t breath, they (1)............. die. But how clean is the air people breath?

If they breath dirty air, they will have breathing problems and (2)............. ill. Plants and animals need

clean air, too. (3)............. of things in our lives create harmful gases and make the air dirty, like cars,

motorbike and factories. Dirty air called (4).............. air. Air pollution can also makes our Earth

warmer. The problem of air pollution started with the burning a lot of coal in homes and factories.

1- A. will B. is C. are D. won’t

2- A. become B. became C. becomes D. will become

3- A. Some B. Many C. Few D. A lot

4- A. pollute B. polluted C. pollution D. polluting

II- Read the passage carefully then answer the questions below. (1.0 pt)

My name is Dick. I always wish to have a motorhome in my life. My motorhome won’t need to be big,

but it may have everything I need inside.

There will be a sofa and a lovely table in the head of the car. This will be the place for my friends when

they visit me. It is like a living room. The biggest part of my motorhome will be the kitchen. It’ll be in

the middle of the car. There will be some model equipments in the kitchen such as dishwasher, fridge,

oven, washing machine.... A small toilet and a shower will be in the kitchen, too. The last part will be

my bedroom. It is also my entertainment place. There will be a small bed, a wireless TV, and a high-

tech computer. It would be fantastic if I had my own motorhome. The best thing about having a

motorhome is that I can drive it myself to everywhere which I am interested.

1- What kind of the house does the writer wish to have?

.......................................................... ................................... ..............................................

2- How many parts are there in the house?

.......................................................... ................................... ..............................................

3- Will this house be big?

.......................................................... ..................................................................................

Page 3: Tài liệu, học tập, trắc nghiệm, tiếng anh, văn bản, biểu ...s1.vndoc.com/.../06/40-de-thi-hoc-ky...6-co-dap-an.pdf · "/ 0. 1 2 %$ 3 ( 0 4 5 6 2 3 ( 5789 578: 2

VnDoc - Tải tài liệu, văn bản pháp luật, biểu mẫu miễn phí

4- Is the kitchen the biggest part of the house?

.......................................................... ................................... ..............................................

D.WRITING (2pts)

I/ Arrange the following sets of words to make meaningful sentences: (1.0 pt)1. recycle / the environment / bottles and cans / we will help / If we /

..................................................................................................................................

2. read and write / you / Could / when you were 6 / ?

...................................................................................................................................

3. will be able to / In 2030, / robots / do many things / like humans /.

....................................................................................................................................

4. visited / We / in the countryside / our grandparents / last week.

……………………………………………………………………………………….

II/ You and your classmates want to make your school “green” place. Write down four rules for

the class members to follow. (1,0pt)RULES FOR A “GREEN” SCHOOL

1…………………………………………………………………………………………

2…………………………………………………………………………………………

3…………………………………………………………………………………………

4…………………………………………………………………………………………

Page 4: Tài liệu, học tập, trắc nghiệm, tiếng anh, văn bản, biểu ...s1.vndoc.com/.../06/40-de-thi-hoc-ky...6-co-dap-an.pdf · "/ 0. 1 2 %$ 3 ( 0 4 5 6 2 3 ( 5789 578: 2

VnDoc - Tải tài liệu, văn bản pháp luật, biểu mẫu miễn phí

ANSWER KEYSA. LISTENING:

(Phần nghe skill 2 -listening 1 -unit 11)Mỗi câu đúng ghi được 0.25 điểm

1. 6A 5. 6B

2. recycling 6. bus

3. saving lights 7. gardening

4. old books 8. uniforms

B. USE OF ENGLISHI/ Find the word which has a different sound in the underlined part (1pt)

Mỗi câu đúng ghi được 0.25 điểm

1. C 2. A 3. B 4. D

II/Choose the best answer by circling A, B, C or D: (1pt)Mỗi câu đúng ghi được 0.25 điểm

1. C 2. D 3. B 4. A

C. READING

I. Choose A, B, C or D to complete the following passage: (1.0 pt)Mỗi câu đúng ghi được 0.25 điểm

1. A 2. A 3. D 4. B

II/ Read the passage carefully then answer the questions below. (1.0 pt)

Mỗi câu đúng ghi được 0.25 điểm

1. He wishes to have a motorhome.

2. There are three parts in the house.

3. No, it won’t.

4. Yes, it is.

D.WRITING (2,0pts)

Mỗi câu đúng ghi được 0.25 điểm

1. If we recycle bottles and cans, we will help the environment.

2. Could you read and write when you were 6?

3. In 2030, robots will be able to do many things like humans.

Page 5: Tài liệu, học tập, trắc nghiệm, tiếng anh, văn bản, biểu ...s1.vndoc.com/.../06/40-de-thi-hoc-ky...6-co-dap-an.pdf · "/ 0. 1 2 %$ 3 ( 0 4 5 6 2 3 ( 5789 578: 2

VnDoc - Tải tài liệu, văn bản pháp luật, biểu mẫu miễn phí

4. We visited our grandparents in the countryside last week.

II/.Write a short paragraph about your dream house, using the cues given to help your writing.(1,0pt)

Mỗi câu đúng ghi được 0.25 điểm

Tùy vào viết của học sinh giáo viên cho điểm.

Page 6: Tài liệu, học tập, trắc nghiệm, tiếng anh, văn bản, biểu ...s1.vndoc.com/.../06/40-de-thi-hoc-ky...6-co-dap-an.pdf · "/ 0. 1 2 %$ 3 ( 0 4 5 6 2 3 ( 5789 578: 2

VnDoc - Tải tài liệu, văn bản pháp luật, biểu mẫu miễn phí

PHÒNG GD&ĐT BẢO LỘC----------------------

ĐỀ KIỂM TRA HỌC KÌ 2NĂM HỌC 2015 - 2016

MÔN: TIẾNG ANH – LỚP 6

Thời gian làm bài 45 phút

I. Vocabulary and GrammarCircle the best option A, B, or C to complete each of the following sentences.

(2.5 points)1. I’m ____. I’d like some noodles.

A. thirsty B. hungry C. empty

2. There is____ water in the bottle.

A. any B. many C. some

3. What’s he doing? He’s ____.

A. swims B. swimming C. swim

4. We go to the movies____ Sunday evening.

A. to B. at C. on

5. It’s very ____ in the winter.

A. cold B. hot C. cool

6. They are doing ____ homework at the moment.

A. them B. theirs C. their

7. He sometimes____ to pop music.

A. listens B. reads C. watches

8. – How ____ is a sandwich? – It’s 2,000 dong.

A. many B. much C. old

9. – How____ does she go jogging? – Every evening.

A. many B. much C. often

10. ____ is the weather like in the summer?

A. What B. Which C. How

II. ReadingA. Fill in the blanks with the words from the box (2.5 points)

Page 7: Tài liệu, học tập, trắc nghiệm, tiếng anh, văn bản, biểu ...s1.vndoc.com/.../06/40-de-thi-hoc-ky...6-co-dap-an.pdf · "/ 0. 1 2 %$ 3 ( 0 4 5 6 2 3 ( 5789 578: 2

VnDoc - Tải tài liệu, văn bản pháp luật, biểu mẫu miễn phí

much to natural picnicgo they tent sometimes

Huong and Hoa are close friends. They often (1)____ to the movie theater at the weekend.

They (2) _______ go to the zoo. They also have a (3) ______, but not always. They go (4)

____ the countryside. They always take a (5) ____, some water and a camping stove.

Sometimes, (6) ____ camp overnight. They like camping very (7) ____. They can enjoy

the fresh air and the (8) ____ beauty .

B. Read the passage and do the tasks that follow.

My uncle is an engineer. His name is Hung. He is going on vacation this summer.

First, he is going to visit Ha Long Bay. He is going to stay in a small hotel near the bay

for two days. Then he is going to visit Da Lat for three days. Finally, he is going to

visit some friends in Ho Chi Minh City . He is going to stay there for one day . He is

going to walk along Saigon River. He is going to fly home.

Decide if these statements are True (T) or False (F) (1.0point)

T F

1. Mr. Hung is going to visit three cities this summer.

2. Hẹ is going to visit Ha Long Bay first, then Ho Chi Minh City, and

finally Da Lat.

3. He is going to stay in Da Lat for three days.

4. He is going to travel home by train.

Answer the following questions (1.0point)

1. Where is Mr. Hung going to stay in Ha Long Bay?

………………………………………………………………

2. Is he going to visit some friends in Ho Chi Minh City?

………………………………………………………………

III. WritingA. Match a sentence in column A with a sentence in column B (2.0 points)

Page 8: Tài liệu, học tập, trắc nghiệm, tiếng anh, văn bản, biểu ...s1.vndoc.com/.../06/40-de-thi-hoc-ky...6-co-dap-an.pdf · "/ 0. 1 2 %$ 3 ( 0 4 5 6 2 3 ( 5789 578: 2

VnDoc - Tải tài liệu, văn bản pháp luật, biểu mẫu miễn phí

A B

1. How do you feel? a. Two kilos.

2. What’s your favorite sport? b. I feel happy.

3. How much meat do you need? c. Yes, it is.

4. Is Hanoi smaller than Ho Chi Minh City? d. It’s soccer.

Your answers:

1- ….. 2 – ….. 3 – ….. 4 – …..

B. Rearrange these sets of words to make complete sentences (l.0 point)1. very/ my/ is/ beautiful/ country

2. smaller/ is/ house/ my/ than / Lan’s house.

———– THE END———–

Page 9: Tài liệu, học tập, trắc nghiệm, tiếng anh, văn bản, biểu ...s1.vndoc.com/.../06/40-de-thi-hoc-ky...6-co-dap-an.pdf · "/ 0. 1 2 %$ 3 ( 0 4 5 6 2 3 ( 5789 578: 2

VnDoc - Tải tài liệu, văn bản pháp luật, biểu mẫu miễn phí

TRƯỜNG THCS HUỲNH THÚC KHÁNG----------------------------------

Họ và tên: ………………………………….

Lớp: …………………………………

ĐỀ KIỂM TRA SỐ 4 HỌC KỲ 2NĂM HỌC 2015 - 2016

MÔN: TIẾNG ANH - LỚP 6 THÍ ĐIỂM

Thời gian: 45 phút

A. LISTENING(2,5pts):Question 1: Listen and number the pictures from 1 to 6. (1.25pts)

a. ………… b. ……………. c. ……………..

d. …………… e. ……………..

Question 2: Listen and tick T/ F (1,25pts):

T F

1. Mi thinks she’ll put a recycling bin in every classroom.

2. Then she’ll buy energy-saving lights for every school.

3. Nam is from class 6A.

4. He thinks that it will be good if students go to school by bus

5. Finally, he’ll organize some uniform fairs.

B. READING: (2,5pts)Question 1: Read the text and answer the questions (1,5pts):

I always wish to have a motorhome in my life. My motorhome will not need to be big, but it might

have everything I need inside.

There will be a small sofa and a lovely table in the head of the car. This will be the place for my

friends when they visit me. It’s like a living room. The biggest part of my motorhome will be a

kitchen. It will be in the middle of the car. There will be an automatic dishwasher, a modern fridge,

a small oven and an automatic washing machine in the kitchen. A small toilet will be in the kitchen

Page 10: Tài liệu, học tập, trắc nghiệm, tiếng anh, văn bản, biểu ...s1.vndoc.com/.../06/40-de-thi-hoc-ky...6-co-dap-an.pdf · "/ 0. 1 2 %$ 3 ( 0 4 5 6 2 3 ( 5789 578: 2

VnDoc - Tải tài liệu, văn bản pháp luật, biểu mẫu miễn phí

too. The last part will be my bedroom, also my entertainment place. There will be a small bed, a

wireless TV and a hi-tech computer.

Everything might be small in my motorhome. It would be a fantastic if I had my own motorhome.

The best thing about having a motorhome is that I can drive it myself and travel around.

1. What kind of house do you wish to have?

……………………………………………………………………………………………

2. How many rooms are there in the house?

………………………………………….. ……………………………………………….

3. Will a big toilet be in the living room?

……………………………………………………………………………………………..

Question 2: Choose the correct word marked A, B, C, D to fill each blank in the following

passage (1pt)AIR QUALITY

People need to breathe. If they don’t breath, they will die. But how clean is the air people breathe?

If they breathe (1) ………………air, they will have breathing problems and become ill. Plants and

animals need clean air too. A lot of things (2) ………………. our lives create harmful gasses and

make the air dirty, like cars, motorbikes and factories. Dirty air is called “ (4) …………… air”. Air

pollution can also make our Earth (4) ……………. The problem of air pollution started with the

burning of coal in homes and factories.

1. A. clean B. fresh C. dirty D. green

2. A. at B. in C. to D. on

3. A. polluted B. polluting C. pollution D. pollute

4. A. warm B. bigger C. beautiful D. warmer

C. WRITING (2,5pts)I. Do as directed in brackets. (1,5pts)

1. recycle/ the environment. /bottles and cans/ we will help / If we/ (put the words in the correct

order)

...........................................................................................................................................

2. When I see Mary tomorrow, I (invite) ………………… to our party. (Use the correct form the

verb in the bracket)

3. We use recycled paper. We save trees. (Combine sentences to make a conditional sentence- type

1)

..........................................................................................................................................

II. Answer the following questions (1pts)

Page 11: Tài liệu, học tập, trắc nghiệm, tiếng anh, văn bản, biểu ...s1.vndoc.com/.../06/40-de-thi-hoc-ky...6-co-dap-an.pdf · "/ 0. 1 2 %$ 3 ( 0 4 5 6 2 3 ( 5789 578: 2

VnDoc - Tải tài liệu, văn bản pháp luật, biểu mẫu miễn phí

1. Where will your house be in the future?

……………………………………………………………………………………………………

2. What will you do at school to help the environment?

……………………………………………………………………………………………………

D. LANGUAGE FORCUS (2.5pts)

I. Choose the best answer (1,5pts)1. The people here are ………………

A. friendly B. delicious C. polluted D. long

2. He likes playing table tennis, ………………. he can’t play it.

A. and B. but C. so D. or

3. In the future, we won’t go on holiday to the beach but we …… go on holiday to the moon.

A. must B. can’t C. might D. won’t

4. If we cut down more forests, there …………………more floods.

A. Are B. Were C. have been D. will be

5. Find the word which has a different sound.

A. clothing B. brother C. thick D. there

A. ask B. plastic C. back D. grandparent

II. Matching the causes in column A with the effects in column B(1pt)

A B Answer

a. Air pollution 1. Causes hearing problem. a…………

b. Water pollution 2. Causes breathing problem. b………..

c. Noise pollution 3. Make plants die. c………..

d. Soil pollution 4. Makes fish die. d………..

Page 12: Tài liệu, học tập, trắc nghiệm, tiếng anh, văn bản, biểu ...s1.vndoc.com/.../06/40-de-thi-hoc-ky...6-co-dap-an.pdf · "/ 0. 1 2 %$ 3 ( 0 4 5 6 2 3 ( 5789 578: 2

VnDoc - Tải tài liệu, văn bản pháp luật, biểu mẫu miễn phí

PHÒNG GD&ĐT MƯỜNG ẢNGTRƯỜNG THCS ẲNG CANG

-------------------------Họ và tên:…………………………

Lớp:……………. STT:…………...

ĐỀ KIỂM TRA ĐỊNH KÌ BÀI SỐ 3Năm học 2015 - 2016

Môn: Tiếng Anh - Lớp 6Thời gian làm bài: 45 phút (không kể thời gian giao đề)

(Học sinh làm bài trực tiếp trên tờ giấy này)

Điểm GV chấm Lời phê của Thầy Cô giáo Ký duyệt

Bằng số:

Bằng chữ:

Đề: 1 - Mã 01A. LISTENING

Part 1. Listen and choose the correct answers. (1,25 điểm)

Câu 1:What do we need for lunch?

A. vegetables B.meat C. fruit D. cabbageCâu 2:What does your mom give you?

A. a bowl of rice B. a bowl of noodlesC. two bowls of soup D. a bowl of soup

Câu 3:Who does he often go camping with?A. his wife B. his family C. his friends D. his parents

Câu 4:What does the sign say?A. “Go ahead”.B. “Turn right”. C. “Turn left”. D. “Go around”.

Câu 5:What does his oldest brother always drink at the family’s parties?A. milk B. wine C. juice D. beer

Part 2. Listen and tick ( ) the box. (1,25 điểm)

Câu 6:A. Can you see the man in the fur hat?

B. Can you see the man in the flat?C. Can you see the man with his cat?

D. Can you see the man in the jacket?Câu 7:

Page 13: Tài liệu, học tập, trắc nghiệm, tiếng anh, văn bản, biểu ...s1.vndoc.com/.../06/40-de-thi-hoc-ky...6-co-dap-an.pdf · "/ 0. 1 2 %$ 3 ( 0 4 5 6 2 3 ( 5789 578: 2

VnDoc - Tải tài liệu, văn bản pháp luật, biểu mẫu miễn phí

A.When is this holiday?B.When is his birthday?

C.When is she leaving?D.When will she come?

Câu 8:A. Listen, and color, and repeat, and draw.

B. Listen, and color, and drive, and draw.C. Listen, and color, and read, and draw.

D. Listen, and color, and write, and draw.Câu 9:

A. Can I color the one behind the ski?B. Can I color the one next to the ski?

C. Can I color the one next to the sea?D. Can I color the one next to the tree?

Câu 10:A.What did Theresa do on Friday evening?

B.What did Richard do on Friday evening?C.What did Richard do on Sunday evening?

D.What did Charles do on Friday afternoon?B. LANGUAGE FOCUS AND VOCABULARIES.

I. Circle the best option A, B, C or D to complete each of the following sentences. (2,0 điểm)

Câu 11: I am so thirsty. I would like some _______, please.

A. bread B. orange juice C. biscuits D. riceCâu 12:My sisters never _______ any vegetables.

A. eat B. to eat C. eating D. eatsCâu 13:What do you _______ for breakfast?

A. to have B. have C. has D. havingCâu 14:What color is her hair? – It is _______.

A. short B. long C. black D. curlyCâu 15: I’m very hungry. I’d like some ________, please.A. water B. Coca Cola C. biscuits D. orange juice

Câu 16:My mother is very tall but my father is very _______.A. cold B. thin C. fat D. short

Câu 17: _______ you like a cup of tea?

Page 14: Tài liệu, học tập, trắc nghiệm, tiếng anh, văn bản, biểu ...s1.vndoc.com/.../06/40-de-thi-hoc-ky...6-co-dap-an.pdf · "/ 0. 1 2 %$ 3 ( 0 4 5 6 2 3 ( 5789 578: 2

VnDoc - Tải tài liệu, văn bản pháp luật, biểu mẫu miễn phí

A. Are B.Would C. Does D. DoCâu 18: _______ apples do you want?

A. How much B. How long C. How far D. How manyII. Find only mistake (A,B,C or D) that underlined in each sentence (0,5điểm)

Câu 19:My mother is very tall and she has full eyes.A B C D

Câu 20: Can you see the girl with tall hair over there?A B C D

C. READINGI. Read the passage and fill in each blank with a suitable phrase, clause or sentence from the box.

There are more phrase/ clauses/ sentences than needed. (1,25 điểm)

A. Saturday evenings

B. we go to bed

C. we go to bed very early

D. half past seven

E. noodles and coffee or orange juice

F. we get up at seven

G. tea or coffee after the meal

We have breakfast at (1)_______. We don’t have a big breakfast because we have to go to

work very early. We usually have (2)_______.

We have dinner at about seven. It is the big meal of the day and we have meat or fish with

vegetables and potatoes or rice. We drink (3)_______. We have a glass of milk before (4)_______.

On (5)_______ we usually go to a restaurant for dinner at about eight o’clock.

II. Read the following passage and decide whether the following statements are true (T) or false

(F). (1,25 điểm)

My sister, Lien, is a teacher. She is very tall with an oval face. She has long brown hair and

round eyes. She is very beautiful with full lips and a small nose.

She likes vegetables and she eats a lot of them. She doesn’t eat much meat or sugar. She never

smokes or drinks wine. In her free time, she usually goes out for a walk with her friends.

T F

Câu 1: Lien is good-looking.

Câu 2: She is not very tall.

Page 15: Tài liệu, học tập, trắc nghiệm, tiếng anh, văn bản, biểu ...s1.vndoc.com/.../06/40-de-thi-hoc-ky...6-co-dap-an.pdf · "/ 0. 1 2 %$ 3 ( 0 4 5 6 2 3 ( 5789 578: 2

VnDoc - Tải tài liệu, văn bản pháp luật, biểu mẫu miễn phí

Câu 3: She eats a lot of vegetables.

Câu 4: She eats a lot of meat.

Câu 5: She usually stays at home in her free time.

D. WRITINGI. Write complete sentences using the words given. (1,25 điểm)

Câu 1: Is there/ milk/ in/ bottle?______________________________________________________________________

Câu 2: How many/ orange/ would/ like?______________________________________________________________________

Câu 3:My favorite food/ cheese.______________________________________________________________________

Câu 4: Carrots/ good for you.______________________________________________________________________

Câu 5:My mother/ have/ short hair/ brown eyes.______________________________________________________________________

II. Write about what you eat/ drink every day. (1,25 điểm)

Câu 1:What you often eat for breakfast______________________________________________________________________

Câu 2:What you often eat for lunch______________________________________________________________________

Câu 3:What you often eat for dinner______________________________________________________________________

Câu 4:What you often drink______________________________________________________________________

Câu 5:What you never eat______________________________________________________________________

----------- HẾT ----------

Page 16: Tài liệu, học tập, trắc nghiệm, tiếng anh, văn bản, biểu ...s1.vndoc.com/.../06/40-de-thi-hoc-ky...6-co-dap-an.pdf · "/ 0. 1 2 %$ 3 ( 0 4 5 6 2 3 ( 5789 578: 2

VnDoc - Tải tài liệu, văn bản pháp luật, biểu mẫu miễn phí

ANSWER KEYSA. LISTENINGPart 1. Listen and choose the correct answers. (Mỗi câu đúng 0,25 điểm)

1 2 3 4 5

A

B

C

D

Part 2. Listen and tick ( ) the box. (Mỗi câu đúng 0,25 điểm)

6 7 8 9 10

A

B

C

D

B. LANGUAGE FOCUS AND VOCABULARIES.

Từ câu 11 đến câu 20 (Mỗi câu đúng 0,25 điểm)

11 12 13 14 15 16 17 18 19 20

A

B

C

D

D. READING (Mỗi câu đúng 0,25 điểm)

I. 1. D 2. E 3. G 4. B 5. A

II. 1. T 2. F 3. T 4. F 5. F

C. WRITING (Mỗi câu đúng 0,25 điểm)I. 1. Is there any milk in the bottle?

2. How many oranges would you like?

3. My favorite food is cheese.

4. Carrots are good for you.

5. My mother has short hair and brown eyes.

Page 17: Tài liệu, học tập, trắc nghiệm, tiếng anh, văn bản, biểu ...s1.vndoc.com/.../06/40-de-thi-hoc-ky...6-co-dap-an.pdf · "/ 0. 1 2 %$ 3 ( 0 4 5 6 2 3 ( 5789 578: 2

VnDoc - Tải tài liệu, văn bản pháp luật, biểu mẫu miễn phí

II. 1. I often eat ............. for my breakfast.

2. I often have ................ for lunch.

3. I often eat ................... for dinner.

4. I often drink ........................

5. I never eat ..........................

----------- HẾT ----------

Page 18: Tài liệu, học tập, trắc nghiệm, tiếng anh, văn bản, biểu ...s1.vndoc.com/.../06/40-de-thi-hoc-ky...6-co-dap-an.pdf · "/ 0. 1 2 %$ 3 ( 0 4 5 6 2 3 ( 5789 578: 2

VnDoc - Tải tài liệu, văn bản pháp luật, biểu mẫu miễn phí

TRƯỜNG THCS HỒ TÙNGMẬU-----------------

Họ và tên: ...................................................

Lớp: 6 .....

ĐỀ KIỂM TRA SỐ 3 HỌC KỲ 2NĂM HỌC 2015 - 2016

MÔN: TIẾNG ANH - LỚP 6

Thời gian: 45 phút

Ngày thi: Thứ 7, 20/02/2016

Mark Teacher’s remark

PART 1 - LISTENING.1. Listen to the dialogue and write true or false (1pt) (Nghe đoạn hội thoại và viết câu đúng (T),

câu sai (F))a. There is some meat and some noodles for lunch. ……………

b. There are some oranges and some bananas. ……………

c. There is some water. ……………

d. There isn’t any milk. ……………

2. Listen twice about Chi and fill in the blanks with the words from the box. Number(0) is an

example (1pt) (Nghe đoạn văn về Chi và điền vào chỗ trống với những từ trong khung)

weak gymnast light tall thin

Chi is a (0) gymnast. She is(1) ………...... . She is (2)……...... . She (3)………….. But she is

not(4)………….. . She is strong.

PART 2 – LANGUAGE FOCUS1. Odd one out (1pt) (Chọn từ khác loại)

a. A.carrot B. matter C. cabbage D. apple

b. A. tired B. drink C. full D. tall

c. A.by B. hungry C. short D. thirsty

Page 19: Tài liệu, học tập, trắc nghiệm, tiếng anh, văn bản, biểu ...s1.vndoc.com/.../06/40-de-thi-hoc-ky...6-co-dap-an.pdf · "/ 0. 1 2 %$ 3 ( 0 4 5 6 2 3 ( 5789 578: 2

VnDoc - Tải tài liệu, văn bản pháp luật, biểu mẫu miễn phí

d. A.body B. ear C. hair D. nose

2. Underline the right answer (1pt) (Gạch chân đáp án đúng)a. Lan isn’t short. She is ……………….… . (long, tall, fat, thin)

b. There is not .................. water in the bottle. (a/ an/ some/ any)

c. What ………..…. he like? - He would like a cake. (do, would, is, are)

d. …………….. is an iced-tea? – It’s 5.000đ. (How much, How many, How, What)

3. Put these verbs in the correct form (1pt) (Đặt động từ theo dạng đúng)

a. She .................... a round face. (have)

b. There ..................... some water in the bottle. (be)

c Would you ................. some oranges? (like)

d. How does Nam feel? - He ................ cold. (feel)

PART 4 - READING1. Reading 1: Read the passage.

Ly is a beautiful girl with long black hair. She is tall and slim. She has an oval face. Her

favorite food is beef. She has bread and milk for breakfast every day. She likes juice and ice-cream.

Her parents’ favourite drinks are milk and orange juice. They like them very much. She is going

to market to buy food for lunch with her sister now. Today, they have fish, vegetables and potatoes for

lunch. They buy some oranges and apples, too. They are their favorite fruits.

* Complete these sentences with suitable words from the passage. (Đọc đoạn văn sau đó hoàn

thành câu với những từ thích hợp từ đoạn văn) (1pt)a. Huong has ………….............. hair

b. She is …………………….. and slim.

c. ................................ is her favorite drink..

d. Her parent’s favourite drinks are ........................ and orange juice.

2. Reading 2: Read the passage then answer the questions. (2pts)(Đọc đoạn văn và trả lời câu hỏi)

Today is Sunday. She is having a small party tonight, so Mrs Hoa goes to the market in the morning.

She needs many things. She wants some meat, one kilo of fish, five hundred grams of beef. And she

wants some vegetables. She needs three cans of peas and one kilo of tomatoes. She also wants ten cans

of soda and four bottles of orange juice. She goes home by her car.

* Questions:

a. Where does Mrs Hoa go in the morning?

Page 20: Tài liệu, học tập, trắc nghiệm, tiếng anh, văn bản, biểu ...s1.vndoc.com/.../06/40-de-thi-hoc-ky...6-co-dap-an.pdf · "/ 0. 1 2 %$ 3 ( 0 4 5 6 2 3 ( 5789 578: 2

VnDoc - Tải tài liệu, văn bản pháp luật, biểu mẫu miễn phí

=>…………………………………………………………………………………….

b. How many kilos of fish does she want?

=>…………………………………………………………………………………….

c. Does she want five kilos of beef?

=>……………………………………………………………………………………..

d. How much orange juice does she want?

=>……………………………………………………………………………………..

PART 5 - WRITING1. Put the words in the correct order to make sentences (1pt) (Đặt từ theo trật tự đúng để tạo

thành câu hoàn chỉnh)a. mother / my / tall / is / and / thin.

=> …………………………………………………………………………………..

b. has / Hoa / oval / an / face.

=>………………………………………………………………….………………..

c. need / I / a / toothpaste / of / tube.

=> …………………………………………………………………………………..

d. eggs / How many / want / do / you / ?

=> ………………………………………………………………………………….

2. Make question for under part in these sentences? (1pt) (Đặt câu hỏi cho phần được gạch chân)

a. Hung feels cold.

=>……………………………………………………………………………………

b. I want some hot drinks.

=>...............................................................................................................................

c. Yes, there is some lemonade.

=>................................................................................................................................

d My favorite drink is soda.

=>................................................................................................................................

- The end -

Page 21: Tài liệu, học tập, trắc nghiệm, tiếng anh, văn bản, biểu ...s1.vndoc.com/.../06/40-de-thi-hoc-ky...6-co-dap-an.pdf · "/ 0. 1 2 %$ 3 ( 0 4 5 6 2 3 ( 5789 578: 2

VnDoc - Tải tài liệu, văn bản pháp luật, biểu mẫu miễn phí

TRƯỜNG THCS TẢO DƯƠNG VĂN---------------------

Name: …………………………Class : 6…..

ĐỀ THI HỌC KỲ 2NĂM HỌC: 2015 - 2016

MÔN: TIẾNG ANH - LỚP 6Thời gian làm bài: 45 phút

I. PHONETICS: Choose the word that has the underlined part pronounced differentlyfrom the rest. (1,5pt)

1. A. camp B. plan C. plane D. badminton

2. A. sky B. thirsty C. hungry D. finally

3. A. ride B. night C. river D. tired4. A. window B. flow C. how D. know

5. A. engineer B. geography C. language D. great6. A. lettuce B. tube C. music D. use

II. GRAMMAR AND VOCABULARY

Circle the letter A, B, C or D as your best choice to complete each sentence. (2pts)1. My sister is a nurse. She works in the ………

A. hospital B. factory C. farm D. School

2. They ………….. not buy books.

A. are B. do C. does D. am

3. How much _________do you want ?

A. eggs B. tea C. oranges D. apples

4. How __________ do you go jogging ? - Once a day.

A. usually B. often C. never D. always

5. There is __________bread on the table .

A. a B. an C. some D. two

6. I like _________ . I have a rope.

A. skip B. skips C. skiping D. skipping

7. __________________go to Da Lat ?

A. Let's B. Why don’t we C. What about D. How about

8. I like fruit. I like _____________very much.

Page 22: Tài liệu, học tập, trắc nghiệm, tiếng anh, văn bản, biểu ...s1.vndoc.com/.../06/40-de-thi-hoc-ky...6-co-dap-an.pdf · "/ 0. 1 2 %$ 3 ( 0 4 5 6 2 3 ( 5789 578: 2

VnDoc - Tải tài liệu, văn bản pháp luật, biểu mẫu miễn phí

A. carrots B. tomatoes C. peas D. apples

Give the correct verb forms or tenses. (1,5 pt)

1. My school (have)……………………two floors.

2 She (drink)...........................a lot of juice every day.

->………………………………………………………………………………………

3. Lan and Nga (see) ....................................a new film next week.

->………………………………………………………………………………………

4. What about (visit)…………………………………..our parents?

5. He (ride)………….....his bicycle every day. He (ride)……………...his bicycle now

->………………………………………………………………………………………

III.READING: Read the following passage and then do the tasks that follow. (2pts)

South Vietnam enjoys a tropical climate, so there are two seasons all year round. They are

known as the dry season and the rainy season. It’s very hot in the dry season, but it’s cool in the

rainy season. In the dry season, we often go camping and swimming. In the rainy season we don’t

often go out. We sometimes go fishing in the morning because it often rains in the afternoon.

A. Tick True or False.

FACTS T F

1. There are two seasons in the south of Vietnam.

2. It’s very cold in the dry season.

3. They often go camping and swimming.

4. in the rainy season, it often rains in the afternoon.

B. Answer the following sentences. (1pt)

1. How many seasons are there in South Vietnam?

................................................................................................................

2. What do they do in the rainy season?

................................................................................................................

3. Is it cool in the dry season?

...............................................................................................................

4. Why do they go fishing in the morning?

Page 23: Tài liệu, học tập, trắc nghiệm, tiếng anh, văn bản, biểu ...s1.vndoc.com/.../06/40-de-thi-hoc-ky...6-co-dap-an.pdf · "/ 0. 1 2 %$ 3 ( 0 4 5 6 2 3 ( 5789 578: 2

VnDoc - Tải tài liệu, văn bản pháp luật, biểu mẫu miễn phí

....................................................................................................................

IV. WRITING:Write complete sentences using the suggested words. (2pts)

1. Ba/always/walk/school.

......................................................................................................................

2. The Mekong River/long/than/the Red River.

.....................................................................................................................?

3. Ho Chi Minh City / large /than / Hanoi.

......................................................................................................................

4. How / rice /and /many/ vegetables / he/ produce?

......................................................................................................................

Page 24: Tài liệu, học tập, trắc nghiệm, tiếng anh, văn bản, biểu ...s1.vndoc.com/.../06/40-de-thi-hoc-ky...6-co-dap-an.pdf · "/ 0. 1 2 %$ 3 ( 0 4 5 6 2 3 ( 5789 578: 2

VnDoc - Tải tài liệu, văn bản pháp luật, biểu mẫu miễn phí

ANSWER KEYSI. PHONETICS: (1,5 pt)

1 2 3 4 5 6

B. plan A. sky C. river C. how great lettuce

II. GRAMMAR AND VOCABULARY

Circle the letter A, B, C or D as your best choice to complete each sentence. (2pts)

1 2 3 4 5 6 7 8

A. hospital B. do B. tea B. often C. some D. skipping B.

Why don’t we

D.

apples

Give the correct verb forms or tenses. (1,5pt)

1 2 3 4 5 6

has drinks Are going to see visiting rides Is riding

III. READING:

A. Tick True or False. (1pt)

1 2 3 4

T F T T

B. Answer the following sentences. (2pts)

1. There are two seasons in South Vietnam.

2. In the rainy season, they don’t often go out. They sometimes go fishing.

3. Yes, it isn’t.

4. Because it often rains in the afternoon.

IV. WRITING: (2pts)1. Ba always walks to school

2. The Mekong River is longer than the Red River.

3. Ho Chi Minh City is larger than Hanoi.

4. How many vegetables and rice does he produce?

Page 25: Tài liệu, học tập, trắc nghiệm, tiếng anh, văn bản, biểu ...s1.vndoc.com/.../06/40-de-thi-hoc-ky...6-co-dap-an.pdf · "/ 0. 1 2 %$ 3 ( 0 4 5 6 2 3 ( 5789 578: 2

VnDoc - Tải tài liệu, văn bản pháp luật, biểu mẫu miễn phí

PHÒNG GD&ĐT CAM LỘ-------------------------

ĐỀ CHÍNH THỨC

KIỂM TRA HỌC KỲ IINăm học: 2013-2014

Môn: TIẾNG ANH – LỚP 6Thời gian: 45 phút (không kể thời gian phát đề)

SECTION I: PHONETICS (1pt)

1. A. lamp B. table C. family D. plan2. A. skip B. swim C. fried D. milk

3. A. hungry B. sky C. healthy D. thirsty4. A. books B erasers C. rulers D. bags

SECTION II: VOCABULARY, GRAMMAR AND STRUCTURES

Part 1: Choose the best answer (2,5pts)1. How …………… kilos of beef does she want?

A. many B. much C. often D. about

2. His father is waiting ……………. a bus.

A. to B. for C. of D. at

3. We go there by car and they go ……… foot.

A. on B. by C. to D. with

4. Listen! Who ……… to your sister?

A. is going to talk B. talks C. does talk D. is talking

5. Fall means ……… in British English.

A. summer B. winter C. spring D. autumn

6. Tokyo is ………….. than Mexico City.

A. smaller B. smallest C. the smallest D. small

7. I don’t have ……… apples but I have ……………oranges.

A. any/some B. any/any C. some/any D. some/some

8. What color …………….her eyes?

A. is B. are C. do D. does

9. ………………… is a pen?. It is 5,000 dong.

A. How many B. How C. How often D. How much

10. She isn’t strong. She is ……………

A. weak B. tall C. short D. heavy

Page 26: Tài liệu, học tập, trắc nghiệm, tiếng anh, văn bản, biểu ...s1.vndoc.com/.../06/40-de-thi-hoc-ky...6-co-dap-an.pdf · "/ 0. 1 2 %$ 3 ( 0 4 5 6 2 3 ( 5789 578: 2

VnDoc - Tải tài liệu, văn bản pháp luật, biểu mẫu miễn phí

Part 2: Supply the correct form of verbs: (1,5pts)1. I (write) ………to my parents now, I (write) ……. to them every weekend.

2. You (go) ……………..to the party tonight?

3. Mary (listen)…………….to music at the moment.

4. He (want) …………….(be) ………….a doctor.

SECTION III: SPEAKINGMatch the question in column A with the right answer in column B. (1pt)

A B

1. What weather does she like?

2. How often do you go shopping?

3. How do you go to school?

4. Why don’t we go to the beach?

a. I walk to school.

b. Good idea.

c. Once a week.

d. She likes warm weather.

1…………… 2………………….. 3………………… 4…………………

SECTION IV: READING (2pts)

Read the text and decide if these statements are True (T) or False (F)My uncle is an engineer. His name is Hung. He is going on vacation this summer. First, he is

going to visit Ha Long Bay. He is going to stay in a small hotel near the bay for two days. Then he

is going to visit Da Lat for three days. Finally, he is going to visit some friends in Ho Chi Minh City.

He is going to stay there for one day. He is going to walk along Saigon River. He is going to fly

home.

True(T)/ False(F) statements:1……. Mr. Hung is going to visit three cities this summer.

2. ……He is going to visit Ha Long Bay first, then Ho Chi Minh City, and finally Da Lat.

3. ……He is going to stay in Da Lat for three days.

4……..He is going to go home by train.

SECTION V: WRITING (2pts)Arrange the words into the sentence:1. want / Tom / Mary / and / glasses / two / of / water.

.....................................................................................................................................

2. sister / my / going / is / to / live / country / the / in.

.....................................................................................................................................

Page 27: Tài liệu, học tập, trắc nghiệm, tiếng anh, văn bản, biểu ...s1.vndoc.com/.../06/40-de-thi-hoc-ky...6-co-dap-an.pdf · "/ 0. 1 2 %$ 3 ( 0 4 5 6 2 3 ( 5789 578: 2

VnDoc - Tải tài liệu, văn bản pháp luật, biểu mẫu miễn phí

3. the / spring / in / what / weather / is / like / the?

....................................................................................................................................

4. building / tallest / the / is / this / the / in / world.

.....................................................................................................................................

The end

Page 28: Tài liệu, học tập, trắc nghiệm, tiếng anh, văn bản, biểu ...s1.vndoc.com/.../06/40-de-thi-hoc-ky...6-co-dap-an.pdf · "/ 0. 1 2 %$ 3 ( 0 4 5 6 2 3 ( 5789 578: 2

VnDoc - Tải tài liệu, văn bản pháp luật, biểu mẫu miễn phí

PHÒNG GD&ĐT CAM LỘ---------------------

HƯỚNG DẪN CHẤMĐỀ KIỂM TRA HỌC KÌ II - NĂM HỌC 2013 – 2014

Môn: TIẾNG ANH - LỚP 6

SECTION I: PHONETICS (1pt)

1. B 2. C 3. B 4. A

SECTION II: VOCABULARY, GRAMMAR AND STRUCTURES

Part 1: Choose the best answer (2,5pts)1. A 2. B 3. A 4. D 5. D

6. A 7. A 8. B 9. D 10. A

Part 2: Supply the correct form of verbs: (1,5pts)

1. am writing / write

2. Are you going

3. is listening

4. wants/ to be

SECTION III: SPEAKINGMatch the question in column A with the right answer in column B. (1pt)

1. d 2. c 3. a 4. b

SECTION IV: READING (2pts)

Read the text and decide if these statements are True (T) or False (F)1. T 2. F 3. T 4. F

SECTION V: WRITING (2pts)Arrange the words into the sentence:

1. Tom and Mary want two glasses of water.

2. My sister is going to live in the country.

3. What is the weather like in the spring?

4. This is the tallest building in the world.

The end

Page 29: Tài liệu, học tập, trắc nghiệm, tiếng anh, văn bản, biểu ...s1.vndoc.com/.../06/40-de-thi-hoc-ky...6-co-dap-an.pdf · "/ 0. 1 2 %$ 3 ( 0 4 5 6 2 3 ( 5789 578: 2

VnDoc - Tải tài liệu, văn bản pháp luật, biểu mẫu miễn phí

PHÒNG GD&ĐT ỨNG HÒATRƯỜNG THCS NGUYỄN THƯỢNG HIỀN

----------------Họ và tên: ....................................................

Lớp: 6 ......

ĐỀ KIỂM TRA HỌC KỲ 2NĂM HỌC 2015 - 2016

MÔN: TIẾNG ANH - LỚP 6 THÍ ĐIỂMThời gian làm bài: 45 phút

ĐỀ SỐ 1

Mark Remark

Part A: Listening (2,0ms)

I. Mi and Nam are talking about what they will do if they become the president of the 3Rs club.Listen and fill one word in each gap to complete the table for their ideas. (1.0m)

Mi’s ideasPut a recycling bin in every (1) .………………………..………

(2) ………………………..…… energy - saving lights.

Nam’s ideas(3)…………………….………..students to go to school by bus.

Set up a gardening (4) …………………………….

II. Listen again and write True (T) or False (F). (1.0m)

Statements T - F

1. Mi is from class 6A and Nam is from class 6E.

2. Mi thinks, she will organize a few book fairs.

3. At book fairs, students can swap their new books.

4. Students can grow vegetables in the school garden.

1. ………….

2. ………….

3. ………….

4. ………….

PART B: Phonetic

I. Choose the word that has the underlined part pronounced differently from the others (1m)1..A.call B. fall C. math D. wall

2.A.clear B. dear C. hear D. pear

Page 30: Tài liệu, học tập, trắc nghiệm, tiếng anh, văn bản, biểu ...s1.vndoc.com/.../06/40-de-thi-hoc-ky...6-co-dap-an.pdf · "/ 0. 1 2 %$ 3 ( 0 4 5 6 2 3 ( 5789 578: 2

VnDoc - Tải tài liệu, văn bản pháp luật, biểu mẫu miễn phí

3.A. come B. home C. some D. none

4.A. cow B. now C. how D. show

PART C: Language focus (2.5 ms)

Choose the best answer (A, B, C or D) to complete these sentences.1. ……………… the film is late, I will wait to watch it.

A. Because B. When C. But D. Although

2. My father first ………………Da Lat in 2010.

A. go B. went C. visited D. been

3. If the weather ……………… good, I ………………. camping with classmates.

A. is / go B. is / will go C. will be / go D. will be / will go

4. Walking to school will help ……………… air pollution.

A. reduce B. reuse C. recycle D. refill

5. My sister ……………… home for school yet.

A. didn’t leave B. don’t leave C. hasn’t left D. isn’t leaving

6. Amsterdam is one of the ……………… cities in the world.

A. peacefulest B. peacefuler C. most peaceful D. more peaceful

7. Which one is different?

A. running B. during C. swimming D. fishing

8.____________, you will succeed.

A. Work hard B. Working hard C. If you work hard D. If only you work hard

9.The people here are ______________

A. friendly B. delicious C. polluted D. long

10.He likes playing table tennis, ______________he can’t play it.

A. and B. but C. so D. or

PART D: Reading (2.0ms)I. Choose the correct answer A, B, C or D to complete the passage. (1.0m)

People need to breathe. If they don’t breathe, they (1) ____________ die. But how clean is the air

people breathe? If they breathe (2) ___________ air, they will have breathing problems. Plants and

animals need (3) __________ air too. A lot of the things in our lives create harmful gases and (4)

Page 31: Tài liệu, học tập, trắc nghiệm, tiếng anh, văn bản, biểu ...s1.vndoc.com/.../06/40-de-thi-hoc-ky...6-co-dap-an.pdf · "/ 0. 1 2 %$ 3 ( 0 4 5 6 2 3 ( 5789 578: 2

VnDoc - Tải tài liệu, văn bản pháp luật, biểu mẫu miễn phí

_________the air dirty, like cars, motorbikes, and factories. Dirty air is called polluted air. Air

pollution can also make our Earth warmer.

1. A. won’t B. are C. must D. will

2. A. clean B. fresh C. dirty D. cleaner

3. A. clean B. safe C. dirty D. polluted

4. A. make B. get C. give D. made

II. Read the following passage and answer the questions. (1.0m)

There are two main kinds of sports: team sports and individual sports. Team sports are such sports as

baseball, basketball and volleyball. Team sports require two separate teams. The teams play against

each other. They compete against each other because they want to get the best score. For example, in a

football game, if team A scores 4 goals and team B scores 3 goals, team A wins the game. Team sports

are sometimes called competitive sports.

1. Are there three main kinds of sports?

-> ……………………………………………………………………………………..

2. How many teams do team sports require?

-> ……………………………………………………………………………………..

3. Why do two separate teams compete against each other?

-> ……………………………………………………………………………………..

4. Are team sports sometimes called competitive sports?

-> ……………………………………………………………………………………..

PART E: Writing (2.5ms)I. Rewrite these sentences as directed in the brackets. (1.0m)

1. I had a bad cold, so I didn’t go to school yesterday. (Rewrite this sentence with “because”)-> ……………………………………………………………………………………..

2. Lan can sing very well. She can’t play the piano. (Combine these sentences with “although”)-> ……………………………………………………………………………………..

3. We make the air dirty because we use the car all the time. (Rewrite this sentence with “If”)-> ……………………………………………………………………………………..

4. most/ London/ tea/ is/ popular/ the/ drink/ in. (Rearrange the words to make a sentence)

Page 32: Tài liệu, học tập, trắc nghiệm, tiếng anh, văn bản, biểu ...s1.vndoc.com/.../06/40-de-thi-hoc-ky...6-co-dap-an.pdf · "/ 0. 1 2 %$ 3 ( 0 4 5 6 2 3 ( 5789 578: 2

VnDoc - Tải tài liệu, văn bản pháp luật, biểu mẫu miễn phí

-> ……………………………………………………………………………………..

II. Write 50 -70 words about your dream house in the future. (1.5m)

1. Where will your house be? 3. What will there be in the house?

2. What will it be surrounded by? 4. What will help you do the housework

In the future, my dream house .....................................................................................................................

.......................................................................................................................................................................

.......................................................................................................................................................................

.......................................................................................................................................................................

.......................................................................................................................................................................

.......................................................................................................................................................................

.......................................................................................................................................................................

.......................................................................................................................................................................

.......................................................................................................................................................................

.......................................................................................................................................................................

.......................................................................................................................................................................

.......................................................................................................................................................................

.......................................................................................................................................................................

.......................................................................................................................................................................

.......................................................................................................................................................................

Page 33: Tài liệu, học tập, trắc nghiệm, tiếng anh, văn bản, biểu ...s1.vndoc.com/.../06/40-de-thi-hoc-ky...6-co-dap-an.pdf · "/ 0. 1 2 %$ 3 ( 0 4 5 6 2 3 ( 5789 578: 2

VnDoc - Tải tài liệu, văn bản pháp luật, biểu mẫu miễn phí

PHÒNG GD&ĐT ỨNG HÒATRƯỜNG THCS NGUYỄN THƯỢNG HIỀN

----------------Họ và tên: ....................................................

Lớp: 6 ......

ĐỀ KIỂM TRA HỌC KỲ 2NĂM HỌC 2015 - 2016

MÔN: TIẾNG ANH - LỚP 6 THÍ ĐIỂMThời gian làm bài: 45 phút

ĐỀ SỐ 2

Mark Remark

PART A: Listening

I. Mi and Nam are talking about what they will do if they become the president of the 3Rs club.Listen and fill one word in each gap to complete the table for their ideas. (1.0m)

Mi’s ideaPut a recycling bin in every (1) .………………………..………

(2) ………………………..…… energy - saving lights.

Nam’s idea(3)…………………….………..students to go to school by bus.

Set up a gardening (4) …………………………….

II. Listen again and write True (T) or False (F). (1.0m)

Statements T - F

1. Mi is from class 6E and Nam is from class 6A.

2. Nam thinks, he will organize some uniform fairs.

3. At uniform fairs, students can swap their new uniforms.

4. Students can grow flowers in the school garden.

1. ………….

2. ………….

3. ………….

4. ………….

PART B: Phonetic

Choose the word that has the underlined part pronounced differently from the others1. A. thank B.that C. this D. those

2. A. arm B. charm C. farm D. warm

Page 34: Tài liệu, học tập, trắc nghiệm, tiếng anh, văn bản, biểu ...s1.vndoc.com/.../06/40-de-thi-hoc-ky...6-co-dap-an.pdf · "/ 0. 1 2 %$ 3 ( 0 4 5 6 2 3 ( 5789 578: 2

VnDoc - Tải tài liệu, văn bản pháp luật, biểu mẫu miễn phí

3. A. come B. home C. some D. none

4. A. cow B. now C. how D. show

PART C: Language focus (2.5 ms)

Choose the best answer (A, B, C or D) to complete these sentences.1. Many students …………….. aerobics to keep fit.

A. have B. go C. play D. do

2. We will have a …………….. to take care of our health.

A. home robot B. doctor robot C. teaching robot D. worker robot

3. In the future, I will live ………………….because I like swimming.

A. in the mountain B. on the Moon C. on the ocean D. under the ground

4. My future house will have a hi-tech robot to ……………………..the floors.

A. clean B. cleaning C. cleaned D. to clean

5.We might have a ……………………. TV so we can watch TV programs from space.

A. wire B. wireless C. wiring D. wires

6. Hoa has never ……………………… to London .

A. been B. is C. was D. be

7. VTV1 is a ………………………. channel.

A. local B. national C. international D. B&C

8. In the future, we won’t go on holiday to the beach but we ________ go on holiday to the moon.

A. must B.can’t C.might D. won’t

9. If we throw trash on the road, we will_________our environment.

A. pollution B. pollute C. polluting D. to pollute

10. They couldn’t go on a picnic ______________the weather was bad.

A. because B. when C. but D. or

PART D: Reading (2.0ms)I. Read the text and answer the questions: (1ms)

A robot cannot think or do things alone. People use a computer to control it. Today, robots are

not very popular because they are too expensive, but they are very useful. They can help us save a lot

of time and work. A robot can do the same work for 24 hours, and yet, it does not complain or get tired.

Page 35: Tài liệu, học tập, trắc nghiệm, tiếng anh, văn bản, biểu ...s1.vndoc.com/.../06/40-de-thi-hoc-ky...6-co-dap-an.pdf · "/ 0. 1 2 %$ 3 ( 0 4 5 6 2 3 ( 5789 578: 2

VnDoc - Tải tài liệu, văn bản pháp luật, biểu mẫu miễn phí

In the US, people use robots to guard some important places. These robots can listen to certain noises

and send signals for help if there’s trouble or danger. In Japan, people use robots in factories to build

cars. In the future, scientists will design many types of intelligent robots. Their robots will be able to do

many more complicated things. However, some people do not like robots. They fear (sợ) that one day

robots will be too powerful.

1. Questions:

1. Why do people use a computer to control a robot?

…………………………………………………………………………..

2. Why aren’t robots popular today?

……………………………………………………………………………………………

3. What do people in the US use robots for?

……………………………………………………………………………………

4. What do robots in factories in Japan do?

…………………………………………………………………………………….

II. Complete the passage with the words provided: (1ms)Against – team – score - competitive

There are two main kinds of sports: (1) ……………..... sports and individual sports. Team sports

are such sports as baseball, basketball and volleyball. Team sports require two separate teams. The

teams play (2).......……………… each other. They compete against each other in order to get the best

(3).....………………. ., for example, in a football game, if team A gets 4 points and team B gets 2

points, team A wins the game. Team sports are sometimes called (4)………………….. sports.

PART E: Writing (2.5ms)I. Rewrite these sentences as directed in the brackets. (1.0m)

1. He was lazy, so he got a bad mark. (Rewrite this sentence with “because”)-> ……………………………………………………………………………………..

2. It rained hard yesterday. I went to school on time. (Combine these sentences with “although”)-> ……………………………………………………………………………………..

3. We make the air dirty because we use the car all the time. (Rewrite this sentence with “If”)-> ……………………………………………………………………………………..

4. the/ Sydney/ city/ is/ cleanest/ world/ the/ in. (Rearrange the words to make a sentence)

Page 36: Tài liệu, học tập, trắc nghiệm, tiếng anh, văn bản, biểu ...s1.vndoc.com/.../06/40-de-thi-hoc-ky...6-co-dap-an.pdf · "/ 0. 1 2 %$ 3 ( 0 4 5 6 2 3 ( 5789 578: 2

VnDoc - Tải tài liệu, văn bản pháp luật, biểu mẫu miễn phí

-> ……………………………………………………………………………………..

II. Write 50 -70 words about your dream house in the future. (1.5m)

1. Where will your house be? 3. What will there be in the house?

2. What will it be surrounded by? 4. What will help you do the housework?

In the future, my dream house .....................................................................................................................

.......................................................................................................................................................................

.......................................................................................................................................................................

.......................................................................................................................................................................

.......................................................................................................................................................................

.......................................................................................................................................................................

.......................................................................................................................................................................

.......................................................................................................................................................................

.......................................................................................................................................................................

.......................................................................................................................................................................

.......................................................................................................................................................................

.......................................................................................................................................................................

.......................................................................................................................................................................

.......................................................................................................................................................................

Page 37: Tài liệu, học tập, trắc nghiệm, tiếng anh, văn bản, biểu ...s1.vndoc.com/.../06/40-de-thi-hoc-ky...6-co-dap-an.pdf · "/ 0. 1 2 %$ 3 ( 0 4 5 6 2 3 ( 5789 578: 2

VnDoc - Tải tài liệu, văn bản pháp luật, biểu mẫu miễn phí

PHÒNG GD&ĐT ỨNG HÒATRƯỜNG THCS NGUYỄN THƯỢNG HIỀN

----------------Họ và tên: ....................................................

Lớp: 6 ......

ĐỀ KIỂM TRA HỌC KỲ 2NĂM HỌC 2015 - 2016

MÔN: TIẾNG ANH - LỚP 6 THÍ ĐIỂMThời gian làm bài: 45 phút

ĐỀ SỐ 3

Mark Remark

PART A: Listening (2,0ms)

I. Mi and Nam are talking about what they will do if they become the president of the 3Rs club.Listen and fill one word in each gap to complete the table for their ideas. (1.0m)

Mi’s ideasPut a recycling bin in every (1) .………………………..………

(2) ………………………..…… energy - saving lights.

Nam’s ideas(3)…………………….………..students to go to school by bus.

Set up a gardening (4) …………………………….

II. Listen again and write True (T) or False (F). (1.0m)

Statements T - F

1. Mi is from class 6A and Nam is from class 6E.

2. Mi thinks, she will organize a few book fairs.

3. At book fairs, students can swap their new books.

4. Students can grow vegetables in the school garden.

1. ………….

2. ………….

3. ………….

4. ………….

PART B: Phonetic

Choose the word that has the underlined part pronounced differently from the others (1M)1.A. drive B. drip C. drill D. drink

2A. travel B. trap C. track D. trace

Page 38: Tài liệu, học tập, trắc nghiệm, tiếng anh, văn bản, biểu ...s1.vndoc.com/.../06/40-de-thi-hoc-ky...6-co-dap-an.pdf · "/ 0. 1 2 %$ 3 ( 0 4 5 6 2 3 ( 5789 578: 2

VnDoc - Tải tài liệu, văn bản pháp luật, biểu mẫu miễn phí

3.A.drunk B. drug C. truth D. trust

4.A.call B. fall C. math D. wall

PART C: Language focus (2.5 ms)

Choose the best answer (A, B, C or D) to complete these sentences.1. Last weekend Phong and Lan ___________their grandparents.

A. visited B. was C. did D. are

2. All of us ___________at home yesterday evening.

A. were B. was C. did D. are

3. If we throw trash on the road, we will_________our environment.

A. pollution B. pollute C. polluting D. to pollute

4. In the future, we won’t go on holiday to the beach but we ________ go on holiday to the moon.

A. must B. can’t C. might D. won’t

5. ____________, you will succeed.

A. Work hard B. Working hard C. If you work hard D. If only you work

hard

6. Walking to school will help ……………… air pollution.

A. reduce B. reuse C. recycle D. refill

7. My sister ……………… home for school yet.

A. didn’t leave B. don’t leave C. hasn’t left D. isn’t leaving

8. He likes playing table tennis, ______________he can’t play it.

A. and B. but C. so D. or

9. ______________walk in the rain. It’s not good for your health.

A. Won’t B. Can’t C. Don’t D. Mustn’t

10. We will have a ………………to look after the children.

A. dishwasher B. television C. robot D. computer

PART D: Reading (2.0ms)I Read the text and fill in each blank with one correct word (1m)

countryside career voted playerPele is widely regarded as the best football (1) ………………………… of all time. He was

born on October 21st, 1940 in the (2) ………………………… of Brazil. Pele’s father was a

Page 39: Tài liệu, học tập, trắc nghiệm, tiếng anh, văn bản, biểu ...s1.vndoc.com/.../06/40-de-thi-hoc-ky...6-co-dap-an.pdf · "/ 0. 1 2 %$ 3 ( 0 4 5 6 2 3 ( 5789 578: 2

VnDoc - Tải tài liệu, văn bản pháp luật, biểu mẫu miễn phí

professional football player and taught him how to play at a very young age. Pele began his

(3) …………………………..at the age of 15 . In 1958, at the age of 17, he won his first World Cup.

He won three World Cup and scored 1,281 goals in his 22 year career. He was

(4) …………………………Football Player of the century. During his career, he became well – known

around the world as The King of Football.

II. Read the test and answer the questions: ( 1m)In the past, robots had a minor role. They could only do very simple things

In the future, robots will play a very important role. They will be able to do many things like humans.

By the end of this century, we will have to explore the Moon to find important minerals. It is

dangerous for people so we will use robots instead of humans. It won’t be very expensive because

robots won’t need money, food, air or water. They will only use solar energy. Robots will get minerals

from the Moon and they will bring them back to the Earth.

1. Could robots do simple things in the past?

→ ……………………………………………………………………………………………………...

2. How will robots be in the future?

→ ……………………………………………………………………………………………………...

3. What will we have to do by the end of this century?

→ ……………………………………………………………………………………………………...

4. Is It expensive ? Why?

→ ……………………………………………………………………………………………………...

PART E: Writing (2.0ms)I. Rewrite or combine the sentences, using the suggested words (1p)

1.My brother can swim very well. He can’t play badminton (but)

- My brother can ..........................................................................................................................

2. I don’t like watching news. It’s boring ( because)

- I don’t .....................................................................................................................................

3. I was tired last night. I stayed up late to watch a game show (Although)

- Although ......................................................................................................................................

Page 40: Tài liệu, học tập, trắc nghiệm, tiếng anh, văn bản, biểu ...s1.vndoc.com/.../06/40-de-thi-hoc-ky...6-co-dap-an.pdf · "/ 0. 1 2 %$ 3 ( 0 4 5 6 2 3 ( 5789 578: 2

VnDoc - Tải tài liệu, văn bản pháp luật, biểu mẫu miễn phí

4. cut/ more/if / we/ down/ trees,/ more / there/ floods/ be/ will (Rearrange the words to make a

sentence)

.........................................................................................................................................................

II. Write 50 -70 words about your dream house in the future. (1.5m)

1. Where will your house be? 3. What will there be in the house?

2. What will it be surrounded by? 4. What will help you do the housework?

In the future, my dream house ....................................................................................................................

.......................................................................................................................................................................

.......................................................................................................................................................................

.......................................................................................................................................................................

.......................................................................................................................................................................

.......................................................................................................................................................................

.......................................................................................................................................................................

.......................................................................................................................................................................

.......................................................................................................................................................................

.......................................................................................................................................................................

.......................................................................................................................................................................

.......................................................................................................................................................................

.......................................................................................................................................................................

.......................................................................................................................................................................

Page 41: Tài liệu, học tập, trắc nghiệm, tiếng anh, văn bản, biểu ...s1.vndoc.com/.../06/40-de-thi-hoc-ky...6-co-dap-an.pdf · "/ 0. 1 2 %$ 3 ( 0 4 5 6 2 3 ( 5789 578: 2

VnDoc - Tải tài liệu, văn bản pháp luật, biểu mẫu miễn phí

PHÒNG GD&ĐT ỨNG HÒATRƯỜNG THCS NGUYỄN THƯỢNG HIỀN

----------------Họ và tên: ....................................................

Lớp: 6 ......

ĐỀ KIỂM TRA HỌC KỲ 2NĂM HỌC 2015 - 2016

MÔN: TIẾNG ANH - LỚP 6 THÍ ĐIỂMThời gian làm bài: 45 phút

ĐỀ SỐ 4

Mark Remark

PART A: Listening (2,0ms)

I. Mi and Nam are talking about what they will do if they become the president of the 3Rs club.Listen and fill one word in each gap to complete the table for their ideas. (1.0m)

Mi’s ideasPut a recycling bin in every (1) .………………………..………

(2) ………………………..…… energy - saving lights.

Nam’s ideas(3)…………………….………..students to go to school by bus.

Set up a gardening (4) …………………………….

II. Listen again and write True (T) or False (F). (1.0m)

Statements T - F

1. Mi is from class 6A and Nam is from class 6E.

2. Mi thinks, she will organize a few book fairs.

3. At book fairs, students can swap their new books.

4. Students can grow vegetables in the school garden.

1. ………….

2. ………….

3. ………….

4. ………….

PART B: Phonetic

Choose the word that has the underlined part pronounced differently from the others (1M)1A. thank B.that C. this D. those

2A. cost B. most C.sometimes D. post

Page 42: Tài liệu, học tập, trắc nghiệm, tiếng anh, văn bản, biểu ...s1.vndoc.com/.../06/40-de-thi-hoc-ky...6-co-dap-an.pdf · "/ 0. 1 2 %$ 3 ( 0 4 5 6 2 3 ( 5789 578: 2

VnDoc - Tải tài liệu, văn bản pháp luật, biểu mẫu miễn phí

3.A. arm B. charm C. farm D. warm

4.A. come B. home C. some D. none

PART C: Language focus (2.5 ms)

Choose the best answer (A, B, C or D) to complete these sentences.1.I think “Tom and Jerry is the best ______________ “

A. film B. sport C. newsreel D. cartoon

2.In the future, we won’t go on holiday to the beach but we ________ go on holiday to the moon.

A. must B. can’t C. might D. won’t

3.They couldn’t go on a picnic ______________the weather was bad.

A. because B. when C. but D. or

4.My house is ______________to the supermarket.

A. to the left B. behind C. near D. next

5.You have to throw the ball into the net when you play___________

A. basketball B. football C.tennis D. badminton

6.If we throw trash on the road, we will_________our environment.

A. pollution B. pollute C. polluting D. to pollute

7. We will have a …………….. to take care of our health.

A. home robot B. doctor robot C. teaching robot D. worker robot

8. In the future, I will live ………………….because I like swimming.

A. in the mountain B. on the Moon C. on the ocean D. under the ground

9. Walking to school will help ……………… air pollution.

A. reduce B. reuse C. recycle D. refill

10. My sister ……………… home for school yet.

A. didn’t leave B. don’t leave C. hasn’t left D. isn’t leaving

PART D: Reading (2.0ms)I. Read the text carefully and answer the questions: (1pt)

In the future, we live in amazing new homes. Our houses will use the sun or the wind to make

electricity. We won’t have cookers, dishwashers or washing machines. We will have robots to cook our

Page 43: Tài liệu, học tập, trắc nghiệm, tiếng anh, văn bản, biểu ...s1.vndoc.com/.../06/40-de-thi-hoc-ky...6-co-dap-an.pdf · "/ 0. 1 2 %$ 3 ( 0 4 5 6 2 3 ( 5789 578: 2

VnDoc - Tải tài liệu, văn bản pháp luật, biểu mẫu miễn phí

dinner and wash our clothes. We will have robots to look after our children, and to feed our cats and

dogs.

We won’t use computers. We will have special remote control units. We can surf the Internet, send and

receive e-mails and order food from the markets without getting out of bed.

We won’t have TVs or CD players because we will have watches that play music, take pictures and tell

time. Our cars might not use gas. They might use energy from air, water or plants.

1/ Where will we live in the future?

…………………………………………………………………………………………………….

2/ Will we need dishwashers or washing machines?

…………………………………………………………………………………………………….

3/ Who will help us do the housework?

………………………………………………………………………………………………….....

4/ What kind of energy might our cars use?

…………………………………………………………………………………………………….

II. Fill in each blank with a right word in the box (1pt) :

factory – new - rubbish – bottles – is

Recycling is using (1) ................................. to remake new goods that can be sold again. For

example, when you put (2) .................................... and cans in the recycling bin, people take them to a

(3) ............................ . In this factory, people reprocess these bottles and cans into many

(4) ............................ things.

PART E: Writing (2.5ms)

I. Rewrite these sentences as directed in the brackets. (1.0m)1. We make the air dirty because we use the car all the time. (Rewrite this sentence with “If”)

-> ……………………………………………………………………………………..

2. The/ Sydney/ city/ is/ cleanest/ world/ the/ in. (Rearrange the words to make a sentence)

-> ……………………………………………………………………………………..

3. He was lazy, so he got a bad mark. (Rewrite this sentence with “because”)-> ……………………………………………………………………………………..

4. It rained hard yesterday. I went to school on time. (Combine these sentences with “although”)

Page 44: Tài liệu, học tập, trắc nghiệm, tiếng anh, văn bản, biểu ...s1.vndoc.com/.../06/40-de-thi-hoc-ky...6-co-dap-an.pdf · "/ 0. 1 2 %$ 3 ( 0 4 5 6 2 3 ( 5789 578: 2

VnDoc - Tải tài liệu, văn bản pháp luật, biểu mẫu miễn phí

-> ……………………………………………………………………………………..

II. Write 50 -70 words about your dream house in the future. (1.5m)

1. Where will your house be? 3. What will there be in the house?

2. What will it be surrounded by? 4. What will help you do the housework?

In the future, my dream house ....................................................................................................................

.......................................................................................................................................................................

.......................................................................................................................................................................

.......................................................................................................................................................................

.......................................................................................................................................................................

.......................................................................................................................................................................

.......................................................................................................................................................................

.......................................................................................................................................................................

.......................................................................................................................................................................

.......................................................................................................................................................................

.......................................................................................................................................................................

.......................................................................................................................................................................

.......................................................................................................................................................................

.......................................................................................................................................................................

Page 45: Tài liệu, học tập, trắc nghiệm, tiếng anh, văn bản, biểu ...s1.vndoc.com/.../06/40-de-thi-hoc-ky...6-co-dap-an.pdf · "/ 0. 1 2 %$ 3 ( 0 4 5 6 2 3 ( 5789 578: 2

VnDoc - Tải tài liệu, văn bản pháp luật, biểu mẫu miễn phí

ANSWER KEYSMÃ ĐỀ 2Part A: Listening)

I. Mi and Nam are talking about what they will do if they become the president of the 3Rs club.Listen and fill one word in each gap to complete the table for their ideas. (1.0m)

(1) classroom

(2) Buy

(3) Encourage

(4) group

II. Listen again and write True (T) or False (F). (1.0m)1. T

2. T

3. F

4. F

PART B: Phonetic

Choose the word that has the underlined part pronounced differently from the others.5. A 6. D 7. B 8. DPART C: Language focus (2.5 ms)

Choose the best answer (A, B, C or D) to complete these sentences.1. D. do

2. B. doctor robot

3. C. on the ocean

4. A. clean

5.B. wireless

6. A. been

7. B. national

8. C.might

9. B. pollute

10. A. because

Part D: Reading (2.0ms)I. Read the text and answer the questions: (1ms)

Page 46: Tài liệu, học tập, trắc nghiệm, tiếng anh, văn bản, biểu ...s1.vndoc.com/.../06/40-de-thi-hoc-ky...6-co-dap-an.pdf · "/ 0. 1 2 %$ 3 ( 0 4 5 6 2 3 ( 5789 578: 2

VnDoc - Tải tài liệu, văn bản pháp luật, biểu mẫu miễn phí

1. Questions:1. Why do people use a computer to control a robot?

…………………………………………………………………………..

2. Why aren’t robots popular today?

……………………………………………………………………………………………

3. What do people in the US use robots for?

……………………………………………………………………………………

4. What do robots in factories in Japan do?

…………………………………………………………………………………….

II. Complete the passage with the words provided: (1ms)

(1) team

(2) Against

(3) score

(4) competitive

Part E: Writing (2.5ms)I. Rewrite these sentences as directed in the brackets. (1.0m)

1. He got a bad mark because he was lazy

2. Although it rained hard yesterday,I went to school on time.

3. If we use the car all the time We “ll make the air dirty.

4. Sydney is the cleanest city in the world.

II. Write 50 -70 words about your dream house in the future. (1.5m)

Page 47: Tài liệu, học tập, trắc nghiệm, tiếng anh, văn bản, biểu ...s1.vndoc.com/.../06/40-de-thi-hoc-ky...6-co-dap-an.pdf · "/ 0. 1 2 %$ 3 ( 0 4 5 6 2 3 ( 5789 578: 2

VnDoc - Tải tài liệu, văn bản pháp luật, biểu mẫu miễn phí

UBND HUYỆN HOÀI NHƠNPHÒNG GD&ĐT HOÀI NHƠN

--------------------

ĐỀ THI HỌC KỲ IINĂM HỌC 2014 -2015

Môn: Tiếng Anh 6 – Ngày thi: ....05/2015

Thời gian: 45 phút (không kể thời gian phát đề)

Họ và tên: ...................................................Lớp: 6 ........SBD:…………………

Giám thị 1 Giám thị 2 Mã phách

--------------------------------------------------------------------------------------- ------------------------------------

Điểm Bằng chữ Giám thị 1 Giám thị 2 Mã phách

ĐỀ 1

I. Choose the best answer A, B, C or D: (3ms)1. Miss Chi has _________.

A. black long hair B. long black hair C. hair long black D. A&B

2. What color are his eyes? – They are ________.

A. tall B. short C. long D. blue

3. I am hot and thirsty. I’d like to drink some_________.

A. meat B. apples C. water D. fish

4. The Great Wall of China is the world’s ………………………structure.

A. long B. longer C. longest D. the longest

5. Laura is from Canada. She speaks English and ________.

A. Vietnamese B. France C. Japanese D. French

6. What is there _____, Mum? – Fish, chicken, rice and vegetables.

A. eats B. to eat C. eating D. eat

7. I want to buy _______cooking oil and a kilo of beef.

A. a bottle of B. a bar of C. a tube of D. a glass of

8. Mike _______ his bike to school at the moment.

Page 48: Tài liệu, học tập, trắc nghiệm, tiếng anh, văn bản, biểu ...s1.vndoc.com/.../06/40-de-thi-hoc-ky...6-co-dap-an.pdf · "/ 0. 1 2 %$ 3 ( 0 4 5 6 2 3 ( 5789 578: 2

VnDoc - Tải tài liệu, văn bản pháp luật, biểu mẫu miễn phí

A. are riding B. riding C. rides D. is riding

9. We never go camping because we don’t have a _________.

A. tent B. camera C. fishing rod D. toothpaste

10. _____does Tom play? – Badminton.

A. When sports B. Which sports C. How sports D. Why sports

11 Why are wild animals in _________. - Because we are cutting and burning the forests.

A. dangerous B. dangerously C. danger D. endanger

12. _______ does Dung go to the swimming pool? – Twice a week.

A. How often B. How far C. How long D. How old

II. Find a mistake A, B, C or D: (1m)1. Hoa is going visit her grandparents tomorrow morning.

A B C D

2. Thuy always is late for school in the morning.

A B C D

3. Kim Tuyen wants to buy a tube of soap.

A B C D

4. How many milk does Mr. Hai’s cows produce? – A little.

A B C D

III. Read the passage then write (T) for true sentence and write (F) for false sentence: (2ms)

BUDAPESTBudapest has a population of over two million people. One in every five Hungarian lives there. The River

Danube flows inside the city. The town on the west bank is older and beautiful. The town on the east bank is

bigger and modern. There are six bridges across the river.

The public transport system in Budapest is one of the cheapest in the world. You can travel easily by bus,

tram, or taxi. But driving a car in Budapest is not a good idea! There are not many car parks. The cars are old,

and the pollution is bad.

New words:

Budapest: Thủ đô của nước Hungary

modern: hiện đại

east: phía đông

west: phía tây

Page 49: Tài liệu, học tập, trắc nghiệm, tiếng anh, văn bản, biểu ...s1.vndoc.com/.../06/40-de-thi-hoc-ky...6-co-dap-an.pdf · "/ 0. 1 2 %$ 3 ( 0 4 5 6 2 3 ( 5789 578: 2

VnDoc - Tải tài liệu, văn bản pháp luật, biểu mẫu miễn phí

public transport system: hệ thống giao thông công cộng

trạm: tàu điện

bank: bờ sông

T F

1. Budapest has a population of over two thousand people.

2. The River Danube flows inside the city.

3. The town on the west bank is older than the one in the east bank.

4. The town on the east bank is modern.

5. There are five bridges across the river.

6. The public transport system in Budapest is cheap.

7. It’s not good to drive a car in Budapest.

8. The pollution is good in Budapest.

IV. Read the passage and answer the questions below: (2ms)

Miss Anh is our teacher. She teaches us Literature. She is tall and think but she is not weak. She plays a lot

of sports. Her favorite sports is badminton. She plays it three times a week. She likes jogging, too. She jogs in the

park near her house every morning. In her free time she often listens to music and watches TV.

1. What does Miss Anh?

………………………………………………………………………………………………………………….

2. Which sports does she play?

………………………………………………………………………………………………………………….

3. How often does she play badminton?

………………………………………………………………………………………………………………….

4. What does she often do in her free time?

………………………………………………………………………………………………………………….

V. Rewrite the sentence using the given words: (2ms)

1. Vietnam has lots of beautiful beaches.

There are ……………………………………………………………………………………………………

2. No river in the world is longer than The Nile river.

The Nile ………………………………………………………………………………………………………….

3. Ho Chi Minh city is bigger than Da Nang city.

Da Nang city……………………………………………………………………………………………………….

Page 50: Tài liệu, học tập, trắc nghiệm, tiếng anh, văn bản, biểu ...s1.vndoc.com/.../06/40-de-thi-hoc-ky...6-co-dap-an.pdf · "/ 0. 1 2 %$ 3 ( 0 4 5 6 2 3 ( 5789 578: 2

VnDoc - Tải tài liệu, văn bản pháp luật, biểu mẫu miễn phí

4. My mother usually drives to work.

My mother usually goes …………………………………………………………………………………………

The end- Good luck!

Page 51: Tài liệu, học tập, trắc nghiệm, tiếng anh, văn bản, biểu ...s1.vndoc.com/.../06/40-de-thi-hoc-ky...6-co-dap-an.pdf · "/ 0. 1 2 %$ 3 ( 0 4 5 6 2 3 ( 5789 578: 2

VnDoc - Tải tài liệu, văn bản pháp luật, biểu mẫu miễn phí

UBND HUYỆN HOÀI NHƠNPHÒNG GD&ĐT HOÀI NHƠN

--------------------

ĐỀ THI HỌC KỲ IINĂM HỌC 2014 -2015

Môn: Tiếng Anh 6 – Ngày thi: ....05/2015

Thời gian: 45 phút (không kể thời gian phát đề)

Họ và tên: ...................................................Lớp: 6A........SBD:…………………

Giám thị 1 Giám thị 2 Mã phách

--------------------------------------------------------------------------------------- ------------------------------------

Điểm Bằng chữ Giám thị 1 Giám thị 2 Mã phách

ĐỀ 2I. Choose the best answer A, B, C or D: (3ms)

1. Laura is from Canada. She speaks English and ________.

A. Vietnamese B. France C. Japanese D. French

2. What is there _____, Mum? – Fish, chicken, rice and vegetables.

A. eats B. to eat C. eating D. eat

3. I want to buy _______cooking oil and a kilo of beef.

A. a bottle of B. a bar of C. a tube of D. a glass of

4. Mike _______ his bike to school at the moment.

A. are riding B. riding C. rides D. is riding

5 Why are wild animals in _________. - Because we are cutting and burning the forests.

A. dangerous B. dangerously C. danger D. endanger

6. _______ does Dung go to the swimming pool? – Twice a week.

A. How often B. How far C. How long D. How old

7. Miss Chi has _________.

A. black long hair B. long black hair C. hair long black D. A&B

8. What color are his eyes? – They are ________.

Page 52: Tài liệu, học tập, trắc nghiệm, tiếng anh, văn bản, biểu ...s1.vndoc.com/.../06/40-de-thi-hoc-ky...6-co-dap-an.pdf · "/ 0. 1 2 %$ 3 ( 0 4 5 6 2 3 ( 5789 578: 2

VnDoc - Tải tài liệu, văn bản pháp luật, biểu mẫu miễn phí

A. tall B. short C. long D. blue

9. I am hot and thirsty. I’d like to drink some_________.

A. meat B. apples C. water D. fish

10. The Great Wall of China is the world’s ………………………structure.

A. long B. longer C. longest D. the longest

11. We never go camping because we don’t have a _________.

A. tent B. camera C. fishing rod D. toothpaste

12. _____does Tom play? – Badminton.

A. When sports B. Which sports C. How sports D. Why sports

II. Find a mistake A, B, C or D: (1m)1. Thanh wants to buy a tube of soap.

A B C D

2. How many milk does Mr. Hoang’s cows produce? – A little.

A B C D

3. Hanh is going visit her grandparents tomorrow evening.

A B C D

4. Man always is late for school in the afternoon.

A B C D

III. Read the passage then choose (true)T or (false) F: (2ms)

BUDAPESTBudapest has a population of over two million people. One in every five Hungarian lives there. The River

Danube flows inside the city. The town on the west bank is older and beautiful. The town on the east bank is

bigger and modern. There are six bridges across the river.

The public transport system in Budapest is one of the cheapest in the world. You can travel easily by bus,

tram, or taxi. But driving a car in Budapest is not a good idea! There are not many car parks. The cars are old,

and the pollution is bad.

New words:

Budapest: Thủ đô của nước Hungary

modern: hiện đại

east: phía đông

west: phía tây

Page 53: Tài liệu, học tập, trắc nghiệm, tiếng anh, văn bản, biểu ...s1.vndoc.com/.../06/40-de-thi-hoc-ky...6-co-dap-an.pdf · "/ 0. 1 2 %$ 3 ( 0 4 5 6 2 3 ( 5789 578: 2

VnDoc - Tải tài liệu, văn bản pháp luật, biểu mẫu miễn phí

public transport system: hệ thống giao thông công cộng

trạm: tàu điện

bank: bờ sông

T F

1. Budapest has a population of over two thousand people.

2. The River Danube flows inside the city.

3. The town on the west bank is older than the one in the east bank.

4. The town on the east bank is modern.

5. There are five bridges across the river.

6. The public transport system in Budapest is cheap.

7. It’s not good to drive a car in Budapest.

8. The pollution is good in Budapest.

IV. Read the passage and answer the questions below: (2ms)

Miss Anh is our teacher. She teaches us Literature. She is tall and think but she is not weak. She plays a lot

of sports. Her favorite sports is badminton. She plays it three times a week. She likes jogging, too. She jogs in the

park near her house every morning. In her free time she often listens to music and watches TV.

1. What does Miss Anh?

………………………………………………………………………………………………………………….

2. Which sports does she play?

………………………………………………………………………………………………………………….

3. How often does she play badminton?

………………………………………………………………………………………………………………….

4. What does she often do in her free time?

………………………………………………………………………………………………………………….

V. Rewrite the sentence using the given words: (2ms)

1. Ho Chi Minh city is bigger than Da Nang city.

Da Nang city……………………………………………………………………………………………………….

2. My mother usually drives to work.

My mother usually goes …………………………………………………………………………………………

3. Vietnam has lots of beautiful beaches.

There are ……………………………………………………………………………………………………

Page 54: Tài liệu, học tập, trắc nghiệm, tiếng anh, văn bản, biểu ...s1.vndoc.com/.../06/40-de-thi-hoc-ky...6-co-dap-an.pdf · "/ 0. 1 2 %$ 3 ( 0 4 5 6 2 3 ( 5789 578: 2

VnDoc - Tải tài liệu, văn bản pháp luật, biểu mẫu miễn phí

4. No river in the world is longer than The Nile river.

The Nile …………………………………………………………………………………………………

The end- Good luck!

Page 55: Tài liệu, học tập, trắc nghiệm, tiếng anh, văn bản, biểu ...s1.vndoc.com/.../06/40-de-thi-hoc-ky...6-co-dap-an.pdf · "/ 0. 1 2 %$ 3 ( 0 4 5 6 2 3 ( 5789 578: 2

VnDoc - Tải tài liệu, văn bản pháp luật, biểu mẫu miễn phí

UBND HUYỆN HOÀI NHƠNPHÒNG GD&ĐT HOÀI NHƠN

--------------------

ĐỀ THI HỌC KỲ IINĂM HỌC 2014 -2015

Môn: Tiếng Anh 6 – Ngày thi: ....05/2015

Thời gian: 45 phút (không kể thời gian phát đề)

Họ và tên: ...................................................Lớp: 6A........SBD:…………………

Giám thị 1 Giám thị 2 Mã phách

--------------------------------------------------------------------------------------- ------------------------------------

Điểm Bằng chữ Giám thị 1 Giám thị 2 Mã phách

ĐỀ 3I. Choose the best answer A, B, C or D: (3ms)

1. The Great Wall of China is the world’s ………………………structure.

A. long B. longer C. longest D. the longest

2. Laura is from Canada. She speaks English and ________.

A. Vietnamese B. France C. Japanese D. French

3. Miss Chi has _________.

A. black long hair B. long black hair C. hair long black D. A&B

4. What color are his eyes? – They are ________.

A. tall B. short C. long D. blue

5. I am hot and thirsty. I’d like to drink some_________.

A. meat B. apples C. water D. fish

6. What is there _____, Mum? – Fish, chicken, rice and vegetables.

A. eats B. to eat C. eating D. eat

7. _____does Tom play? – Badminton.

A. When sports B. Which sports C. How sports D. Why sports

8 Why are wild animals in _________. - Because we are cutting and burning the forests.

Page 56: Tài liệu, học tập, trắc nghiệm, tiếng anh, văn bản, biểu ...s1.vndoc.com/.../06/40-de-thi-hoc-ky...6-co-dap-an.pdf · "/ 0. 1 2 %$ 3 ( 0 4 5 6 2 3 ( 5789 578: 2

VnDoc - Tải tài liệu, văn bản pháp luật, biểu mẫu miễn phí

A. dangerous B. dangerously C. danger D. endanger

9. _______ does Dung go to the swimming pool? – Twice a week.

A. How often B. How far C. How long D. How old

10. I want to buy _______cooking oil and a kilo of beef.

A. a bottle of B. a bar of C. a tube of D. a glass of

11. Mike _______ his bike to school at the moment.

A. are riding B. riding C. rides D. is riding

12. We never go camping because we don’t have a _________.

A. tent B. camera C. fishing rod D. toothpaste

II. Find a mistake A, B, C or D: (1m)1. Tuan always is late for school in the morning.

A B C D

2. Nga is going visit her grandparents tomorrow morning.

A B C D

3. Phong wants to buy a tube of soap.

A B C D

4. How many milk does Mrs. Mai’s cows produce? – A little.

A B C D

III. Read the passage then choose (true)T or (false) F: (2ms)

BUDAPESTBudapest has a population of over two million people. One in every five Hungarian lives there. The River

Danube flows inside the city. The town on the west bank is older and beautiful. The town on the east bank is

bigger and modern. There are six bridges across the river.

The public transport system in Budapest is one of the cheapest in the world. You can travel easily by bus,

tram, or taxi. But driving a car in Budapest is not a good idea! There are not many car parks. The cars are old,

and the pollution is bad.

New words:

Budapest: Thủ đô của nước Hungary

modern: hiện đại

east: phía đông

west: phía tây

Page 57: Tài liệu, học tập, trắc nghiệm, tiếng anh, văn bản, biểu ...s1.vndoc.com/.../06/40-de-thi-hoc-ky...6-co-dap-an.pdf · "/ 0. 1 2 %$ 3 ( 0 4 5 6 2 3 ( 5789 578: 2

VnDoc - Tải tài liệu, văn bản pháp luật, biểu mẫu miễn phí

public transport system: hệ thống giao thông công cộng

trạm: tàu điện

bank: bờ sông

T F

1. Budapest has a population of over two thousand people.

2. The River Danube flows inside the city.

3. The town on the west bank is older than the one in the east bank.

4. The town on the east bank is modern.

5. There are five bridges across the river.

6. The public transport system in Budapest is cheap.

7. It’s not good to drive a car in Budapest.

8. The pollution is good in Budapest.

IV. Read the passage and answer the questions below: (2ms)

Miss Anh is our teacher. She teaches us Literature. She is tall and think but she is not weak. She plays a lot

of sports. Her favorite sports is badminton. She plays it three times a week. She likes jogging, too. She jogs in the

park near her house every morning. In her free time she often listens to music and watches TV.

1. What does Miss Anh?

………………………………………………………………………………………………………………….

2. Which sports does she play?

………………………………………………………………………………………………………………….

3. How often does she play badminton?

………………………………………………………………………………………………………………….

4. What does she often do in her free time?

………………………………………………………………………………………………………………….

V. Rewrite the sentence using the given words: (2ms)

4. My mother usually drives to work.

My mother usually goes ………………………………………………………………………………………

2. Ho Chi Minh city is bigger than Da Nang city.

Da Nang city……………………………………………………………………………………………………….

3. Vietnam has lots of beautiful beaches.

There are ……………………………………………………………………………………………………

Page 58: Tài liệu, học tập, trắc nghiệm, tiếng anh, văn bản, biểu ...s1.vndoc.com/.../06/40-de-thi-hoc-ky...6-co-dap-an.pdf · "/ 0. 1 2 %$ 3 ( 0 4 5 6 2 3 ( 5789 578: 2

VnDoc - Tải tài liệu, văn bản pháp luật, biểu mẫu miễn phí

2. No river in the world is longer than The Nile river.

The Nile ………………………………………………………………………………………………………….

The end- Good luck!

Page 59: Tài liệu, học tập, trắc nghiệm, tiếng anh, văn bản, biểu ...s1.vndoc.com/.../06/40-de-thi-hoc-ky...6-co-dap-an.pdf · "/ 0. 1 2 %$ 3 ( 0 4 5 6 2 3 ( 5789 578: 2

VnDoc - Tải tài liệu, văn bản pháp luật, biểu mẫu miễn phí

UBND HUYỆN HOÀI NHƠNPHÒNG GD&ĐT HOÀI NHƠN

--------------------

ĐỀ THI HỌC KỲ IINĂM HỌC 2014 -2015

Môn: Tiếng Anh 6 – Ngày thi: ....05/2015

Thời gian: 45 phút (không kể thời gian phát đề)

Họ và tên: ...................................................Lớp: 6A........SBD:…………………

Giám thị 1 Giám thị 2 Mã phách

--------------------------------------------------------------------------------------- ------------------------------------

Điểm Bằng chữ Giám thị 1 Giám thị 2 Mã phách

ĐỀ 4I. Choose the best answer A, B, C or D: (3ms)

1. What is there _____, Mum? – Fish, chicken, rice and vegetables.

A. eats B. to eat C. eating D. eat

2. I want to buy _______cooking oil and a kilo of beef.

A. a bottle of B. a bar of C. a tube of D. a glass of

3. Mike _______ his bike to school at the moment.

A. are riding B. riding C. rides D. is riding

4. _____does Tom play? – Badminton.

A. When sports B. Which sports C. How sports D. Why sports

5. _______ does Dung go to the swimming pool? – Twice a week.

A. How often B. How far C. How long D. How old

6. Miss Chi has _________.

A. black long hair B. long black hair C. hair long black D. A&B

7. What color are his eyes? – They are ________.

A. tall B. short C. long D. blue

8. I am hot and thirsty. I’d like to drink some_________.

Page 60: Tài liệu, học tập, trắc nghiệm, tiếng anh, văn bản, biểu ...s1.vndoc.com/.../06/40-de-thi-hoc-ky...6-co-dap-an.pdf · "/ 0. 1 2 %$ 3 ( 0 4 5 6 2 3 ( 5789 578: 2

VnDoc - Tải tài liệu, văn bản pháp luật, biểu mẫu miễn phí

A. meat B. apples C. water D. fish

9 Why are wild animals in _________. - Because we are cutting and burning the forests.

A. dangerous B. dangerously C. danger D. endanger

10. The Great Wall of China is the world’s ………………………structure.

A. long B. longer C. longest D. the longest

11. Laura is from Canada. She speaks English and ________.

A. Vietnamese B. France C. Japanese D. French

12. We never go camping because we don’t have a _________.

A. tent B. camera C. fishing rod D. toothpaste

II. Find a mistake A, B, C or D: (1m)1. Hai is going visit his grandparents tomorrow morning.

A B C D

3. Thoa wants to buy a tube of soap.

A B C D

4. How many milk does Mrs. Thai’s cows produce? – A little.

A B C D

2. Thuan always is late for school in the morning.

A B C D

III. Read the passage then choose (true)T or (false) F: (2ms)

BUDAPESTBudapest has a population of over two million people. One in every five Hungarian lives there. The River

Danube flows inside the city. The town on the west bank is older and beautiful. The town on the east bank is

bigger and modern. There are six bridges across the river.

The public transport system in Budapest is one of the cheapest in the world. You can travel easily by bus,

tram, or taxi. But driving a car in Budapest is not a good idea! There are not many car parks. The cars are old,

and the pollution is bad.

New words:

Budapest: Thủ đô của nước Hungary

modern: hiện đại

east: phía đông

west: phía tây

Page 61: Tài liệu, học tập, trắc nghiệm, tiếng anh, văn bản, biểu ...s1.vndoc.com/.../06/40-de-thi-hoc-ky...6-co-dap-an.pdf · "/ 0. 1 2 %$ 3 ( 0 4 5 6 2 3 ( 5789 578: 2

VnDoc - Tải tài liệu, văn bản pháp luật, biểu mẫu miễn phí

public transport system: hệ thống giao thông công cộng

trạm: tàu điện

bank: bờ sông

T F

1. Budapest has a population of over two thousand people.

2. The River Danube flows inside the city.

3. The town on the west bank is older than the one in the east bank.

4. The town on the east bank is modern.

5. There are five bridges across the river.

6. The public transport system in Budapest is cheap.

7. It’s not good to drive a car in Budapest.

8. The pollution is good in Budapest.

IV. Read the passage and answer the questions below: (2ms)

Miss Anh is our teacher. She teaches us Literature. She is tall and think but she is not weak. She plays a lot

of sports. Her favorite sports is badminton. She plays it three times a week. She likes jogging, too. She jogs in the

park near her house every morning. In her free time she often listens to music and watches TV.

1. What does Miss Anh?

………………………………………………………………………………………………………………….

2. Which sports does she play?

………………………………………………………………………………………………………………….

3. How often does she play badminton?

………………………………………………………………………………………………………………….

4. What does she often do in her free time?

………………………………………………………………………………………………………………….

V. Rewrite the sentence using the given words: (2ms)

1. My mother usually drives to work.

My mother usually goes ………………………………………………………………………………………

2. No river in the world is longer than The Nile river.

The Nile ………………………………………………………………………………………………………….

3. Ho Chi Minh city is bigger than Da Nang city.

Da Nang city……………………………………………………………………………………………………….

Page 62: Tài liệu, học tập, trắc nghiệm, tiếng anh, văn bản, biểu ...s1.vndoc.com/.../06/40-de-thi-hoc-ky...6-co-dap-an.pdf · "/ 0. 1 2 %$ 3 ( 0 4 5 6 2 3 ( 5789 578: 2

VnDoc - Tải tài liệu, văn bản pháp luật, biểu mẫu miễn phí

4. Vietnam has lots of beautiful beaches.

There are ……………………………………………………………………………………………………

The end- Good luck!

Page 63: Tài liệu, học tập, trắc nghiệm, tiếng anh, văn bản, biểu ...s1.vndoc.com/.../06/40-de-thi-hoc-ky...6-co-dap-an.pdf · "/ 0. 1 2 %$ 3 ( 0 4 5 6 2 3 ( 5789 578: 2

VnDoc - Tải tài liệu, văn bản pháp luật, biểu mẫu miễn phí

ANSWER KEYSĐỀ 1KEY FOR PART I:

1. B. long black hair 2. D. blue 3. C. water

4. C. longest 5. D. French 6. B. to eat

7. A. a bottle of 8. D. is riding 9. A. tent

10. B. Which sports 11. C. danger 12. A. How often

KEY FOR PART II:

1.B 2.A 3.C 4.A

KEY FOR PART III:

T F

1. Budapest has a population of over two thousand people. F

2. The River Danube flows inside the city. T

3. The town on the west bank is older than the one in the east bank. T

4. The town on the east bank is modern. T

5. There are five bridges across the river. F

6. The public transport system in Budapest is cheap. T

7. It’s not good to drive a car in Budapest. T

8. The pollution is good in Budapest. F

KEY FOR PART IV:

1. Miss Anh / She is a teacher. (0,5m)

Nếu: Miss Anh / She is our teacher. (0,25m)

2. She plays badminton and goes jogging. (0,5m)

Nếu chỉ: She plays badminton hoặc: She goes jogging. ( 0,25m)

3. She plays it three times a week. Hoặc: Three times a week. (0,5m)

Nếu: She play it three times a week. (0,25m)

4. In her free time she often listens to music and watches TV. (0,5m)

Nếu: Listening to music and watching TV. (0.5m)

Page 64: Tài liệu, học tập, trắc nghiệm, tiếng anh, văn bản, biểu ...s1.vndoc.com/.../06/40-de-thi-hoc-ky...6-co-dap-an.pdf · "/ 0. 1 2 %$ 3 ( 0 4 5 6 2 3 ( 5789 578: 2

VnDoc - Tải tài liệu, văn bản pháp luật, biểu mẫu miễn phí

Nếu: In her free time she often listen to music and watches TV. (0,25m)

Nếu: In her free time she often listens to music and watch TV. (0,25m)

Nếu: In her free time she often listen to music and watch TV. (0 m)

KEY FOR PART V:

1. There are lots of beautiful beaches in Viet Nam. (0,5m)2. The Nile is the longest river in the world.

3. Da Nang city is not as big as Ho Chi Minh city. (0,5m)

Hoặc: Da Nang city is smaller than Ho Chi Minh city/ one. (0,5m)

4. My mother usually goes to work by car. (0,5m)

ĐỀ 2KEY FOR PART I:

1. D. French 2. B. to eat 3. A. a bottle of

4. D. is riding 5. C. danger 6. A. How often

7. B. long black hair 8. D. blue 9. C. water

10. C. longest 11. A. tent 12. B. Which sports

KEY FOR PART II:

1.C 2.A 3.B 4.A

KEY FOR PART III:

T F

1. Budapest has a population of over two thousand people. F

2. The River Danube flows inside the city. T

3. The town on the west bank is older than the one in the east bank. T

4. The town on the east bank is modern. T

5. There are five bridges across the river. F

6. The public transport system in Budapest is cheap. T

7. It’s not good to drive a car in Budapest. T

8. The pollution is good in Budapest. F

Page 65: Tài liệu, học tập, trắc nghiệm, tiếng anh, văn bản, biểu ...s1.vndoc.com/.../06/40-de-thi-hoc-ky...6-co-dap-an.pdf · "/ 0. 1 2 %$ 3 ( 0 4 5 6 2 3 ( 5789 578: 2

VnDoc - Tải tài liệu, văn bản pháp luật, biểu mẫu miễn phí

KEY FOR PART IV:1. Miss Anh / She is a teacher. (0,5m)

Nếu: Miss Anh / She is our teacher. (0,25m)

2. She plays badminton and goes jogging. (0,5m)

Nếu chỉ: She plays badminton hoặc: She goes jogging. ( 0,25m)

3. She plays it three times a week. Hoặc: Three times a week. (0,5m)

Nếu: She play it three times a week. (0,25m)

4. In her free time she often listens to music and watches TV. (0,5m)

Nếu: Listening to music and watching TV. (0.5m)

Nếu: In her free time she often listen to music and watches TV. (0,25m)

Nếu: In her free time she often listens to music and watch TV. (0,25m)

Nếu: In her free time she often listen to music and watch TV. (0 m)

KEY FOR PART V:1. Da Nang city is not as big as Ho Chi Minh city. (0,5m)

Hoặc: Da Nang city is smaller than Ho Chi Minh city. (0,5m)

2. My mother usually goes to work by car. (0,5m)

3. There are lots of beautiful beaches in Viet Nam. (0,5m)4. The Nile is the longest river in the world.

ĐỀ 3

KEY FOR PART I:

1. C. longest 2. D. French 3. B. long black hair

4. D. blue 5. C. water 6. B. to eat

7. B. Which sports 8. C. danger 9. A. How often

10. A. a bottle of 11. D. is riding 12. A. tent

KEY FOR PART II:

1.A 2.B 3.C 4.A

KEY FOR PART III:

T F

Page 66: Tài liệu, học tập, trắc nghiệm, tiếng anh, văn bản, biểu ...s1.vndoc.com/.../06/40-de-thi-hoc-ky...6-co-dap-an.pdf · "/ 0. 1 2 %$ 3 ( 0 4 5 6 2 3 ( 5789 578: 2

VnDoc - Tải tài liệu, văn bản pháp luật, biểu mẫu miễn phí

1. Budapest has a population of over two thousand people. F

2. The River Danube flows inside the city. T

3. The town on the west bank is older than the one in the east bank. T

4. The town on the east bank is modern. T

5. There are five bridges across the river. F

6. The public transport system in Budapest is cheap. T

7. It’s not good to drive a car in Budapest. T

8. The pollution is good in Budapest. F

KEY FOR PART IV:1. Miss Anh / She is a teacher. (0,5m)

Nếu: Miss Anh / She is our teacher. (0,25m)

2. She plays badminton and goes jogging. (0,5m)

Nếu chỉ: She plays badminton hoặc: She goes jogging. ( 0,25m)

3. She plays it three times a week. Hoặc: Three times a week. (0,5m)

Nếu: She play it three times a week. (0,25m)

4. In her free time she often listens to music and watches TV. (0,5m)

Nếu: Listening to music and watching TV. (0.5m)

Nếu: In her free time she often listen to music and watches TV. (0,25m)

Nếu: In her free time she often listens to music and watch TV. (0,25m)

Nếu: In her free time she often listen to music and watch TV. (0 m)

KEY FOR PART V:1. My mother usually goes to work by car. (0,5m)

2. Da Nang city is not as big as Ho Chi Minh city. (0,5m)

Hoặc: Da Nang city is smaller than Ho Chi Minh city. (0,5m)

3. There are lots of beautiful beaches in Viet Nam. (0,5m)4. The Nile is the longest river in the world.

ĐỀ 4

KEY FOR PART I:

1. B. to eat 2. A. a bottle of 3. D. is riding

Page 67: Tài liệu, học tập, trắc nghiệm, tiếng anh, văn bản, biểu ...s1.vndoc.com/.../06/40-de-thi-hoc-ky...6-co-dap-an.pdf · "/ 0. 1 2 %$ 3 ( 0 4 5 6 2 3 ( 5789 578: 2

VnDoc - Tải tài liệu, văn bản pháp luật, biểu mẫu miễn phí

4. B. Which sports 5. A. How often 6. B. long black hair

7. D. blue 8. C. water 9. C. danger

10. C. longest 11. D. French 12. A. tent

KEY FOR PART II:

1.B 2.C 3.A 4.A

KEY FOR PART III:

T F

1. Budapest has a population of over two thousand people. F

2. The River Danube flows inside the city. T

3. The town on the west bank is older than the one in the east bank. T

4. The town on the east bank is modern. T

5. There are five bridges across the river. F

6. The public transport system in Budapest is cheap. T

7. It’s not good to drive a car in Budapest. T

8. The pollution is good in Budapest. F

KEY FOR PART IV:1. Miss Anh / She is a teacher. (0,5m)

Nếu: Miss Anh / She is our teacher. (0,25m)

2. She plays badminton and goes jogging. (0,5m)

Nếu chỉ: She plays badminton hoặc: She goes jogging. ( 0,25m)

3. She plays it three times a week. Hoặc: Three times a week. (0,5m)

Nếu: She play it three times a week. (0,25m)

4. In her free time she often listens to music and watches TV. (0,5m)

Nếu: Listening to music and watching TV. (0.5m)

Nếu: In her free time she often listen to music and watches TV. (0,25m)

Nếu: In her free time she often listens to music and watch TV. (0,25m)

Nếu: In her free time she often listen to music and watch TV. (0 m)

Page 68: Tài liệu, học tập, trắc nghiệm, tiếng anh, văn bản, biểu ...s1.vndoc.com/.../06/40-de-thi-hoc-ky...6-co-dap-an.pdf · "/ 0. 1 2 %$ 3 ( 0 4 5 6 2 3 ( 5789 578: 2

VnDoc - Tải tài liệu, văn bản pháp luật, biểu mẫu miễn phí

KEY FOR PART V:1. My mother usually goes to work by car. (0,5m)

2. The Nile is the longest river in the world.3. Da Nang city is not as big as Ho Chi Minh city. (0,5m)

Hoặc: Da Nang city is smaller than Ho Chi Minh city. (0,5m)

4. There are lots of beautiful beaches in Viet Nam. (0,5m)

Page 69: Tài liệu, học tập, trắc nghiệm, tiếng anh, văn bản, biểu ...s1.vndoc.com/.../06/40-de-thi-hoc-ky...6-co-dap-an.pdf · "/ 0. 1 2 %$ 3 ( 0 4 5 6 2 3 ( 5789 578: 2

VnDoc - Tải tài liệu, văn bản pháp luật, biểu mẫu miễn phí

UBND HUYỆN HOÀI NHƠNPHÒNG GD&ĐT HOÀI NHƠN

--------------------

ĐỀ THI HỌC KỲ IINĂM HỌC 2014 -2015

Môn: Tiếng Anh 6 Thí điểm – Ngày thi: ...........

Thời gian: 45 phút (không kể thời gian phát đề)

Họ và tên :……………………………

Trường : …………………...…………

Lớp : 6A......

Giám thị 1: Giám thị 2 : Mã phách

---------------------------------------------------------------------------------------------------------------------

Điểm bằng số Điểm bằng chữ Giám khảo 1 Giám khảo 2 Mã ghi lại

Đề 1A. LISTENING (2MS)

Part 1 . Listen and Number.(1m)

1 a. Wireless TV

b. automatic washing machine

c. modern fridge

d. hi-tech robot

e. automatic dishwasher

Part 2 . Listen and fill in the gaps: (1m)

1. Hai practises at the ……………………….. three times a week.

2. Trung can …………….. the guitar.

3. Alice likes ………………… ice skating.

4. Trung …………………. swimming on hot days.

B. LEXICO-GRAMMAR (2.5MS)

I. Choose the best answer: (1.5 point)1. ………..……the program is late, we will wait to watch it.

A. When B. Because C. Although D. So

Page 70: Tài liệu, học tập, trắc nghiệm, tiếng anh, văn bản, biểu ...s1.vndoc.com/.../06/40-de-thi-hoc-ky...6-co-dap-an.pdf · "/ 0. 1 2 %$ 3 ( 0 4 5 6 2 3 ( 5789 578: 2

VnDoc - Tải tài liệu, văn bản pháp luật, biểu mẫu miễn phí

2. As it doesn’t snow in Vietnam, we cannot ………………….skiing.

A. go B. play C. do D. are

3 The ………………….has a fish’s head and a lion’s body in Singapore

A. Big Ben B. Temple C. Merlion D. Eiffel Tower

4. My brother likes watching TV………..…… I like going out with my friends

A. and B. but C. or D. so

5.Find the words which have sounds / a:/

A. apple B. glass C. hand D. plant

6…………… means creating new products from used materials

A. reuse B. recycle C. reduce D. replant

II. Underline the best answer in these sentences below: (1 point)Ex: He……..tall and thin (am/ is/ are/ has).

1. I ………………(read / am reading/ have read) this book many times. I like it very much .

2. If we cut down more trees, there …………….(are/were / have been/ will be) more floods

3. Tuan Anh ……………….(have never seen/ has never seen/ never has seen) a giraffe before.

4. My dog is the ……………(friendliest/ most friendly/ most friendliest) dog in the village.

C. READING (3 MS)I. Fill in each blank with one suitable word to complete the passage bellow: (1 point)

exciting tourists quiet make well-known experience

Considered Vietnam’s most famous seaside resort town, Nha Trang attracts (1)…………..not only

with its beautiful beaches but also with its (2)……………………atmosphere of a young, growing

city. If you visit Nha Trang during the period of January to August you

will(3)…………………….the most wonderful weather for swimming and sunbathing. The town is

also (4)………………….for its seafood including dishes made from fresh fish , crabs, and lobsters

II. Read the passage and answer the following questions: (2,0 points)REBECCA STEVENS

Rebecca Stevens was the first woman to climb Mount Everest. Before she went up the highest

mountain in the world, she was a journalist and lived in South London.

In 1993, Rebecca left her job and her family and traveled to Asia with some other climbers. She

found that life on Everest is very difficult. “You must carry everything on your back,” she explained,

“ so you can only take things that you will need. You can’t wash on the mountain, and in the end I

didn’t even take a toothbrush. I am usually a clean person but there is no water, only snow. Water is

very heavy so you only take enough to drink!”

Page 71: Tài liệu, học tập, trắc nghiệm, tiếng anh, văn bản, biểu ...s1.vndoc.com/.../06/40-de-thi-hoc-ky...6-co-dap-an.pdf · "/ 0. 1 2 %$ 3 ( 0 4 5 6 2 3 ( 5789 578: 2

VnDoc - Tải tài liệu, văn bản pháp luật, biểu mẫu miễn phí

Rebecca became famous when she reached the top of Mount Everest on May 17, 1993. After that,

she wrote a book about the trip and people often asked her to talk about it. She got a new job too, on

a science program on television.

Questions:

1. Where did Rebecca Stevens come from ?

………………………………………..

2. What did she do before she climbed Everest ?

...............................................................................

3. How is life on Everest?

.............................................................................

4. Did Rebecca become famous on May 17, 1992 ?

..............................................................…………………………….

D. WRITING (2.5MS)I. Complete the sentences using the words and the given pictures: (1m)

1. If people use the ………………… bags and put

trash into the ……………………, they will save the

environment.

2. Robots will be able to do the ………………and

cut the ……………………. for us.

II. Rearrange the words to make a meaningful sentence: (0.5ms)1 TV/ often/ do / How / watch / you / ?/.

…..………………………………………………………………………….

2. in/ My/ the world / hometown / the / is / beautiful / most / place.

…………………………………………………………………………................................................

III. Rewrite these sentences (1 ms)

1. You save electricity. You turn off your computer when you don’t use.

If ………………………………………………………………………………

2. We will probably go to the moon for our summer holidays in 2050.

We might……………………………………………………………………….

Page 72: Tài liệu, học tập, trắc nghiệm, tiếng anh, văn bản, biểu ...s1.vndoc.com/.../06/40-de-thi-hoc-ky...6-co-dap-an.pdf · "/ 0. 1 2 %$ 3 ( 0 4 5 6 2 3 ( 5789 578: 2

VnDoc - Tải tài liệu, văn bản pháp luật, biểu mẫu miễn phí

UBND HUYỆN HOÀI NHƠNPHÒNG GD&ĐT HOÀI NHƠN

--------------------

ĐỀ THI HỌC KỲ IINĂM HỌC 2014 -2015

Môn: Tiếng Anh 6 Thí điểm – Ngày thi: ...........

Thời gian: 45 phút (không kể thời gian phát đề)

Họ và tên :……………………………

Trường : …………………...…………

Lớp : 6A......

Giám thị 1: Giám thị 2 : Mã phách

---------------------------------------------------------------------------------------------------------------------

Điểm bằng số Điểm bằng chữ Giám khảo 1 Giám khảo 2 Mã ghi lại

Đề 2A. LISTENING (2MS)

Part 1 . Listen and Number.(1m)

1 a. Wireless TV

b. hi-tech robot

c. modern fridge

d. automatic washing machine

e. automatic dishwasher

Part 2 . Listen and fill in the gaps: (1m)

1. Hai practises at the ……………………….. three times a week.

2. Alice likes ………………… ice skating.

3. Trung can …………….. the guitar.

4. Trung …………………. swimming on hot days.

B. LEXICO-GRAMMAR (2.5MS)I. Choose the best answer: (1.5 point)

1. My brother likes watching TV………..…… I like going out with my friends

A. and B. but C. or D. so

2. Find the words which have sounds / a:/

Page 73: Tài liệu, học tập, trắc nghiệm, tiếng anh, văn bản, biểu ...s1.vndoc.com/.../06/40-de-thi-hoc-ky...6-co-dap-an.pdf · "/ 0. 1 2 %$ 3 ( 0 4 5 6 2 3 ( 5789 578: 2

VnDoc - Tải tài liệu, văn bản pháp luật, biểu mẫu miễn phí

A. apple B. glass C. hand D. plant

3 …………… means creating new products from used materials.

A. reuse B. recycle C. reduce D. replant

4. ………..……the program is late, we will wait to watch it.

A. When B. Because C. Although D. So

5. As it doesn’t snow in Vietnam , we cannot ………………….skiing.

A. go B. play C. do D. are

6 The ………………….has a fish’s head and a lion’s body in Singapore

A. Big Ben B. Temple C. Merlion D. Eiffel Tower

II. Underline the best answer in these sentences below: ( 1 point)

Ex: He……..tall and thin ( am/is/are/has).

1. I ………………(read/ have read/ am reading) this book many times. I like it very much .

2. If we cut down more trees, there …………….(are/ will be/ were /have been/) more floods

3. Tuan Anh ……………….(has never seen/ have never seen/ never has seen) a giraffe before.

4. My dog is the ……………(most friendliest/ most friendly/ friendliest) dog in the village.

C. READING (3 MS)

I. Fill in each blank with one suitable word to complete the passage bellow: (1point)

experience exciting make tourists quiet well-known

Considered Vietnam’s most famous seaside resort town, Nha Trang attracts (1)…………..not only

with its beautiful beaches but also with its (2)……………………atmosphere of a young, growing

city. If you visit Nha Trang during the period of January to August you

will(3)…………………….the most wonderful weather for swimming and sunbathing. The town is

also (4)………………….for its seafood including dishes made from fresh fish , crabs, and lobsters

II. Read the passage and answer the following questions: (2,0 points)REBECCA STEVENS

Rebecca Stevens was the first woman to climb Mount Everest. Before she went up the highest

mountain in the world, she was a journalist and lived in South London.

In 1993, Rebecca left her job and her family and traveled to Asia with some other climbers. She

found that life on Everest is very difficult. “ You must carry everything on your back,” she

explained, “ so you can only take things that you will need. You can’t wash on the mountain, and in

the end I didn’t even take a toothbrush. I am usually a clean person but there is no water, only snow.

Water is very heavy so you only take enough to drink!”

Page 74: Tài liệu, học tập, trắc nghiệm, tiếng anh, văn bản, biểu ...s1.vndoc.com/.../06/40-de-thi-hoc-ky...6-co-dap-an.pdf · "/ 0. 1 2 %$ 3 ( 0 4 5 6 2 3 ( 5789 578: 2

VnDoc - Tải tài liệu, văn bản pháp luật, biểu mẫu miễn phí

Rebecca became famous when she reached the top of Mount Everest on May 17, 1993. After that,

she wrote a book about the trip and people often asked her to talk about it. She got a new job too, on

a science program on television.

Questions:

1. Where did Rebecca Stevens come from?

………………………………………..

2. What did she do before she climbed Everest?

...............................................................................

3. How is life on Everest?

.............................................................................

4. Did Rebecca become famous on May 17, 1992?

..............................................................…………………………….

D. WRITING (2.5MS)

I. Complete the sentences using the words and the given pictures: (1m)

1. If people use the ………………… bags and put trash

into the ……………………, they will save the

environment.

2. Robots will be able to do the ………………and cut

the ……………………. for us.

II. Rearrange the words to make a meaningful sentence: (0.5ms)

1 in/ My/ the world / hometown / the / is / beautiful / most / place .

…..………………………………………………………………………….

2. TV/ often/ do / How / watch / you / ?/ .

…………………………………………………………………………................................................

III. Rewrite these sentences (1 ms)1. You save electricity. You turn off your computer when you don’t use.

If ………………………………………………………………………………

2. We will probably go to the moon for our summer holidays in 2050.

We might……………………………………………………………………….

Page 75: Tài liệu, học tập, trắc nghiệm, tiếng anh, văn bản, biểu ...s1.vndoc.com/.../06/40-de-thi-hoc-ky...6-co-dap-an.pdf · "/ 0. 1 2 %$ 3 ( 0 4 5 6 2 3 ( 5789 578: 2

VnDoc - Tải tài liệu, văn bản pháp luật, biểu mẫu miễn phí

ANSWER KEYSĐề 1A. Listening:

I( 4 câu x 0,25)a. 1 b. 4 c.3 d. 5 e.2

II( 4 câu x 0,25): Không tính điểm cho lỗi sai chính tả1. (judo) club 2. play 3. watching 4. goes

B. LEXICO-GRAMMARI. (6 câu x 0,25)

1.C 2.A 3C 4B

5. B & D (khoanh một đáp án không được điểm) 6B

II. (4 câu x 0,25)1. have read 2. will be 3. has never seen 4. friendliest

C. READINGI. (4 câu x 0,25)

1. tourists 2. exciting 3. experience 4.well-known

II. (4 câu x 0,5)

1. Rebecca Stevens came from England.

2. She was a journalist.

3. Life on Everest is very difficult.

4. No, she didn’t.

D. WRITINGI. (4 từ x 0,25)

1. reusable– recycling bin/ garbage bin

2. laundry - grass

II. (2 câu x 0,25)1. How often do you watch TV?

2. My hometown is the most beautiful place in the world.

III. (2 câu x 0,5)1. If you turn off your computer when you don’t use, You will save electricity.

2. We might go to the moon for our summer holidays in 2050.

Đề 2

Page 76: Tài liệu, học tập, trắc nghiệm, tiếng anh, văn bản, biểu ...s1.vndoc.com/.../06/40-de-thi-hoc-ky...6-co-dap-an.pdf · "/ 0. 1 2 %$ 3 ( 0 4 5 6 2 3 ( 5789 578: 2

VnDoc - Tải tài liệu, văn bản pháp luật, biểu mẫu miễn phí

A.Listening:I. (4 câu x 0,25)

a. 1 b. 5 c.3 d. 4 e.2

II. (4 câu x 0,25): Không tính điểm cho lỗi sai chính tả

1. (judo) club 2. watching 3. play 4. goes

B. LEXICO-GRAMMAR

I. (6 câu x 0,25)1. B 2. B & D (khoanh một đáp án không được điểm)

3. B 4. C 5. A 6. C

II. (4 câu x 0,25)

1. have read 2. will be 3. has never seen 4. friendliest

C. READING

I. (4 câu x 0,25)1. tourists 2. exciting 3. experience 4.well-known

II. (4 câu x 0,5)1. Rebecca Stevens came from England.

2. She was a journalist.

3. Life on Everest is very difficult.

4. No, she didn’t.

D. WRITING

I. (4 từ x 0,25)1. reusable– recycling bin/ garbage bin

2. laundry - grass

II. (2 câu x 0,25)

1.. My hometown is the most beautiful place in the world

2. How often do you watch TV?

III. (2 câu x 0,5)1. If you turn off your computer when you don’t use, You will save electricity.

2. We might go to the moon for our summer holidays in 2050.

Page 77: Tài liệu, học tập, trắc nghiệm, tiếng anh, văn bản, biểu ...s1.vndoc.com/.../06/40-de-thi-hoc-ky...6-co-dap-an.pdf · "/ 0. 1 2 %$ 3 ( 0 4 5 6 2 3 ( 5789 578: 2

VnDoc - Tải tài liệu, văn bản pháp luật, biểu mẫu miễn phí

PHÒNG GD&ĐT LÀO CAITRƯỜNG THCS VẠN HÒA

----------------------(Đề thi gồm 02 trang)

ĐỀ KIỂM TRA HỌC KỲ IINĂM HỌC 2015 - 2016

MÔN: TIẾNG ANH - LỚP 6Thời gian làm bài: 45 phút

(không kể thời gian giao đề)

Mark Teacher’s comment

Section I : Listening (2ps)

Question 1: Listen to the text and answer the following questions. (1p)1. What are Vui and her four friends planning?

.................................................................................................................................

2. What is Vui going to bring?

..............................................................................................................................

3. What is Ly going to bring?

................................................................................................................................

4. What is Lan going to bring?

..............................................................................................................................

Question 2: Listen and write the numbers you hear. (1pt)

1) 2 12 22 20

2) 09 119 190 19

3) 13 33 73 53

4) 1001 1010 1100

Section II: Grammar and Vocabulary (3ps)Question 3: Which is different? (0,5p)1- A. weak B. season C. beach D. weather

2- A. favorite B. cake C. table D. camera

Question 4: - Which is different? (0,5p)

1- A. Badminton B. Picnic C. basketball D. volleyball

2- A. Mountain B. Beach C. Country D. Forest

Page 78: Tài liệu, học tập, trắc nghiệm, tiếng anh, văn bản, biểu ...s1.vndoc.com/.../06/40-de-thi-hoc-ky...6-co-dap-an.pdf · "/ 0. 1 2 %$ 3 ( 0 4 5 6 2 3 ( 5789 578: 2

VnDoc - Tải tài liệu, văn bản pháp luật, biểu mẫu miễn phí

Question 5: - Choose the best answer. (1p)1. We buy of potatoes. My son likes potatoes very much.

A. a lot B. some C. any D. a little

2. Mai ……………Math in her room at the moment?

A.studying B. studys C. Is Mai studying D. studies

3. are you going to stay there? - For a week.

A. How often B. How

C. How much D. How long

4. Ba doesn’t play badminton when it’s cold. He plays badminton when it’s cold

A. never B. sometimes C. often D. always

Question 6: - Fill in the correct verb form (1p)1. Nam likes.............................(watch) cartoon.

2. You should ……….. (save) water.

3. ............................................... (visit) Sapa this summer vacation?- Yes, I am.

4. Let’s………………….. (travel) by bus.

Section III: Reading (2,5ps)

Question 7- Read the text and answer the question. (1,25ps)There are four seasons in a year. They are: spring, summer, fall and winter. In spring, the

weather is usually warm. Sometimes, it is cold, but not very cold. There are many flowers in spring.

after spring comes summer. In summer, the days are long and the nights are short. We often go to

the seaside in summer. Fall is the season of fruits.We can enjoy different kinds of fruits, especially

in tropical countries like Vietnam. In winter, it is usually very cold. The days are short and the

nights are long.

1- How many seasons are there in Vietnam?

→ . . . . . . . . . . . . . . . . . . . . . . . . . . . . . . . . . . . . . . . . . . . . . . . . . . . . . . . . . . . . . . .

2- What are they?

→ . . . . . . . . . . . . . . . . . . . . . . . . . . . . . . . . . . . . . . . . . . . . . . . . . . . . . . . . . . . . . . .

3- Are the days long in winter?

→ . . . . . . . . . . . . . . . . . . . . . . . . . . . . . . . . . . . . . . . . . . . . . . . . . . . . . . . . . . . . . . .

4- What is the weather like in spring?

→ . . . . . . . . . . . . . . . . . . . . . . . . . . . . . . . . . . . . . . . . . . . . . . . . . . . . . . . . . . . . . . .

5- Which season does it comes after spring?

→ . . . . . . . . . . . . . . . . . . . . . . . . . . . . . . . . . . . . . . . . . . . . . . . . . . . . . . . . . . . . . . .

Question 8. Read the text and fill in the blanket. (1,25ps)

Page 79: Tài liệu, học tập, trắc nghiệm, tiếng anh, văn bản, biểu ...s1.vndoc.com/.../06/40-de-thi-hoc-ky...6-co-dap-an.pdf · "/ 0. 1 2 %$ 3 ( 0 4 5 6 2 3 ( 5789 578: 2

VnDoc - Tải tài liệu, văn bản pháp luật, biểu mẫu miễn phí

Fishing is my favorite ……….. (1). I often fish……………...(2) hours without catching

anything. But this does not worry me. Some fishermen are unlucky. Instead of catching fish, they

catch rubbish. I am even less lucky. I never catch anything – not even rubbish. I always go

home ……………..(3) an empty bag. “You must give up fishing!” my friends say. It’s waste of

time. But they don’t know one important thing: I don’t really ………….. (4) fishing. I only want

to ………………..(5) in a boat and do nothing at all.

Section IV: Writing (2,5ps)Question 9. Complete the second sentence so that it has a similar meaning to the first one.

Write the answers on your answer paper. (1ps)1. The blue dress is cheaper than the red one.

The red dress is............................................................................

2. Nam is taller than any students in my class.

Nam is the...................................................................................

3. There are 500 students in my school.

My school..................................................................................

4. Why don't we go to Huong Pagoda?

What about ...............................................................?

Question 10 - Viết về kế hoạch của em trong kỳ nghỉ hè dựa trên các câu hỏi gợi ý sau (1,5ps)

1, What are you going to do?

2, How are you going to travel?

3, Where are you going to stay?

4, How long are you going to stay?

5.Who do you go with?

6.How do you feel after the holiday?

Page 80: Tài liệu, học tập, trắc nghiệm, tiếng anh, văn bản, biểu ...s1.vndoc.com/.../06/40-de-thi-hoc-ky...6-co-dap-an.pdf · "/ 0. 1 2 %$ 3 ( 0 4 5 6 2 3 ( 5789 578: 2

VnDoc - Tải tài liệu, văn bản pháp luật, biểu mẫu miễn phí

HƯỚNG DẪN CHẤM ĐỀ KIỂM TRA HỌC K Ỳ IINĂM HỌC 2015 - 2016

MÔN: TIẾNG ANH - LỚP 6A- Cách chấm

- Chấm theo thang điểm 10

- Lấy điểm lẻ nhỏ nhất đến 0,25

B- ĐÁP ÁN VÀ BIỂU ĐIỂMSection I: Listening (2ps)

Question 1Tape-script:

Vui and her four friends are planning a camping vacation. They are going to to camp for three days.

What are they going to bring?

Vui and her friends are going to camp for three days in Sa Pa. Vui is going to bring a tent and some

food. Lan is going to bring a ball. Ly is going to bring her camera to take some photos. Nga and

Mai are going to bring some drinks.

1. They are planning a camping vacation.

2. She is going to bring a tent and some food.

3. She is going to bring her camera.

4. She is going to bring a ball.

5. They are going to bring some drinks.

Question 2: Listen and write the numbers you hear(1 pt)22 - 190 – 13 - 1010

Section II: Grammar and Vocabulary (3ps)I - (0,5 điểm - Mỗi câu trả lời được 0,25 điểm)

1- D. weather 2- D. camera

II - (0,5 điểm - Mỗi câu trả lời được 0,25 điểm)

1- B. Picnic 2- C. County

III- (1 điểm - Mỗi câu trả lời được 0,25 điểm)

Câu 1 2 3 4

Đáp án A C D A

IV- (2 điểm - Mỗi câu trả lời được 0,5 điểm)

Page 81: Tài liệu, học tập, trắc nghiệm, tiếng anh, văn bản, biểu ...s1.vndoc.com/.../06/40-de-thi-hoc-ky...6-co-dap-an.pdf · "/ 0. 1 2 %$ 3 ( 0 4 5 6 2 3 ( 5789 578: 2

VnDoc - Tải tài liệu, văn bản pháp luật, biểu mẫu miễn phí

Câu 1 2 3 4

Đáp án watching save Are you going to visit travel

Section III : Reading (2,5ps)Read the text and fill in the blanket:

1. sport 2. for 3. with 4. like 5. sit

V- (3 điểm- Mỗi câu trả lời được 0,5 điểm)

1- There are four seasons in Vietnam.

2- They are: Spring, summer, fall and winter.

3- No, they are not.

4- The weather is usually warm in spring.

5- Summer comes after spring.

6- Yes, there are.

Section IV: Writing (2,5ps)VI- (2 điểm - Mỗi câu trả lời được 0,5 điểm)

1. I am going to visit………………

2. I am going to travel by………….

3. I am going to stay……………….

4. I am going to stay for…………...

Page 82: Tài liệu, học tập, trắc nghiệm, tiếng anh, văn bản, biểu ...s1.vndoc.com/.../06/40-de-thi-hoc-ky...6-co-dap-an.pdf · "/ 0. 1 2 %$ 3 ( 0 4 5 6 2 3 ( 5789 578: 2

VnDoc - Tải tài liệu, văn bản pháp luật, biểu mẫu miễn phí

I. Listen and choose the correct answers: (2p)1. What would Nhan like?

A. He’d like noodles B. He’d like some oranges

C. He’d like chicken D. He’d like some apples

2. What would Phuong like?

A. She’d like a hot drink B. She’d like a drink

C. She’d like some orange juice D. She’d like a cold drink

3. How does Ba feel?

A. He is hungry B. He is hot C. He is full D. He is thirsty

4. How does Huong feel?

A. She is full B. She is hot C. She is tired D. She is cold

II. Phonetics: (0.5p)

Choose the word that has the underlined part pronounced differently from the others:1. A.fall B.cat C.fat D.can

2. A.thirsty B.thirteen C.thirty D.tired

III. Read the text carefully and decided whether these statements are T/ F (2p)

Mrs Lien is a nurse. She is thirty-five years old. She lives in a house near a park. She works in

a hospital. She goes to work by car. She goes home at 5:00 pm. She is in her house and she is

cooking now.

True / False:

1/ Mrs Lien is thirty years old . _______

2/ She works in a hospital. _______

3/ She goes to work by car. _______

4/ She goes home at 6:00 pm _______

IV. Choose the correct answer. Only one answer is correct (2.5p)1/ Let’s ______ badminton.

A. to play B. plays C. play D. played

2/ Nga goes to the zoo three _________ a week.

PHÒNG GD&ĐT CHÂU THÀNHTrường THCS Biên Giới

---------------

ĐỀ THI KHẢO SÁT CHẤT LƯỢNG HKIINĂM HỌC 2015 – 2016

MÔN: TIẾNG ANH - LỚP 6Thời gian: 60 phút (không kể thời gian phát đề)

Page 83: Tài liệu, học tập, trắc nghiệm, tiếng anh, văn bản, biểu ...s1.vndoc.com/.../06/40-de-thi-hoc-ky...6-co-dap-an.pdf · "/ 0. 1 2 %$ 3 ( 0 4 5 6 2 3 ( 5789 578: 2

VnDoc - Tải tài liệu, văn bản pháp luật, biểu mẫu miễn phí

A. time B. twice C. times D. once

3/ We __________ at the moment.

A. is skip B. are skipping C. skip D. skips

4/ _________ do you watch T.V?- Twice a week.

A. How B. How often C. How much D. How old

5/ My brother ________ fishing on Sundays.

A. sometimes goes B. going C. go D.goes sometimes

6/ Is it _____ in the summer? Yes, it is.

A. warm B. cool C. cold D. hot

7/ There are _________ books on the table.

A. any B. some C. a D. an

8/ She isn’t heavy. She is ________ .

A. thin B. fat C.strong D. light

9/ Do you play soccer every day ?

A. Yes, I do. B. Yes, I play. C. No, I do. D. I play soccer.

10/ They are ________ badminton now.

A. play B. plays C. to play D. playing

V. Complete the questions with: How many / How much / How often / How / Which / What /

How long / Where (2p)1) ______________ do you go swimming? Once a week.

2) ______________ seasons are there in a year? Four.

3) ______________ beef do you want ? - Two kilos.

4) ______________ are you going to stay with your aunt ? For a week.

5) ______________ would you like? I’d like some noodles.

6) ______________ do you feel?

7) ______________ sports do you play?

8)________________ do you live?

VI. Do as directed (Làm theo yêu cầu) (1p)

1. The first season in the year. It’s the ________ . (Điền đúng từ chỉ mùa)

2. We go to the zoo on Mondays. (never) (Đặt trạng từ “never” vào vị trí đúng của câu)

...........................................................................................................................................

3. Mr. Minh goes to work six days a week. (Đặt câu hỏi cho cụm từ gạch dưới)

...........................................................................................................................................

4. It / often / cool / fall. (Viết câu từ những từ gợi ý)

Page 84: Tài liệu, học tập, trắc nghiệm, tiếng anh, văn bản, biểu ...s1.vndoc.com/.../06/40-de-thi-hoc-ky...6-co-dap-an.pdf · "/ 0. 1 2 %$ 3 ( 0 4 5 6 2 3 ( 5789 578: 2

VnDoc - Tải tài liệu, văn bản pháp luật, biểu mẫu miễn phí

...........................................................................................................................................

Page 85: Tài liệu, học tập, trắc nghiệm, tiếng anh, văn bản, biểu ...s1.vndoc.com/.../06/40-de-thi-hoc-ky...6-co-dap-an.pdf · "/ 0. 1 2 %$ 3 ( 0 4 5 6 2 3 ( 5789 578: 2

VnDoc - Tải tài liệu, văn bản pháp luật, biểu mẫu miễn phí

HƯỚNG DẪN CHẤM VÀ ĐÁP ÁNĐỀ THI KHẢO SÁT CHẤT LƯỢNG HKII - NĂM HỌC 2015 – 2016

MÔN: TIẾNG ANH - LỚP 6I. 2p

*Bài Nghe:Nhan is hungry. He’d like noodles

Phuong is thirsty. She’d like a drink.

Ba is full.

Huong is cold.

1. A 2. B 3. C 4. D

II. 0,5p1. A 2. D

III. 2p1. F 2. T 3. T 4. F

IV. 2,5p1. C 2. C 3. B 4. B 5. A

6. D 7. B 8. D 9. A 10. D

V. 2p

1. How often

2. How many

3. How much

4. How long

5. What

6. How

7. Which

8. Where

VI. 1p1. spring

2. We never go to the zoo on Mondays.

3. How often does Mr Minh go to work?

4. It is often cool in the fall.

Page 86: Tài liệu, học tập, trắc nghiệm, tiếng anh, văn bản, biểu ...s1.vndoc.com/.../06/40-de-thi-hoc-ky...6-co-dap-an.pdf · "/ 0. 1 2 %$ 3 ( 0 4 5 6 2 3 ( 5789 578: 2

VnDoc - Tải tài liệu, văn bản pháp luật, biểu mẫu miễn phí

I/ Circle the word whose underlined, bold part is pronounced differently from the others (1pt)1. A. sometimes B. spring C. idea D. twice

2. A. thick B. tooth C. this D. thirsty3. A. routine B. mouth C. mountain D. round

4. A. bottle B. box C. volleyball D. coldII/ Circle the best words or phrases to complete the sentences (2pts)

5. Lan is hungry. She’d like some …

A. orange juice B. noodles C. ice - cream D. a drink

6. What is … dinner today? – There is some meat and some rice.

A. in B. on C. at D. for

7. My daughter …. jogging every morning.

A. goes B. does C. makes D. takes

8. Her lips aren’t full. They are …

A. fat B. thin C. short D. tall

9. I don’t have … oranges but I have … apples.

A. any/ any B. some/ any C. some/some D. any/ some

10. Why don’t we go to the movies tonight? -………….

A. Yes, I do. B. I don’t think so. C. That’s a good idea. D. No, we don’t.

11. Tokyo is ……..than Mexico City.

A. small B. smaller C. smallest D. the smallest

12. Miss Chi has full lips and ….

A. teeth small white B. white small teeth C. white teeth small D. small white teeth

III/ Give the correct form or tense of the verb in each bracket (1pt)13. When it is hot, Ba (go) swimming.

14. There (not, be) any water in the bottle.

15. Lan and Nga (drink) iced tea at the moment.

PHÒNG GD&ĐT TÂN CHÂUTRƯỜNG THCS THỊ TRẤN TÂN CHÂU

-------------------

ĐỀ KIỂM TRA HỌC KỲ IINĂM HỌC 2013 - 2014

MÔN: TIẾNG ANH - LỚP 6Thời gian làm bài: 60 phút

Page 87: Tài liệu, học tập, trắc nghiệm, tiếng anh, văn bản, biểu ...s1.vndoc.com/.../06/40-de-thi-hoc-ky...6-co-dap-an.pdf · "/ 0. 1 2 %$ 3 ( 0 4 5 6 2 3 ( 5789 578: 2

VnDoc - Tải tài liệu, văn bản pháp luật, biểu mẫu miễn phí

16. Lan (visit) her grandmother tomorrow morning.

IV/ Match the questions in column A with the answers in column B. (1pt)

A B Answers

17. What is there to drink? A. It’s cool. 17.

18. How much are your shoes? B. She is Chinese. 18.

19. What is the weather like in the fall? C.They are fifty thousand dong. 19

20. What’s her nationality? D. There is some milk. 20

V/ Complete the passage with the suitable words from the box and answer the questions below(3pts)

never watching homework goes

Mai is a good student in my class. She is never late for school. At home, she often does her (21) and

helps her parents with the housework. She likes (22) television and reading Hoa Hoc Tro Newspaper.

She plays badminton and (23) swimming with her friends twice a week. She (24) plays video games in

her free time.

Answer the questions about Mai

25. Is Mai often late for school?

26. What does she often do at home?

27. How often does she go swimming?

28. Does she often play video games in her free time?

VI/ Rearrange the words into the meaningful sentences (1pt)29. meat/ much/ want/ you/ how/ do?

30. the/ aren’t/ kitchen/ cooking oil/ there/ bottles/ any/ of/ in

VII/ Answer about you (1pt)

31. What is your favorite food?

32. What are you going to do after exam?

…. THE END….

Page 88: Tài liệu, học tập, trắc nghiệm, tiếng anh, văn bản, biểu ...s1.vndoc.com/.../06/40-de-thi-hoc-ky...6-co-dap-an.pdf · "/ 0. 1 2 %$ 3 ( 0 4 5 6 2 3 ( 5789 578: 2

VnDoc - Tải tài liệu, văn bản pháp luật, biểu mẫu miễn phí

ANSWER KEY1. B

2. C

3. A

4. D

5. B

6. D

7. A

8. B

9. D

10. C

11. B

12. D

13. goes

14. isn’t

15. are drinking

16. is going to visit

17. D

18. C

19. A

20. B

21. homework

22. watching

23. goes

24. never

25. No, she isn’t.

26. She often does her homework and helps her parents with the housework.

27. She goes swimming twice a week.

28. No, she doesn’t.

29. How much meat do you want?

30. There aren’t any bottles of cooking oil in the kitchen.

Page 89: Tài liệu, học tập, trắc nghiệm, tiếng anh, văn bản, biểu ...s1.vndoc.com/.../06/40-de-thi-hoc-ky...6-co-dap-an.pdf · "/ 0. 1 2 %$ 3 ( 0 4 5 6 2 3 ( 5789 578: 2

VnDoc - Tải tài liệu, văn bản pháp luật, biểu mẫu miễn phí

31. I like … (chicken).

32. I am going to … (visit my grandmother).

THE END

Page 90: Tài liệu, học tập, trắc nghiệm, tiếng anh, văn bản, biểu ...s1.vndoc.com/.../06/40-de-thi-hoc-ky...6-co-dap-an.pdf · "/ 0. 1 2 %$ 3 ( 0 4 5 6 2 3 ( 5789 578: 2

VnDoc - Tải tài liệu, văn bản pháp luật, biểu mẫu miễn phí

I/ Circle the word whose underlined, bold part is pronounced differently from the others (1pt)1. A. sandwich B. apple C. tomato D. matter

2. A. fly B. usually C. activity D. finally3. A. breakfast B. weak C. heavy D. head

4. A. chocolate B. beach C. chest D. schoolII/ Circle the best words or phrases to complete the sentences (2pts)

5. Nam isn’t tall. He is ….

A. tall B. short C. thin D. fat

6. My mother has …

A. long hair black B. hair long black C. long black hair D. hair black long

7. … meat do you want? – A kilo, please.

A. How many B. How much C. How often D. How long

8. Minh is going to … some photos in Ha Long Bay.

A. make B. catch C. do D. take

9. Why don’t we go to the movies tonight? …

A. Yes, I do. B. No, I don’t. C. That’s a good idea. D. Yes, we do.

10. Mai and Phuong are going on ………….this summer vacation.

A. vacation B. trip C. way D. travel

11. I am going to stay ….. my uncle’s house.

A. for B. with C. at D. to

12. What is your ………..? – I am British.

A. language B. nationality C. country D. city

III/ Give the correct form or tense of the verb in each bracket (1pt)13. When it is cool, Nam (go) jogging.

14. What is Nga doing now? - She (play) volleyball.

15. I am hungry now? What is there (eat)?

PHÒNG GD&ĐT TÂN CHÂUTRƯỜNG THCS THỊ TRẤN TÂN CHÂU

-------------------

ĐỀ KIỂM TRA HỌC KỲ IINĂM HỌC 2012 - 2013

MÔN: TIẾNG ANH - LỚP 6Thời gian làm bài: 60 phút

Page 91: Tài liệu, học tập, trắc nghiệm, tiếng anh, văn bản, biểu ...s1.vndoc.com/.../06/40-de-thi-hoc-ky...6-co-dap-an.pdf · "/ 0. 1 2 %$ 3 ( 0 4 5 6 2 3 ( 5789 578: 2

VnDoc - Tải tài liệu, văn bản pháp luật, biểu mẫu miễn phí

16. Lan (visit) her grandmother tomorrow.

IV/ Match the questions in column A with the answers in column B. (1,5pts)

A B Answers

17. What color are your eyes? A. I’d like noodles. 17 -

18. How much is a sandwich? B. I like soccer. 18 -

19. What would you like for breakfast? C. They are black. 19 -

20. What’s your favorite sport? D. It’s five thousand dong. 20 -

V/ Complete the passage with the words from the box. (1,5pts)Then answer the questions below (1,5pts)

favorite times plays or but every

Miss Huong is a teacher. She teaches us history. She is tall and thin (21) she is not weak. She (22) a lot

of sports. Her (23) sport is aerobics. She does aerobics three (24) a week. She likes jogging, too. She

jogs in the park near her house (25) morning. In her free time, she listens to music (26) watches

television.

Answer the questions (1,5pts)

27. What does Miss Huong do?

28. Which sports does she play?

29. Does she listen to music in her free time?

VI/ Complete the sentences, using the words given (1pt)

30. What/ Nam/ do/ when/ it/ hot?

31. There/ not/ any/ bottles/ cooking oil/ the kitchen.

VII/ Answer about you (1pt)32. What do you do in your free time?

33. How often do you help your mom?

34. What is your favorite food?

35. What are you going to do this summer vacation?

…. THE END….

Page 92: Tài liệu, học tập, trắc nghiệm, tiếng anh, văn bản, biểu ...s1.vndoc.com/.../06/40-de-thi-hoc-ky...6-co-dap-an.pdf · "/ 0. 1 2 %$ 3 ( 0 4 5 6 2 3 ( 5789 578: 2

VnDoc - Tải tài liệu, văn bản pháp luật, biểu mẫu miễn phí

ANSWER KEYS1. C

2. A

3. B

4. D

5. B

6. C

7. B

8. D

9. C

10. A

11. C

12. B

13. goes

14. is playing

15. to eat

16. is going to visit

17. c

18. d

19. a

20. b

21. but

22. plays

23. favorite

24. times

25. every

26. or

27. She is a teacher.

28. She does aerobics.

29. Yes, she does.

30. What does Nam do when it’s hot?

Page 93: Tài liệu, học tập, trắc nghiệm, tiếng anh, văn bản, biểu ...s1.vndoc.com/.../06/40-de-thi-hoc-ky...6-co-dap-an.pdf · "/ 0. 1 2 %$ 3 ( 0 4 5 6 2 3 ( 5789 578: 2

VnDoc - Tải tài liệu, văn bản pháp luật, biểu mẫu miễn phí

31. There aren’t any bottles of cooking oil in the kitchen.

32. I …

33. I often help my mom.

34. I like fish.

35. I am going to visit Hue.

Page 94: Tài liệu, học tập, trắc nghiệm, tiếng anh, văn bản, biểu ...s1.vndoc.com/.../06/40-de-thi-hoc-ky...6-co-dap-an.pdf · "/ 0. 1 2 %$ 3 ( 0 4 5 6 2 3 ( 5789 578: 2

VnDoc - Tải tài liệu, văn bản pháp luật, biểu mẫu miễn phí

I/ Circle the word whose underlined, bold part is pronounced differently from the others (1pt)1. A. sandwich B. apple C. tomato D. matter

2. A. fly B. usually C. activity D. finally3. A. breakfast B. weak C. heavy D. head

4. A. chocolate B. beach C. chest D. schoolII/ Circle the best words or phrases to complete the sentences (2pts)

5. Nam isn’t tall. He is ….

A. tall B. short C. thin D. fat

6. My mother has …

A. long hair black B. hair long black C. long black hair D. hair black long

7. … meat do you want? – A kilo, please.

A. How many B. How much C. How often D. How long

8. Minh is going to … some photos in Ha Long Bay.

A. make B. catch C. do D. take

9. Why don’t we go to the movies tonight? …

A. Yes, I do. B. No, I don’t. C. That’s a good idea. D. Yes, we do.

10. Mai and Phuong are going on ………….this summer vacation.

A. vacation B. trip C. way D. travel

11. I am going to stay ….. my uncle’s house.

A. for B. with C. at D. to

12. What is your ………..? – I am British.

A. language B. nationality C. country D. city

III/ Give the correct form or tense of the verb in each bracket (1pt)13. When it is cool, Nam (go) jogging.

14. What is Nga doing now? - She (play) volleyball.

15. I am hungry now? What is there (eat)?

PHÒNG GD&ĐT TÂN CHÂUTRƯỜNG THCS THỊ TRẤN TÂN CHÂU

-------------------

ĐỀ KIỂM TRA HỌC KỲ IINĂM HỌC 2012 - 2013

MÔN: TIẾNG ANH - LỚP 6Thời gian làm bài: 60 phút

Page 95: Tài liệu, học tập, trắc nghiệm, tiếng anh, văn bản, biểu ...s1.vndoc.com/.../06/40-de-thi-hoc-ky...6-co-dap-an.pdf · "/ 0. 1 2 %$ 3 ( 0 4 5 6 2 3 ( 5789 578: 2

VnDoc - Tải tài liệu, văn bản pháp luật, biểu mẫu miễn phí

16. Lan (visit) her grandmother tomorrow.

IV/ Match the questions in column A with the answers in column B. (1,5pts)

A B Answers

17. What color are your eyes? A. I’d like noodles. 17 -

18. How much is a sandwich? B. I like soccer. 18 -

19. What would you like for breakfast? C. They are black. 19 -

20. What’s your favorite sport? D. It’s five thousand dong. 20 -

V/ Complete the passage with the words from the box. (1,5pts)Then answer the questions below (1,5pts)

favorite times plays or but every

Miss Huong is a teacher. She teaches us history. She is tall and thin (21) she is not weak. She (22) a lot

of sports. Her (23) sport is aerobics. She does aerobics three (24) a week. She likes jogging, too. She

jogs in the park near her house (25) morning. In her free time, she listens to music (26) watches

television.

Answer the questions (1,5pts)

27. What does Miss Huong do?

28. Which sports does she play?

29. Does she listen to music in her free time?

VI/ Complete the sentences, using the words given (1pt)

30. What/ Nam/ do/ when/ it/ hot?

31. There/ not/ any/ bottles/ cooking oil/ the kitchen.

VII/ Answer about you (1pt)32. What do you do in your free time?

33. How often do you help your mom?

34. What is your favorite food?

35. What are you going to do this summer vacation?

…. THE END….

Page 96: Tài liệu, học tập, trắc nghiệm, tiếng anh, văn bản, biểu ...s1.vndoc.com/.../06/40-de-thi-hoc-ky...6-co-dap-an.pdf · "/ 0. 1 2 %$ 3 ( 0 4 5 6 2 3 ( 5789 578: 2

VnDoc - Tải tài liệu, văn bản pháp luật, biểu mẫu miễn phí

ANSWER KEYS1. C

2. A

3. B

4. D

5. B

6. C

7. B

8. D

9. C

10. A

11. C

12. B

13. goes

14. is playing

15. to eat

16. is going to visit

17. c

18. d

19. a

20. b

21. but

22. plays

23. favorite

24. times

25. every

26. or

27. She is a teacher.

28. She does aerobics.

29. Yes, she does.

30. What does Nam do when it’s hot?

Page 97: Tài liệu, học tập, trắc nghiệm, tiếng anh, văn bản, biểu ...s1.vndoc.com/.../06/40-de-thi-hoc-ky...6-co-dap-an.pdf · "/ 0. 1 2 %$ 3 ( 0 4 5 6 2 3 ( 5789 578: 2

VnDoc - Tải tài liệu, văn bản pháp luật, biểu mẫu miễn phí

31. There aren’t any bottles of cooking oil in the kitchen.

32. I …

33. I often help my mom.

34. I like fish.

35. I am going to visit Hue.

Page 98: Tài liệu, học tập, trắc nghiệm, tiếng anh, văn bản, biểu ...s1.vndoc.com/.../06/40-de-thi-hoc-ky...6-co-dap-an.pdf · "/ 0. 1 2 %$ 3 ( 0 4 5 6 2 3 ( 5789 578: 2

VnDoc - Tải tài liệu, văn bản pháp luật, biểu mẫu miễn phí

PHÒNG GD & ĐT TÂN SƠNTRƯỜNG TH & THCS XUÂN SƠN

------------------------

ĐỀ KIỂM TRA HỌC KỲ IINĂM HỌC 2015 – 2016

MÔN: TIẾNG ANH 6

Full name:.........................................

Class: 6A

Điểm Lời Phê

I. Tìm từ không cùng nhóm với các từ còn lại bằng cách khoanh tròn vào chữ cái A, B, C

hoặc D: (2,0 điểm)1. A. ruler B. board C. play D. book

2. A. often B. play C. never D. sometimes

3. A. teacher B. finger C. leg D. hand

4. A. game B. music C. watch D. movie

II. Chọn đáp án đúng nhất (2,5 điểm)

1. ____ do you live ?

A. where B. what C. why D. what

2. I am ___ student at Xuan Son secondary

A. an B. a C. the D. at

3. at the moment, she_____ her homework

A. is B. do C. doing D. is doing

4. He _____ soccer every afternoon

A. play B. plays C. to play D. playing

5. Hoa______ to visit Hanoi, tomorrow

A. is B. going C. is going D. goes

6. You can___ your book on the table

A. put B. putting C. puts D. to put

7. They ____ game together now.

A. is playing B. are playing C. is D. are

Page 99: Tài liệu, học tập, trắc nghiệm, tiếng anh, văn bản, biểu ...s1.vndoc.com/.../06/40-de-thi-hoc-ky...6-co-dap-an.pdf · "/ 0. 1 2 %$ 3 ( 0 4 5 6 2 3 ( 5789 578: 2

VnDoc - Tải tài liệu, văn bản pháp luật, biểu mẫu miễn phí8. They are going ____ in a hotel

A. stay B. staying C. to stay D. stays

9. He usually_____ table tennis with his friends

A. plays B. play C. playing D. play

10. Keep silent! Teacher _____ students

A. is asking b. asks C. ask D. Asking

III. Tìm từ thích hợp trong khung điền vào các câu sau: ( 2 điểm)

about in by at

1. Mai goes to school………….bike every day.

2. I usually go home from school ………..five o’clock.

3. How………going to the zoo?

4. It's hot.......... summer.

1…………2…………….3…………….4………………

IV. cho dạng đúng của động từ ( 2 điểm)1. He___________________ (be going to) visit his friends, next week.

2. At the moment, they_____________________ (learn) English.

3. I often________________ (go) swimming in the summer.

4. Ba _______________________(play) soccer every afternoon.

V. Dịch sang tiếng Anh (1,5 điểm)1. Cô ấy thỉnh thoảng đi bơi vào mùa hè.

………………………………………………………………………..

2. Bạn đến từ đâu?

………………………………………………………………………

3. Mình đến từ nước Anh.

……………………………………………………………………..

Page 100: Tài liệu, học tập, trắc nghiệm, tiếng anh, văn bản, biểu ...s1.vndoc.com/.../06/40-de-thi-hoc-ky...6-co-dap-an.pdf · "/ 0. 1 2 %$ 3 ( 0 4 5 6 2 3 ( 5789 578: 2

VnDoc - Tải tài liệu, văn bản pháp luật, biểu mẫu miễn phí

1

A. LISTENING (2pts)Part 1. Listen and tick () the right box.

SỞ GIÁO DỤC VÀ ĐÀO TẠOTHÀNH PHỐ CẦN THƠ

---------------------

ĐỀ CHÍNH THỨC

ĐỀ KIỂM TRA HỌC KỲ IINĂM HỌC 2014-2015

MÔN: TIẾNG ANH - LỚP 6 (THÍ ĐIỂM)Thời gian làm bài: 60 phút, không kể thời gian phát đề

(Đề kiểm tra có 03 trang)

Page 101: Tài liệu, học tập, trắc nghiệm, tiếng anh, văn bản, biểu ...s1.vndoc.com/.../06/40-de-thi-hoc-ky...6-co-dap-an.pdf · "/ 0. 1 2 %$ 3 ( 0 4 5 6 2 3 ( 5789 578: 2

VnDoc - Tải tài liệu, văn bản pháp luật, biểu mẫu miễn phí

2

Part 2: You will hear a woman called Sarah talking to a group of people about her painting, and thenchoose the correct answer.

5. How often does Sarah paint now?

A. three days a week B. five days a week C. every evening

6. Sarah earns enough money from her painting to __________

A. give up her computing job

B. pay for her flat and car

C. pay for her artist’s materials

7. When she was at primary school, Sarah _________.

A. painted pictures of people

B. learnt to use chalk

C. drew scenes in pencil

8. What pleased Sarah most about her painting holidays?

A. meeting other artists

B. seeing beautiful scenery

C. receiving individual teaching

B. LEXICO-GRAMMAR: (2.0pts)

Choose A, B, C, or D to complete the following sentences.9.

A. killed B. listened C. perfected D. preferred

10. ________many channels are there on your TV?

A. Who B. How C. What D. Where

11. Our uncle goes ______two times a month.

A. fishing B. fish C. to fish D. to fishing

12. Lan: My class has won the aerobics contest.

Mai: ____________

A. Good luck! B. Congratulations! C. See you later. D. Goodbye

13. Home is __________place in the world.

A. the safest B. the most safest C. the safe D. the most safe

14. Have you ________seen a rainbow?

A. never B. before C. after D. ever

15. Exercising is good ________don’t do too much hard.

A. and B. but C. although D. so

Page 102: Tài liệu, học tập, trắc nghiệm, tiếng anh, văn bản, biểu ...s1.vndoc.com/.../06/40-de-thi-hoc-ky...6-co-dap-an.pdf · "/ 0. 1 2 %$ 3 ( 0 4 5 6 2 3 ( 5789 578: 2

VnDoc - Tải tài liệu, văn bản pháp luật, biểu mẫu miễn phí

3

16. To play badminton or tennis well, you need to have a good ________.

A. glove B. ball C. rod D. racket

C. READING (2.5pts)

Part 1. Read the passage and choose the correct word marked A, B or C to fill in each blank. (1.25pts)Cities look fantastic ___(17)___night. In the evening, the sun goes ___(18)___. The sky gets

___(19)___and darker. Slowly the bright colours go, ___(20)___. Everything looks grey. Then the lights

come on. The buildings look beautiful. Cities look ___(21)___at night.

17. A. at B. in C. on

18. A. up B. through C. down

19. A. brighter B. darker C. blacker

20. A. too B. to C. and

21. A. same B. different C. ugly

Part 2. Read the letter and mark the sentences as True (T) or False (F). (1,25pts)I always wish to have a motor home in my life. My motor home will not need to be big, but it might have

everything I need inside.

There will be a small sofa and a lovely table in the head of the car. This will be the place for my friends

when they visit me. It is like a living room. The biggest part of my motor home will be my kitchen. It will

be in the middle of the car. There will be an automatic dishwasher, a modern fridge, a small oven and an

automatic washing machine in the kitchen. A small toilet will be in the kitchen, too. The last part will be my

bedroom, also my entertainment place. There will be a small bed, a wireless TV, and a high-tech computer.

Everything might be small in my motor home. It would be fantastic if I had my own motor home. The best

thing about having a motor home is that I can drive it myself and travel around.

22. What kind of house does the writer wish to have?

A. A space houseboat B. An auto house C. A motor home

23. How many parts are there in the house?

A. 2 B. 3 C. 4

24. What is NOT true about this house?

A. It will not be very big.

B. It has everything that the writer needs inside.

C. The writer will have to do the laundry by hand.

25. Why does the writer love this kind of house?

A. Because the house is big

Page 103: Tài liệu, học tập, trắc nghiệm, tiếng anh, văn bản, biểu ...s1.vndoc.com/.../06/40-de-thi-hoc-ky...6-co-dap-an.pdf · "/ 0. 1 2 %$ 3 ( 0 4 5 6 2 3 ( 5789 578: 2

VnDoc - Tải tài liệu, văn bản pháp luật, biểu mẫu miễn phí

4

B. Because he can invite friends to his house.

C. Because he can travel around with his house.

26. What is the biggest part of the house?

A. The living room B. The kitchen C. The bedroom

PART D: WRITINGPart 1: Finish each sentence so that it means the same as the sentence before it. (1.25pts)

27. I have studied English for three years.

I began …..……………………………………………………………………………………….…...

28. Mike is not as interested in music as his sister is.

Mike’s sister……………………………………………………………………………………..….....

29. My sister can swim well. She can’t play badminton. (but)

My sister ………………………………………...…………………………………………………….

30. She loves French food. She goes to that French restaurant every week. (so)

………………………………………….………………………………………………………………

31. He was tired last night. He stayed up late to watch a football match.

Although……………………………………………………………………………………..................

Part 2. Arrange the words or phrase words to make meaningful sentences. (1.25pts)

32. wearing/ I/ am/ T-shirt/ gave/ you/ me/ birthday/ last

→ ....................................................................................................................................................................

33. me/ is/ in/ it/ competition/ running/ my school’s

→ ....................................................................................................................................................................

34. it/ took/ week/ sister/ my/ last

→ ....................................................................................................................................................................

35. sorry/ am/ I/ hear/ the/ about/ result/ to

→ ....................................................................................................................................................................

36. family/ was/ my/ happy/ fun/ we/ all/ and

→ ....................................................................................................................................................................

-----THE END-----

Page 104: Tài liệu, học tập, trắc nghiệm, tiếng anh, văn bản, biểu ...s1.vndoc.com/.../06/40-de-thi-hoc-ky...6-co-dap-an.pdf · "/ 0. 1 2 %$ 3 ( 0 4 5 6 2 3 ( 5789 578: 2

VnDoc - Tải tài liệu, văn bản pháp luật, biểu mẫu miễn phí

I. Find the word with the different underlined sound.1. A. come B. someone C. brother D. volleyball

2. A. tables B. matches C. watches D. houses3. A. engineer B. greeting C. teeth D. street

4. A. sandwich B. chair C. school D. chicken5. A. local B. flower C. poster D. host

II. Choose the correct option A,B,C,D to complete the sentences.6. There . . . . . a big sofa in the living room.

A. is B. are C. am D. does

7. One of the two girls, Mai is the . . . . .

A. tallest B. tall C. taller D. more tall

8. My mother often cooks . . . . . at Tet.

A. peach blossoms B. Chung cake C. spring rolls D. special food

9. We . . . . . our friends a happy new year.

A. give B. bring C. make D. wish

10. What kind of TV programmes do you like? - . . . . .

A. Yes,I do B. I like cartoons C. English D. sad stories

11. When he was young, he . . . . . swim very well. He won medals and championships.

A. can B. could C. was able D. will be able to

12. If we cut down more forest, there . . . . . more flood.

A. are B. were C. have been D. will be

13. The opposite of “dangerous” is . . . . .

A. polluted B. safe C. good D. dirty

14. Many people like Tom and Jerry . . . . . it is fun and entertaining.

A. and B. but C. or D. because

15. When . . . . . to San Francisco? - In 2010

A. have you gone B. did you go C. do you go D. will you go

16. In the future, robots will help us to surf the . . . . .

PHÒNG GD&ĐT THÁI THỤY KIỂM TRA CHẤT LƯỢNG CUỐI NĂM HỌC 2014-2015MÔN: TIẾNG ANH 6Chương trình Thí điểm

Thời gian làm bài: 45 phút

Page 105: Tài liệu, học tập, trắc nghiệm, tiếng anh, văn bản, biểu ...s1.vndoc.com/.../06/40-de-thi-hoc-ky...6-co-dap-an.pdf · "/ 0. 1 2 %$ 3 ( 0 4 5 6 2 3 ( 5789 578: 2

VnDoc - Tải tài liệu, văn bản pháp luật, biểu mẫu miễn phí

A. websites B. Internet C. channels D. TV

programmes

17. What does 3Rs stand . . . . . ?. It’s Reduce, Reuse, Recycle.

A. at B. for C. on D. by

18. I . . . . . that novel 3 times.

A. read B. is reading C. have read D. will read

19. Look! Nam is . . . . . judo in the playground.

A. dancing B. doing C. playing D. making

20. My brother often . . . . . uniform everyday.

A. wear B. is wearing C. wears D. wore

III. Read and choose the correct answers.

Tet is the most important (21) . . . . . in Vietnam. Vietnamese people eat very special

(22) . . . . . on these days. The most special food is Chung cakes. Most families (23) . . . . .

many Chung cakes. They boil Chung cakes for five or six (24) . . . . . Every family makes spring

rolls because this food is (25) . . . . . and easy to make. People also buy or make mut-jam. People

often(26) . . . . . it with tea.

21. A. festival B. time C. occasion D. vacation

22. A. drinks B. foods C. vegetable D. fruit

23. A. sell B. cook C. buy D. make

24. A. days B. weeks C. hours D. minutes

25. A. delicious B. expensive C. cheap D. terrible

26. A. drink B. cook C. serve D. make

IV. Rewrite the sentences so that it stays the same meaning.27. My school has an art club.

There is …………………………………………………………

28. The dog is in front of the microwave.

The microwave …………………………………………………

29. Nam likes table tennis best.

Nam’s favorite………………………………………………….

V. Rearrange the words to make meaningful sentences.

30. the /my/ father/ goes/ judo/ club/ every/ Sunday/ to.

Page 106: Tài liệu, học tập, trắc nghiệm, tiếng anh, văn bản, biểu ...s1.vndoc.com/.../06/40-de-thi-hoc-ky...6-co-dap-an.pdf · "/ 0. 1 2 %$ 3 ( 0 4 5 6 2 3 ( 5789 578: 2

VnDoc - Tải tài liệu, văn bản pháp luật, biểu mẫu miễn phí

31. DaNang/ city/ is /the/ in/ Central Vietnam/ largest.

32. people/in/ break/ things/ at / my/Tet/shouldn’t/ country.

VI. Listening

1. Listen and write.Name : Mary Smith

(33) How old: ……………..

(34) Hair color: ……………..

(35) Eye color: ……………..

(36) How many brothers: ……………..

2. Listen and fill in each gap with a right word you hear.

And here are some interesting TV (37) . . . . . for you. The musical Green, Green

Summer on (38) . . . . . 1 starts at eight o’clock. At the same time on Channel 2 is The Red

Spotted Squirrel. Home and Garden follows at eight twenty-five. Today you’ll learn how to make a

house for your dog. Channel (39) . . . . . offers you a touching film of friendship, The Lonely

Giraffe. The film starts at (40) . . . . .

- THE END -

Họ và tên thí sinh: ……………………………………………… Số BD: ……………

Page 107: Tài liệu, học tập, trắc nghiệm, tiếng anh, văn bản, biểu ...s1.vndoc.com/.../06/40-de-thi-hoc-ky...6-co-dap-an.pdf · "/ 0. 1 2 %$ 3 ( 0 4 5 6 2 3 ( 5789 578: 2

VnDoc - Tải tài liệu, văn bản pháp luật, biểu mẫu miễn phí

PHÒNG GD-ĐT THÁI THỤY HƯỚNG DẪN CHẤM BÀI KIỂM TRACUỐI NĂM HỌC 2014-2015

MÔN: TIẾNG ANH 6Chương trình Thí Điểm

Bài I ; II và III: gồm 26 câu trắc nghiệm = 6,5 điểm. Học sinh chọn đúng mỗi câu được 0,25 điểm.

Câu 1: D Câu 7: C Câu 13: B Câu 19: B Câu 25: A

Câu 2: A Câu 8: B Câu 14: D Câu 20: C Câu 26: C

Câu 3: A Câu 9: D Câu 15: B Câu 21: A

Câu 4: C Câu 10: B Câu 16: B Câu 22: B

Câu 5: B Câu11: B Câu 17: B Câu 23: D

Câu 6: A Câu12: D Câu 18: C Câu 24: C

Bài IV: gồm 3 câu = 0,75 điểm. Học sinh viết lại đúng mỗi câu được 0,25 điểm.

27.There is an art club in my school.

28.The microwave is behind the dog

29.Nam’s favourite sport is table tennis.

Bài V: gồm 3 câu = 0,75 điểm. Học sinh sắp xếp lại đúng mỗi câu được 0,25 điểm.

30. My father goes to the judo club every Sunday.

31. Da Nang is the largest city in Central Vietnam.

32. People shouldn’t break things at Tet in my country.

Bài VI: gồm 2 bài nghe = 2,0 điểm.

* Bài nghe 1: 1,0 điểm. Học sinh nghe và viết đúng mỗi câu được 0,25 điểm.

Name : Mary Smith

(33) How old: 9

(34) Hair colour blonde

(35) Eye colour blue

(36) How many brothers: 3

* Bài nghe 2: 1,0 điểm. Học sinh nghe và viết đúng mỗi từ được 0,25 điểm

Page 108: Tài liệu, học tập, trắc nghiệm, tiếng anh, văn bản, biểu ...s1.vndoc.com/.../06/40-de-thi-hoc-ky...6-co-dap-an.pdf · "/ 0. 1 2 %$ 3 ( 0 4 5 6 2 3 ( 5789 578: 2

VnDoc - Tải tài liệu, văn bản pháp luật, biểu mẫu miễn phí

(37) programmes

(38) channel

(39) 3

(40) eight thirty

Mỗi câu đúng đạt 0,25 điểm. Tổng số câu 40 câu x 0,25 điểm = 10,0 điểm

Làm tròn:

- 5,25 làm tròn thành 5,0

- 5,5 giữ nguyên

- 5,75 làm tròn thành 6,0

- The end -

Page 109: Tài liệu, học tập, trắc nghiệm, tiếng anh, văn bản, biểu ...s1.vndoc.com/.../06/40-de-thi-hoc-ky...6-co-dap-an.pdf · "/ 0. 1 2 %$ 3 ( 0 4 5 6 2 3 ( 5789 578: 2

VnDoc - Tải tài liệu, văn bản pháp luật, biểu mẫu miễn phí

TRƯỜNG THCS HIẾU THÀNH------------------

ĐỀ THI HỌC KỲ IINĂM HỌC 2015 - 2016

MÔN THI: TIẾNG ANH - LỚP 6Thời gian làm bài: 60 phút

I. LISTENING: (2.0pts)

Part 1: Listen carefully and choose the best answer. You will listen to the tape three times. (1,0pt)1. What does Miss Trinh do?

A. a doctor B. a singer C. a worker D. a teacher

2. What does she look like?

A. tall and thin B. tall and fat C. tall and weak D. tall and light

3. What color is her hair?

A.black B. brown C. yellow D. red

4. Is she young or old?

A. She is young B. She isn't old C. She is old D. A and B

Part 2: Listen carefully and write T (True) or F (False). You will listen to the tape three times.

(1,0pt)

Statements T F

1. First, he’s going to visit Hue.

2. He’s going to stay with his father.

3. Then he’s going to stay at a friend’s house in Nha Trang.

4. There he’s going to visit the citadel.

II. READING: (3.0pts)

1. Read the passage carefully. (1,5pts)Miss Minh is our teacher. She ..............(1) .......... literature. She is tall and thin but she is not weak.

She ..........(2)............... a lot of sports. Her favorite sports is.................(3)............ She does aerobics

three times a week. She likes jogging, too. She jogs in the park ............(4).......... her house every

morning. She listens to music ...........(5).............. watches TV in her free time.

Choose the best answer for each gap:1. A. learns B. studies C. teach D. teaches

Page 110: Tài liệu, học tập, trắc nghiệm, tiếng anh, văn bản, biểu ...s1.vndoc.com/.../06/40-de-thi-hoc-ky...6-co-dap-an.pdf · "/ 0. 1 2 %$ 3 ( 0 4 5 6 2 3 ( 5789 578: 2

VnDoc - Tải tài liệu, văn bản pháp luật, biểu mẫu miễn phí

2. A. likes B. listens C. plays D. has

3. A. aerobics B. basketball C. badminton D. volleyball

4. A. next B. to the left C. near D. to the right

5. A. or B. in C. on D. at

2. Read the passage carefully and then choose the best answer. (1,5pts)My aunt is a teacher. Her name is Huong. She lives is Vinh City. She is going on vacation

for ten days this summer. She is going to visit Hue. She is going to stay in a small hotel near Huong

river. She is going to see the citadel for two days. Then, she is going to have sunbath on beautiful sandy

beach in Nha Trang for two days. After that, She is going to visit Dalat for three days. Finally, she is

going to visit some friends in Ho Chi Minh City. She is going to stay there for one day. She is going to

fly home.

Questions:

1. How many cities is she going to visit this summer vacation?

A. three B. four C. five D. six

2. what place is she going to visit first?

A. Vinh city B. Hue C. Nha Trang D. Dalat

3. What is she going to do in Ho Chi Minh city?

A. visit some friends B. visit Hue C. see the citadel D. visit Dalat

4. How long is she going to stay in Nha Trang?

A. for ten days B. for two days C. for three days D. for one day

5. How is she going home?

A. by bus B. by train C. by plane D. by car

V. WRITING: (3,0pts)A. Complete the second sentence so that it has a similar meaning to the first: (1,0pt)

1. My hair is short and curly.

=> I have ___________________________________________________________________

2. Let’s go to Ha Long Bay this summer vacation.

=> What about ______________________________________________________________?

B. Write about Lan with the given words: (2,0pts)0. Lan/ always/ go to school/ every morning

1. She/usually / play tennis and badminton/ the spring

Page 111: Tài liệu, học tập, trắc nghiệm, tiếng anh, văn bản, biểu ...s1.vndoc.com/.../06/40-de-thi-hoc-ky...6-co-dap-an.pdf · "/ 0. 1 2 %$ 3 ( 0 4 5 6 2 3 ( 5789 578: 2

VnDoc - Tải tài liệu, văn bản pháp luật, biểu mẫu miễn phí

2. Her favorite sport/be/aerobics and she/do/aerobics for three times a week.

3. She/ sometimes/ help her mom/ her free time

4. She/ never/ go fishing/ the weekend.

Ex: Lan always goes to school every morning. She..................................................................

...................................................................................................................................................

....................................................................................................................................................

Page 112: Tài liệu, học tập, trắc nghiệm, tiếng anh, văn bản, biểu ...s1.vndoc.com/.../06/40-de-thi-hoc-ky...6-co-dap-an.pdf · "/ 0. 1 2 %$ 3 ( 0 4 5 6 2 3 ( 5789 578: 2

VnDoc - Tải tài liệu, văn bản pháp luật, biểu mẫu miễn phí

ANSWER KEYSI . LISTENING: (2.0pts)Part 1: 4 câu x 0,25điểm = 1 điểm

1. D 2. A 3. B 4. C

Tape-script 1:

Miss Trinh is a teacher. She teaches literature in a small secondary school in a village. She is tall and

thin. She has long black hair and an oval face. She .has black eyes, a small nose and full lips. She isn't

young and she is a good teacher.

Part 2: 4 câu x 0,25điểm = 1 điểm

1. T 2. F 3. T 4. F

Tape-script 2:

Ba is going on vacation this summer. First, he’s going to visit Hue for a week. He’s going to stay with

his uncle and aunt. Then he’s going to stay at a friend’s house in Nha Trang beach for 3 weeks. There

he’s going to visit the beach and swim. After the vacation he’s going to home by train.

II. READING (3.0pts)

A. 4 câu x 0,25điểm = 1 điểm1. A 2. D 3. B 4. C

B. 4 câu x 0,25điểm = 1 điểm1. A 2. C 3. D 4. B

III. WRITING: (3,0pts)A. 2 câu x 0,5điểm = 1 điểm

1. I have a short curly hair.

2. What about goingto Ha Long Bay this summer vacation.

B. 8 câu x 0,25điểm = 2điểmEx: Lan always goes to school every morning. She usually plays tennis and badminton in the spring.

Her favorite sport is aerobics and she does aerobics for three times a week. She sometimes helps her

mom in her free time. She never goes fishing on the weekend.

Hiếu Thành, ngày 28 tháng 03 năm 2016GVBM

Võ Thị Hồng Như Lý

Page 113: Tài liệu, học tập, trắc nghiệm, tiếng anh, văn bản, biểu ...s1.vndoc.com/.../06/40-de-thi-hoc-ky...6-co-dap-an.pdf · "/ 0. 1 2 %$ 3 ( 0 4 5 6 2 3 ( 5789 578: 2

VnDoc - Tải tài liệu, văn bản pháp luật, biểu mẫu miễn phí

TRƯỜNG THCS PHƯƠNG TRUNG--------------------------

Họ và tên:……………………..

Lớp: 6…………………………

ĐỀ THI HỌC KỲ IINĂM HỌC: 2015 –2016

MÔN THI: TIẾNG ANH - LỚP 6

Thời gian làm bài: 45 phút

I.Find thewordwhich theunderlinedpart is pronounceddifferently fromtheothers. (1.5ps)1. a. full b. summer c. sunny d. sunday

2. a. hot b. not c. come d. on3. a. my b. twenty c. thirty d. easy

4. a. about b. country c. south d. douth5. a. hope b. here c. hour d. House

6. a. read b. teacher c. eat d. great

II. Choose the correct answer. (2.5ps)1. Hoang likes __________ coffee in the morning.

A. drink B. drinking C. drinks D. to drink

2. Lan always __________ her homework?

A. does B. goes C. makes D. doing

3. How __________ oranges do you need?

A. often B. much C. long D. many

4. You should __________ water.

A. save B. saves C. saving D. to save

5. __________ is she going to stay? – For four days.

A. What B. How long C. Where D. When

6. He is traveling to Hanoi __________ bus.

A. in B. at C. from D. by

7. Which is the __________ river in Vietnam?

A. long B. longer C. longest D. the long

8. Nam is __________ TV now.A. watching B. watch C. watchs D. to watch

9.Why don’t we __________ Huong Pagoda?A. going B. to go C. go D. goes

10. Mai is __________ than Huong.

A. happy B. happier C. happiest D. more happy

III. Give the correct form of the verbs in brackets. (2ps)

Page 114: Tài liệu, học tập, trắc nghiệm, tiếng anh, văn bản, biểu ...s1.vndoc.com/.../06/40-de-thi-hoc-ky...6-co-dap-an.pdf · "/ 0. 1 2 %$ 3 ( 0 4 5 6 2 3 ( 5789 578: 2

VnDoc - Tải tài liệu, văn bản pháp luật, biểu mẫu miễn phí

1. They (play) ................................... in the yard now.

2. Lan’s brother (listen) ................................... to music every morning.

3. We shouldn’t (throw) ................................... the trash on our street.

4. The students (see) ................................... Ngoc Son Temple.

IV. Read the passage below then answer the questions. (2ps)Hung and Long are going on vacation this summer. First, they are going to visit Ha Long

Bay for 2 days. They are going to stay with their uncle and aunt there. Next, they are going to

visit their friends in Hanoi for 3 days. They are going to visit Hoan Kiem Lake and Ho Chi Minh

museum. Then they are going to visit Hue for 4 days. They are going to see the Citadel.

After that, they are going to stay at a friend's house in Nha Trang for 3 days. Finally, they

are going to visit Long's grandparents in Ho Chi Minh City for a week.

1. Are Hung and Long going on vacation this summer?

..…………...…………….……...…………….……...…………….…….....…

2. Where are they going to visit first?

..…………...…………….……...…………….……...…………….…….....…

3. What are they going to do in Hue?

..…………...…………….……...…………….……...…………….…….....…

4. How long are they going to stay in Ho Chi Minh City?

..…………...…………….……...…………….……...…………….…….....…

V. Complete the second sentences. (2ps)1. Let’s eat some rice.

-> What about ..…………...…………….……...…………….……...……

2. My school has 21 classrooms.

-> There ..…………...…………….……...…………….……...…………

3. Hoang is 1.75 meters tall. Minh is 1.65 meters tall.

-> Minh is ..…………...…………….……...…………….……...………

4. Jane goes to school on foot.

-> Jane ..…………...…………….……...…………….……...……………

___ The end ___

Page 115: Tài liệu, học tập, trắc nghiệm, tiếng anh, văn bản, biểu ...s1.vndoc.com/.../06/40-de-thi-hoc-ky...6-co-dap-an.pdf · "/ 0. 1 2 %$ 3 ( 0 4 5 6 2 3 ( 5789 578: 2

VnDoc - Tải tài liệu, văn bản pháp luật, biểu mẫu miễn phí

1

PHÒNG GD&ĐT CAM LỘ----------------

ĐỀ CHÍNH THỨC

ĐỀ THI HỌC KỲ IIMÔN TIẾNG ANH - LỚP 6Năm học: 2015 - 2016

(Thời gian 45 phút không kể thời gian phát đề)

A. LISTENING:I. Listen and tick the box. (0,75pts). There is one example.

Page 116: Tài liệu, học tập, trắc nghiệm, tiếng anh, văn bản, biểu ...s1.vndoc.com/.../06/40-de-thi-hoc-ky...6-co-dap-an.pdf · "/ 0. 1 2 %$ 3 ( 0 4 5 6 2 3 ( 5789 578: 2

VnDoc - Tải tài liệu, văn bản pháp luật, biểu mẫu miễn phí

2

II. Listen and write T (True) or F (False). (1,25 pts)

Statements T F1. Jenny thinks we might live with robots2. Henry thinks we might travel to the Moon3. Jenny thinks robots might not clean our houses4. Henry thinks we will stay on the Moon for a short time5. Jenny thinks might help us do the housework

B. USE OF ENGLISH:I. Choose a word which has the underlined part pronounced differently from the rest : (0.5pt)

1. A. bath B. truth C. breathe D. thin2. A. fast B. bag C. stand D. fact

II. Choose A, B, C, or D for each gap in the following sentences: (2.0 pts)1. …………..do you watch Television? Twice a day

A. When B. How often C. How2.We …………..breakfast an hour ago.

A. have B . had C. to have3. Have you ………….been to America ?

A. ever B. never C. for4. Our father goes …………….twice a month

A.. fish B. fishing C. to fish5. We ………….have solar energy in the future

A. would B. could C. will6 Air pollution can cause …………..

A. lung cancer B . blood cancer C. eye cancer7. Please don’t cut down the trees because ………….effects badly on the environment.

A. water pollution B. air pollution C. deforestation8. When your glass is broken, you can ………..it to make a new one .

A. use B. recycle C. reduce

III. Complete the sentence using the tenses (0.5 pt)1. Have you ever (be)………...to Singapore?2. It might ( rain )…………….tomorrow.

C. READING:I. Read the passage and tick true or false (T/ F). (2.0 pt)

MoscowThis building is in Moscow. Moscow is the biggest city in Russia. There are nine

different towers and they are painted different colors. What do you think it look like ? Lot ofgiant ice creams? An enormous cake? The colors are bright but this building is more than 450years old* Answer T/F?

1- .............. Moscow is the biggest city in Russia.

Page 117: Tài liệu, học tập, trắc nghiệm, tiếng anh, văn bản, biểu ...s1.vndoc.com/.../06/40-de-thi-hoc-ky...6-co-dap-an.pdf · "/ 0. 1 2 %$ 3 ( 0 4 5 6 2 3 ( 5789 578: 2

VnDoc - Tải tài liệu, văn bản pháp luật, biểu mẫu miễn phí

3

2- .............. There are nineteen different towers.3- .............. They are painted the same colors.4- .............. This building is more than 450 years old.

D. WRITING:Rearrange these sentences (1.5 pts)

1. If / save / use / we / less /gas /, we / energy/ will...................................................................................................................................2. In the year 2030, / for/ be able to do all / housework /us./ robots/will/the...................................................................................................................................3. went /with/ his /yesterday/ classmates/ Nam/ camping………………………………………………………………………………………

----- The end -----

Page 118: Tài liệu, học tập, trắc nghiệm, tiếng anh, văn bản, biểu ...s1.vndoc.com/.../06/40-de-thi-hoc-ky...6-co-dap-an.pdf · "/ 0. 1 2 %$ 3 ( 0 4 5 6 2 3 ( 5789 578: 2

VnDoc - Tải tài liệu, văn bản pháp luật, biểu mẫu miễn phí

4

HƯỚNG DẪN CHẤMĐỀ THI HỌC KỲ II – NĂM HỌC 2015-2016

MÔN: TIẾNG ANH 6A. LISTENING:I. Listen and tick the box. (0,75pts)

1A 2C 3BII. Listen and write T (True) or F (False) (1.25 pts)

Statements T F1. Jenny thinks we might live with robots T2. Henry thinks we might travel to the Moon T3. Jenny thinks robots might not clean our houses F4. Henry thinks we will stay on the Moon for a short time F5. Jenny thinks robots might help us do the housework T

B. USE OF ENGLISH :I. Choose a word which has the underlined part pronounced differently from the rest : (0.5points)

1. C. breathe 2. A. fast

II. Choose A, B, C, or D for each gap in the following sentences: (2.0 pts)1 2 3 4

B. How often B. had A. ever B. fishing

5 6 7 8C. will A. lung cancer C. deforestation B. recycle

III. Complete the sentence using the tenses (0.5 pt)1. been 2. rain

C. READINGI. Read the passage and tick True or false ( T/ F) (2.0 pt)

1-....... T....... Moscow is the biggest city in Russia2- ....... F...... There are nineteen different towers3-........ F...... They are painted the same colors4- ....... T ..... This building is more than 450 years old

D. WRITING:Rearrange these sentences (1.5pts)1. If we use less gas, we will save energy2. In the year 2030, robots will be able to do all the housework for us.3. Nam went camping with his classmates yesterday. Yesterday, Nam went camping with

his classmates./.

Page 119: Tài liệu, học tập, trắc nghiệm, tiếng anh, văn bản, biểu ...s1.vndoc.com/.../06/40-de-thi-hoc-ky...6-co-dap-an.pdf · "/ 0. 1 2 %$ 3 ( 0 4 5 6 2 3 ( 5789 578: 2

VnDoc - Tải tài liệu, văn bản pháp luật, biểu mẫu miễn phí

PHÒNG GD&ĐT HẢI LĂNG-------------

ĐỀ CHÍNH THỨCMã đề 601

ĐỀ KIỂM TRA HỌC KỲ IINĂM HỌC 2015-2016

Môn: Tiếng Anh 6

Thời gian làm bài: 45 phút

A. LISTENING

Listen and match the verbs in the column A to the words / phrases in column B (1pt).

A B

1. recognise

2. make

3. understand

4. lift

5. guard

a. coffee

b. what someone say

c. our faces

d. the house

e. heavy things

B. PHONETICS

Choose the word that has the underlined part pronounced differently from the others (1pt).6. A. hear B. fear C. dear D. wear

7. A. match B. machine C. champion D. chess

8. A. wish B. high C. price D. mind

9. A. continent B. photo C. popular D. coffee

C. VOCABULARY AND GRAMMARI. Choose the best answers among A, B, C or D (3pts).

10. Sports ……………an important part in our lives.

A. play B. do C. go D. make

11. My mother has ………..…… to Ho Chi Minh city since 1999.

A. go B. went C. to go D. gone

12. I think it’s a very nice town ………….. the weather is good and people are friendly.

A. because B. if C. so D. although

13. If you want to know which film is on tonight, check the TV ……………… .

A. programme B. schedule C. news D. information

14. The ………………….. drink in Vietnam is tea.

A. more popular B. more and more popular C. delicious D. most popular

15. “ ……………… is the longest river in the world?” – I think it’s the Nile River.

Page 120: Tài liệu, học tập, trắc nghiệm, tiếng anh, văn bản, biểu ...s1.vndoc.com/.../06/40-de-thi-hoc-ky...6-co-dap-an.pdf · "/ 0. 1 2 %$ 3 ( 0 4 5 6 2 3 ( 5789 578: 2

VnDoc - Tải tài liệu, văn bản pháp luật, biểu mẫu miễn phí

A. Which B. Where C. How D. Why

II. Supply correct forms of the verbs in brackets (2pt).16.Where is your mother? She (cook) ……………in the kitchen.

17. How often (he/go)………………………fishing?

18. We (visit)……………………Hue this summer vacation.

19. What about (play)…………………badminton?

D. READINGFill each blank in the following passage with a suitable word from the box (1pt).

Hi. My name is Hoa. I’m from Vietnam. I’m (20)…… My country is very beautiful. We have

lots of mountains. We have great (21)……..like Red and Me Kong Rivers. We have lots of lakes, too.

We have lots of rain, so the country is very green. We have (22)…..forests and we have lots of

beautiful beaches but we don’t have any (23)………....

20. A. Vietnam B. Vietnamese C. Chinese D. Japanese

21. A. river B. rivers C. park D. parks

22. A. big B. long C. tall D. high

23. A. fields B. mountains C. hills D. deserts

E. WRITING

Re-write the sentences so that their meaning doesn’t change (2 pts).24. Hanoi is colder than Ho Chi Minh City. Ho Chi Minh City ........................................

25. Her lips are full. She has .........................................................

THE END

Page 121: Tài liệu, học tập, trắc nghiệm, tiếng anh, văn bản, biểu ...s1.vndoc.com/.../06/40-de-thi-hoc-ky...6-co-dap-an.pdf · "/ 0. 1 2 %$ 3 ( 0 4 5 6 2 3 ( 5789 578: 2

VnDoc - Tải tài liệu, văn bản pháp luật, biểu mẫu miễn phí

A. LISTENING

Listen and match the verbs in the column A to the words / phrases in column B (1pt).

A B

1. recognise

2. make

3. understand

4. lift

5. guard

a. coffee

b. what someone say

c. our faces

d. the house

e. heavy things

B. VOCABULARY AND GRAMMAR

I. Choose the best answers among A, B, C or D (3pts).6. Sports ……………an important part in our lives.

A. play B. do C. go D. make

7. My mother has ………..…… to Ho Chi Minh city since 1999.

A. go B. went C. to go D. gone

8. I think it’s a very nice town ………….. the weather is good and people are friendly.

A. because B. if C. so D. although

9. If you want to know which film is on tonight, check the TV ……………… .

A. programme B. schedule C. news D. information

10. The ………………….. drink in Vietnam is tea.

A. more popular B. more and more popular C. delicious D. most popular

11. “ ……………… is the longest river in the world?” – I think it’s the Nile River.

A. Which B. Where C. How D. Why

II. Supply correct forms of the verbs in brackets (2pt).12.Where is your mother? She (cook) ……………in the kitchen.

13. How often (he/go)………………………fishing?

14. We (visit)……………………Hue this summer vacation.

PHÒNG GD&ĐT HẢI LĂNG-------------

ĐỀ CHÍNH THỨC

Mã đề 602

ĐỀ KIỂM TRA HỌC KỲ IINĂM HỌC 2015-2016Môn: Tiếng Anh 6

Thời gian làm bài: 45 phút

Page 122: Tài liệu, học tập, trắc nghiệm, tiếng anh, văn bản, biểu ...s1.vndoc.com/.../06/40-de-thi-hoc-ky...6-co-dap-an.pdf · "/ 0. 1 2 %$ 3 ( 0 4 5 6 2 3 ( 5789 578: 2

VnDoc - Tải tài liệu, văn bản pháp luật, biểu mẫu miễn phí

15. What about (play)…………………badminton?

C. PHONETICSChoose the word that has the underlined part pronounced differently from the others (1pt).

16. A. hear B. fear C. dear D. wear

17. A. match B. machine C. champion D. chess

18. A. wish B. high C. price D. mind

19. A. continent B. photo C. popular D. coffee

D. WRITING

Re-write the sentences so that their meaning doesn’t change (2 pts).20. Hanoi is colder than Ho Chi Minh City. Ho Chi Minh City ........................................

21. Her lips are full. She has .........................................................

E. READINGFill each blank in the following passage with a suitable word from the box (1pt).

Hi. My name is Hoa. I’m from Vietnam. I’m (22)…… My country is very beautiful. We have

lots of mountains. We have great (23)……..like Red and Me Kong Rivers. We have lots of lakes, too.

We have lots of rain, so the country is very green. We have (24)…..forests and we have lots of

beautiful beaches but we don’t have any (25)………....

22. A. Vietnam B. Vietnamese C. Chinese D. Japanese

23. A. river B. rivers C. park D. parks

24. A. big B. long C. tall D. high

25. A. fields B. mountains C. hills D. deserts

THE END

Page 123: Tài liệu, học tập, trắc nghiệm, tiếng anh, văn bản, biểu ...s1.vndoc.com/.../06/40-de-thi-hoc-ky...6-co-dap-an.pdf · "/ 0. 1 2 %$ 3 ( 0 4 5 6 2 3 ( 5789 578: 2

VnDoc - Tải tài liệu, văn bản pháp luật, biểu mẫu miễn phí

ANSWER KEYSMÃ ĐỀ 601A. MỖI CÂU ĐÚNG 0,2 ĐIỂM

1. C 2. A 3. B 4. E 5. D

B. MỖI CÂU ĐÚNG 0,25 ĐIỂM

6. D 7. B. 8. A. 9. B.

C. MỖI CÂU ĐÚNG 0,5 ĐIỂM

10. A. 11. D 12. A 13. B 14. D 15. A

16. She is cooking

17. How often does he go fishing?

18. We are visiting to / will visit Hue this summer vacation.

19. What about playing badminton?

D. READING: Mỗi câu đúng 0,25 điểm

20. B. Vietnamese

21. B. rivers

22. A. big

23. D. deserts

E. WRITING:Mỗi câu đúng 1 điểm24. Ho Chi Minh City is hotter than Hanoi.

25. She has full lips.

MÃ ĐỀ 602A. MỖI CÂU ĐÚNG 0,2 ĐIỂM

1. C 2. A 3. B 4. E 5. D

B. MỖI CÂU ĐÚNG 0,5 ĐIỂM

6. A. 7. D. 8. A. 9. B. 10. A. 11. D

12. She is cooking

13. How often does he go fishing?

14. We are visiting to / will visit Hue this summer vacation.

15. What about playing badminton?

C. MỖI CÂU ĐÚNG 0,25 ĐIỂM16. D. 17. B. 18. A. 19. B.

Page 124: Tài liệu, học tập, trắc nghiệm, tiếng anh, văn bản, biểu ...s1.vndoc.com/.../06/40-de-thi-hoc-ky...6-co-dap-an.pdf · "/ 0. 1 2 %$ 3 ( 0 4 5 6 2 3 ( 5789 578: 2

VnDoc - Tải tài liệu, văn bản pháp luật, biểu mẫu miễn phí

D. WRITING: MỖI CÂU ĐÚNG 1 ĐIỂM20. Ho Chi Minh City is hotter than Hanoi.

21. She has full lips.

E. READING:MỖI CÂU ĐÚNG 0,25 ĐIỂM

22. B. Vietnamese

23. B. rivers

24. A. big

25. D. deserts

Page 125: Tài liệu, học tập, trắc nghiệm, tiếng anh, văn bản, biểu ...s1.vndoc.com/.../06/40-de-thi-hoc-ky...6-co-dap-an.pdf · "/ 0. 1 2 %$ 3 ( 0 4 5 6 2 3 ( 5789 578: 2

VnDoc - Tải tài liệu, văn bản pháp luật, biểu mẫu miễn phí

SỞ GIÁO DỤC & ĐÀO TẠO ĐẮK LẮKTRƯỜNG THCS - THPT ĐÔNG DU

(Đề không chính thức)

KIỂM TRA HỌC KỲ II - NĂM HỌC 2015 - 2016MÔN: ANH VĂN - LỚP 6

Thời gian: 45 phút (Không kể thời gian giao đề)

Class: ............................................................................

Full name: ....................................................................

Mark:

ĐỀ 1:

I. Khoanh tròn vào một từ khác loại với các từ còn lại: (1 điểm)1. A. father B. sister C. mother D. nurse

2. A. bakery B. bookstore C. street D. toy store

3. A. tall B. thin C. short D. play

4. A. water B. fish C. meat D. beef

II. Khoanh tròn vào một đáp án đúng: (4 điểm)1. A weight ______ is strong and fat.

A. teacher B. worker C. lifter D. farmer

2. They go to school ______ Monday.

A. in B. on C.at D. of

3. How _________________ kilos of meat does Lan want ?

A. much B. many C. long D. often

4. She wants a ____________ of chocolates.

A. can B. tube C. box D. bottle

5. Do you want some sandwiches ? – No, I’m not ___________________ . Thanks.

A. hungry B. full C. thirsty D. good

6. _______________ your favorite food ? – I like beef.

A. What is B. What are C. What D. How

7. She isn’t weak. She is_______________

A. thin B. tall C. fat D. strong

8. I like hot _______________

A. season B. weather C. activity D. class

9. I often go swimming when it is _______________

A. cold B. hot C. hungry D. thirsty

Page 126: Tài liệu, học tập, trắc nghiệm, tiếng anh, văn bản, biểu ...s1.vndoc.com/.../06/40-de-thi-hoc-ky...6-co-dap-an.pdf · "/ 0. 1 2 %$ 3 ( 0 4 5 6 2 3 ( 5789 578: 2

VnDoc - Tải tài liệu, văn bản pháp luật, biểu mẫu miễn phí

10. _______________ does your brother go fishing? He goes fishing once a week.

A. How old B. How often C. How D.What

11. What are you doing? - _______________

A. You are reading B. I reading C. I am read D. I am reading

12. What about _______________ by bike?

A. travel B. to travel C. traveling D. travels

13. It’s a beautiful day today. Let’s _______________ for a walk.

A. go B. going C. to go D. to going

14. I _______________ Hue next month.

A. visit B. am going to visit C. to visit D. visiting

15. Nam often _______________ fishing on the weekend.

A. go B. goes C. is going D. going

16. Mount Everest is _______________ mountain in the world.

A. the high B. the higher C. highest D. the highest

III. Cho dạng đúng của từ trong ngoặc: (1 điểm)

1. Would you like ………………………….…….. to my house for dinner? (come)

2. My mother ………………………….….…….. her teeth every night. (brush)

3. My friends ………………………………….. for me at the corner now. (wait)

4. This is the …………………………………….. house in our neighborhood. (small)

IV. Viết câu theo gợi ý: (2 điểm)

1. A glass of lemon juice is 6,000 dong. (Đặt câu hỏi cho từ được gạch dưới)

→……………………………………………………………………………………….……

2. It / hot / summer / but / cold / winter. (Hoàn thành câu với những từ cho sẵn)

→……………………………………………………………………………………….……

3. They go shopping twice a week. (Đổi sang thể phủ định và nghi vấn)

→……………………………………………………………………………………..………

→………………………………………………………………………………………..……

4. Mai goes to school on foot. (Viết câu nghĩa tương đương)

→ Mai walks…………………………………………………………………………………

V. Đọc đoạn văn sau và làm các bài tập kèm theo bên dưới: (2 điểm)

Page 127: Tài liệu, học tập, trắc nghiệm, tiếng anh, văn bản, biểu ...s1.vndoc.com/.../06/40-de-thi-hoc-ky...6-co-dap-an.pdf · "/ 0. 1 2 %$ 3 ( 0 4 5 6 2 3 ( 5789 578: 2

VnDoc - Tải tài liệu, văn bản pháp luật, biểu mẫu miễn phí

In Vietnam, there are two terms in a school year. They are the first term and the second term. At the end

of each term, students usually have final exams. The school year starts in late August and ends in May. A

school day is about four hours long. Students go to school from Monday to Saturday. They never go to

school on Sundays. On this weekend, they often play some sports such as soccer, badminton, or volleyball.

Sometimes they help their parents with the housework.

A. True (T) or False (F).

1. _______ Students never help their parents at home.

2. _______ Students go to school from Monday to Sunday.

3. _______ They often play volleyball on the weekend.

4. _______ The school year starts in May.

B. Answer the questions.1. How many terms are there in a school year in Vietnam?

→………………………………………………………………………………………..……

2. How long is a school day?

→………………………………………………………………………………………..……

- THE END –

Page 128: Tài liệu, học tập, trắc nghiệm, tiếng anh, văn bản, biểu ...s1.vndoc.com/.../06/40-de-thi-hoc-ky...6-co-dap-an.pdf · "/ 0. 1 2 %$ 3 ( 0 4 5 6 2 3 ( 5789 578: 2

VnDoc - Tải tài liệu, văn bản pháp luật, biểu mẫu miễn phí

SỞ GIÁO DỤC & ĐÀO TẠO ĐẮK LẮKTRƯỜNG THCS - THPT ĐÔNG DU

(Đề không chính thức)

KIỂM TRA HỌC KỲ II - NĂM HỌC 2015 - 2016MÔN: ANH VĂN - LỚP 6

Thời gian: 45 phút ( Không kể thời gian giao đề)

Class: ............................................................................

Full name: ....................................................................

Mark:

ĐỀ 2:

I. Khoanh tròn vào một từ khác loại với các từ còn lại: (1 điểm)1. A. morning B. afternoon C. sometimes D. evening

2. A. post-office B. badminton C. hospital D. bookstore

3. A. street B. river C. sea D. lake

4. A. fall B. spring C. summer D. July

II. Khoanh tròn vào một đáp án đúng: (4 điểm)1. What _______________ your father doing now? He is playing badminton.

A. is B. are C. do D. does

2. _______________ the weather like in the winter?

A. How B. How often C. What’s D.What

3. He _______________ goes fishing in the winter. It’s too cold.

A. often B. always C. usually D. never

4. What about _______________ by bike?

A. travel B. to travel C. traveling D. travels

5. Why don’t _______________ to the park?

A. go B. going C. we go D. we going

6. _______________ place are Phuong and Mai going to visit first?

A. Where B. Which C. How D. How long

7. Mai has _______________ black hair.

A. tall B. long C. fat D. a long

8. She does her homework _______________ the evening.

A. on B. in C. at D. with

9. We _______________ to work everyday.

A. goes B. go C. going D. gos

Page 129: Tài liệu, học tập, trắc nghiệm, tiếng anh, văn bản, biểu ...s1.vndoc.com/.../06/40-de-thi-hoc-ky...6-co-dap-an.pdf · "/ 0. 1 2 %$ 3 ( 0 4 5 6 2 3 ( 5789 578: 2

VnDoc - Tải tài liệu, văn bản pháp luật, biểu mẫu miễn phí

10. How _________________ kilos of meat does Lan want?

A. much B. many C. long D. often

11. _______________ color is your hair ?

a. How B. Which C. What D. where

12. How _______________ oranges does she want ?

A. much B. many C. do D . does

13. She needs _______________ of beef.

A. 200 grams B . a tube C. a dozen D. a bottle of

14. Hung is a _______________ He’s strong and heavy.

A. gymnast B. weight-lifter C. driver D. doctor

15. The Mekong River is_______________ than The Red River.

A. longer B. taller C. shorter D. higher

16. My father isn’t fat, he is _______________.

A. very fat B. teacher C. thin D. old

III. Cho dạng đúng của từ trong ngoặc: (1 điểm)

1. Lana always ………………………….…….. her tests carefully. (do)

2. What about ………………………….…….. some coca cola? (have)

3. The boys ………………………….…….. soccer at the moment. (play)

4. Nam is………………………….…….. than Minh. (tall)

IV. Viết câu theo gợi ý: (2 điểm)

1. The weather is cool in autumn. (Đặt câu hỏi cho từ được gạch dưới)

→………………………………………………………………………………………..……

2. I have breakfast at half past seven. (Đổi sang thể phủ định và nghi vấn)

→………………………………………………………………………………………..……

→………………………………………………………………………………………..……

3. Mr. Smith / write a letter / now. (Hoàn thành câu với những từ cho sẵn)

→………………………………………………………………………………………..……

4. No one in my class is taller than Minh. (Viết câu nghĩa tương đương)

→ Minh is the …………………………………………………………………………..……

V. Đọc đoạn văn sau và làm các bài tập kèm theo bên dưới: (2 điểm)

Page 130: Tài liệu, học tập, trắc nghiệm, tiếng anh, văn bản, biểu ...s1.vndoc.com/.../06/40-de-thi-hoc-ky...6-co-dap-an.pdf · "/ 0. 1 2 %$ 3 ( 0 4 5 6 2 3 ( 5789 578: 2

VnDoc - Tải tài liệu, văn bản pháp luật, biểu mẫu miễn phí

Ba and his friends always have a picnic on the weekend. They are going to have a picnic near a lake

next Saturday. Nam is going to bring his camera. Lan is going to bring some food. Ba is going to bring some

drinks. First, they are going to have fried chicken, fried potatoes, apples and soda. After that, they are going

to walk for one or two hours. Finally, they are going to take some photos and go home.

A. True (T) or False (F).1. _______ Lan is going to bring a camera to take some photos.

2. _______ Next Saturday, Ba and his friends are going to have a picnic.

3. _______ They are going to have meat, vegetables, fruits and soda.

4. _______ They are going to swim there.

B. Answer the questions.

1. What is Ba going to bring?

→………………………………………………………………………………………..……

2. Where are they going to have a picnic?

→………………………………………………………………………………………..……

- THE END –

Page 131: Tài liệu, học tập, trắc nghiệm, tiếng anh, văn bản, biểu ...s1.vndoc.com/.../06/40-de-thi-hoc-ky...6-co-dap-an.pdf · "/ 0. 1 2 %$ 3 ( 0 4 5 6 2 3 ( 5789 578: 2

VnDoc - Tải tài liệu, văn bản pháp luật, biểu mẫu miễn phí

ANSWER KEYSĐỀ 1:

I. Khoanh tròn vào một từ khác loại với các từ còn lại: (1 điểm)

1. D. nurse 2. C. street 3. D. play 4. A. water

II. Khoanh tròn vào một đáp án đúng: ( 4 điểm )

1. C. lifter 2. B. on 3. B. many 4. C. box

5. A. hungry 6. A. What is 7. D. strong 8. B. weather

9. B. hot 10. B. How often 11. D. I am reading 12. C. traveling

13. A. go 14. B. am going to visit 15. B. goes 16. D. the highest

III. Cho dạng đúng của từ trong ngoặc: (1 điểm)

1. to come 2. is brushing 3. are waiting 4. Smallest

IV. Viết câu theo gợi ý: (2 điểm)1. How much is a glass of lemon juice?

2. It is hot in (the) summer, but it is cold in (the) winter.

3. They don’t go shopping twice a week.

Do they go shopping twice a week?

4. Mai walks to school.

V. Đọc đoạn văn sau và làm các bài tập kèm theo bên dưới: (2 điểm)

A. True (T) or False (F).1. F. 2. F 3. T 4. F

B. Answer the questions.1. There are two terms (in a school year in Vietnam).

2. A school day / It is about four hours long

- THE END –

Page 132: Tài liệu, học tập, trắc nghiệm, tiếng anh, văn bản, biểu ...s1.vndoc.com/.../06/40-de-thi-hoc-ky...6-co-dap-an.pdf · "/ 0. 1 2 %$ 3 ( 0 4 5 6 2 3 ( 5789 578: 2

VnDoc - Tải tài liệu, văn bản pháp luật, biểu mẫu miễn phí

ĐỀ 2:I. Khoanh tròn vào một từ khác loại với các từ còn lại: (1 điểm)

1. C. sometimes 2. B. badminton 3. A. street 4. D. July

II. Khoanh tròn vào một đáp án đúng: (4 điểm)1. A. is 2. C. What’s 3. D. never 4. C. traveling

5. C. we go 6. B. Which 7. B. long 8. B. in

9. B. go 10. B. many 11. C. What 12. B. many

13. A. 200 grams 14. B. weight-lifter 15. A. longer 16. C. thin

III. Cho dạng đúng của từ trong ngoặc: (1 điểm)1. does 2. having 3. are playing 4. Taller

IV. Viết câu theo gợi ý: (2 điểm)

1. What’s the weather like in autumn?

2. I don’t have breakfast at half past seven.

Do you have breakfast at half past seven?

3. Mr. Smith is writing a letter now.

4. Minh is the tallest student in my class.

V. Đọc đoạn văn sau và làm các bài tập kèm theo bên dưới: (2 điểm)Ba and his friends always have a picnic on the weekend. They are going to have a picnic near a lake

next Saturday. Nam is going to bring his camera. Lan is going to bring some food. Ba is going to bring some

drinks. First, they are going to have fried chicken, fried potatoes, apples and soda. After that, they are going

to walk for one or two hours. Finally, they are going to take some photos and go home.

A. True (T) or False (F).

1. F 2. T 3. F 4. F

B. Answer the questions.

1. He is going to bring his camera

2. They are going to have a picnic near a lake

- THE END –

Page 133: Tài liệu, học tập, trắc nghiệm, tiếng anh, văn bản, biểu ...s1.vndoc.com/.../06/40-de-thi-hoc-ky...6-co-dap-an.pdf · "/ 0. 1 2 %$ 3 ( 0 4 5 6 2 3 ( 5789 578: 2

VnDoc - Tải tài liệu, văn bản pháp luật, biểu mẫu miễn phí

MÃ ĐỀ 01A. PHONIC AND LANGUAGE FOCUS

Câu 1:Chọn từ có cách phát âm khác. (1,0 điểm)

1. A. watches B. brushes C. classes D. lives

2. A. has B. name C. family D. lamp

3. A. teacher B. eat C. ahead D. read

4. A. slow B. go C. nobody D. do

Câu 2: Khoanh tròn chữ cái A, B, C hoặc D để chọn câu trả lời đúng. (2,0 điểm)

1. What would you like ___________dinner?

A. with B. at C. for D. of

2. ___________do you do when it’s hot? - I go swimming.

A. Who B. What C. When D. Where

3. Why don’t we go to the zoo this Sunday morning?

A. You are well B. Because I don’t like animals

C. That’s a good idea D. Thanks a lot

4. How many___________do you want?

A. rice B. rices C. kilo of rice D. kilos of rice

5. Is there ___________milk in the fridge?

A. a B. an C. some D. any

6. I drink an orange juice every morning. It’s my___________drink.

A. good B. like C. favorite D. cold

7. Look! That boy ___________trash on the street.

A. leave B. leaves C. is leaving D. is going to leave.

8. I ___________ Ha Long Bay this summer vacation.

A. visit B. am going to visit C. to visit D. visiting

Câu 3: Chia dạng đúng của động từ trong ngoặc. (2,0 điểm)

TRƯỜNG THCS CẢNH HÓA-------------------------

Họ tên: …………………………….…..…

SBD: ………………………………….

ĐỀ KIỂM TRA HỌC KỲ IINĂM HỌC: 2015 -2016

MÔN: TIẾNG ANH - LỚP: 6Thời gian 45 phút (Không kể thời gian giao đề)

Page 134: Tài liệu, học tập, trắc nghiệm, tiếng anh, văn bản, biểu ...s1.vndoc.com/.../06/40-de-thi-hoc-ky...6-co-dap-an.pdf · "/ 0. 1 2 %$ 3 ( 0 4 5 6 2 3 ( 5789 578: 2

VnDoc - Tải tài liệu, văn bản pháp luật, biểu mẫu miễn phí

1. Nga (watch) ___________TV every morning.

2. They (play) ___________soccer at the moment.

3. We (go) ___________ to the cinema tonight.

4. Would you like ( come) ___________ to my house for dinner tonight?

B. READING

Câu 4: Đọc kĩ đoạn văn và trả lời câu hỏi ở bên dưới. (2,5 điểm)

My name is Huong. I live in the countryside in Vietnam. There are four seasons in my country:

spring, summer, fall and winter. In the spring, the weather is usually warm and beautiful. There are

many flowers in this season. I sometimes go camping. In the summer, it is usually hot and sunny. I like

going to the beach during summer vacation. Fall is a nice season. The weather is often cool. I often go

fishing with my father. In the winter, it is usually cold and cloudy. So after school, I like staying at

home to watch television.

1. Where does Huong live?

.............................................................................................................................................

2. How many seasons are there in Huong’s country? What are they?

............................................................................................................................................

3. What is the weather like in the spring?

.............................................................................................................................................

4. When does she often go fishing?

.............................................................................................................................................

5. What does she like doing in the winter?

.............................................................................................................................................

C. WRITING.Câu 5: sử dụng những gợi ý trong ngoặc để viết lại câu.(2,5 điểm)

1. My father usually drives to work. (Viết lại câu sao cho nghĩa không đổi)

→My father usually goes ………………………………………..…………………….

2. This house is small. (Viết lại câu sao cho nghĩa không đổi)

→ It is…………………………………………………………………………………..

3. Her favorite drink is milk. (Đặt câu với từ gạch chân)

………………………………………………………………………………………….

4. An orange is three thousand dong. (Đặt câu với cụm từ gạch chân)

Page 135: Tài liệu, học tập, trắc nghiệm, tiếng anh, văn bản, biểu ...s1.vndoc.com/.../06/40-de-thi-hoc-ky...6-co-dap-an.pdf · "/ 0. 1 2 %$ 3 ( 0 4 5 6 2 3 ( 5789 578: 2

VnDoc - Tải tài liệu, văn bản pháp luật, biểu mẫu miễn phí

…………………………………………………………………………………………..

5. Minh /a/ going to/ have /are/ near/ his/ lake/ friends/ a/ and/ picnic.

(Sắp xếp lại vị trí đúng của các từ để hoàn thành câu)

…………………………………………………………………………………………

------------------The end-----------------

Page 136: Tài liệu, học tập, trắc nghiệm, tiếng anh, văn bản, biểu ...s1.vndoc.com/.../06/40-de-thi-hoc-ky...6-co-dap-an.pdf · "/ 0. 1 2 %$ 3 ( 0 4 5 6 2 3 ( 5789 578: 2

VnDoc - Tải tài liệu, văn bản pháp luật, biểu mẫu miễn phí

TRƯỜNG THCS CẢNH HÓA---------------------------

Họ tên: …………………………….…..…

SBD: ………………………………….

ĐỀ KIỂM TRA HỌC KỲ IINĂM HỌC: 2015 -2016

MÔN: TIẾNG ANH - LỚP: 6Thời gian 45 phút (Không kể thời gian giao đề)

MÃ ĐỀ 02A. PHONIC AND LANGUAGE FOCUS

Câu 1:Chọn từ có cách phát âm khác. (1,0 điểm)

1. A. bottle B. job C. chocolate D. movie

2. A. clothes B. watches C. benches D. classes

3. A. amazing B. apartment C. address D. armchair

4. A. head B. teacher C. bread D. ahead

Câu 2: Khoanh tròn chữ cái A, B, C hoặc D để chọn câu trả lời đúng. (2,0 điểm)

1. We are going to have an English examination ___________April, 21st .

A. on B. to C. at D. in

2. ___________color is your hair ?

A. How B. Which C. What D. Where

3. What about having a picnic?

A. Yes, I do B. That’s a good idea C. Yes, Please D. No, I don’t

4. How many eggs do you need, Mom ? - ___________.

A. a half dozen B. half a dozen C. a dozen half D. dozen a half

5. There are ___________ milk in the fridge.

A. a B. an C. some D. any

6. I eat an apple every morning. It’s my___________fruit.

A. good B. like C. favorite D. cold

7. Listen! The girl ___________a song in her room.

A. sing B. sings C. is singing D. is going to sing

8. I ___________ Hue next month.

A. visit B. am going to visit C. to visit D. visiting

Câu 3: Chia dạng đúng của động từ trong ngoặc. (2,0 điểm)

1. He usually (jog) ___________ in the morning.

Page 137: Tài liệu, học tập, trắc nghiệm, tiếng anh, văn bản, biểu ...s1.vndoc.com/.../06/40-de-thi-hoc-ky...6-co-dap-an.pdf · "/ 0. 1 2 %$ 3 ( 0 4 5 6 2 3 ( 5789 578: 2

VnDoc - Tải tài liệu, văn bản pháp luật, biểu mẫu miễn phí

2. I (travel) ___________to Binh Thuan tomorrow.

3. They (wait) ___________ for me at the corner now.

4. Would you like (have) ___________ breakfast with eggs, children?

B. READINGCâu 4: Đọc kĩ đoạn văn và trả lời câu hỏi ở bên dưới. (2,5 điểm)

Hi. My name is Mike. I am living with my family in a small house in London. My uncle's farm

is not near my house because it is in the country and I must go there by car when I visit my uncle's

family. I can eat a lot of fruit there because there are many fruit trees in the farm and they produce a lot

of fruit every year. I often swim in a beautiful small river near the farm and I sometimes go fishing

with my uncle. I am going to stay there for a month this summer vacation because I like the farm and

the river there very much.

1. Where is Mike living with his family?

............................................................................................................................................

2. How does he go to his uncle's farm when he visits his uncle's family?

.............................................................................................................................................

3. What can he eat in the farm?

.............................................................................................................................................

4. What does he do when he stays in the farm?

.............................................................................................................................................

5. How long is he going to stay there this summer vacation?

..............................................................................................................................................

C. WRITING

Câu 5: sử dụng những gợi ý trong ngoặc để viết lại câu.(2,5 điểm)

1. My sister usually walks to work. (Viết lại câu sao cho nghĩa không đổi

→ She ………………………………………..…………………………………………….

2. This school is big. (Viết lại câu sao cho nghĩa không đổi

→ It is……………………………………………………………………………………….

3. His favorite food is meat. (Đặt câu với từ gạch chân)

………………………………………………………………………………………………

4. An apple is two thousand dong. (Đặt câu với cụm từ gạch chân)

………………………………………………………………………………………………

Page 138: Tài liệu, học tập, trắc nghiệm, tiếng anh, văn bản, biểu ...s1.vndoc.com/.../06/40-de-thi-hoc-ky...6-co-dap-an.pdf · "/ 0. 1 2 %$ 3 ( 0 4 5 6 2 3 ( 5789 578: 2

VnDoc - Tải tài liệu, văn bản pháp luật, biểu mẫu miễn phí

5. Nga / going to/ is/ Nha Trang/ to/ on/ vacation/ go/ summer/ this.

(Sắp xếp lại vị trí đúng của các từ để hoàn thành câu)

…………………………………………………………………………………………

------------------ The end -----------------

Page 139: Tài liệu, học tập, trắc nghiệm, tiếng anh, văn bản, biểu ...s1.vndoc.com/.../06/40-de-thi-hoc-ky...6-co-dap-an.pdf · "/ 0. 1 2 %$ 3 ( 0 4 5 6 2 3 ( 5789 578: 2

VnDoc - Tải tài liệu, văn bản pháp luật, biểu mẫu miễn phí

HƯỚNG DẪN CHẤM BÀI KIỂM TRA CHẤT LƯỢNG HỌC KÌ IINĂM HỌC: 2015 -2016

MÔN: TIẾNG ANH - LỚP: 6MÃ ĐỀ 01

CÂU NỘI DUNG ĐIỂM

A

5,0 điểm

Câu 11,0 điểm

1. D 2. D 3. C 4. DMỗi câu 0,25 điểm

Câu 22,0 điểm

1. C 2. B 3. C 4. D

5. D 6. C 7. C 8. BMỗi câu 0,25 điểm

Câu 32,0 điểm

1. watches 2. are playing

3. are going to go 4. to comeMỗi câu 0,5 điểm

B2,5 điểm

Câu 42,5 điểm

1. She lives in the countryside in Vietnam. 0,5 điểm

2. There are four seasons in Huong’s country.

They are: spring, summer, fall and winter.0,5 điểm

3. In the spring, the weather is usually warm and

beautiful.0,5 điểm

4. When the weather is cool She goes fishing

with her father.0,5 điểm

5. She likes staying at home to watch television. 0,5 điểm

C2,5 điểm

Câu 52,5 điểm

1. My farther usually goes to work by car. 0,5 điểm

2. It is a small house. 0,5 điểm

3. What is her favorite drink? 0,5 điểm

4. How much is an orange? 0,5 điểm

5. Minh and his friends are going to have a

picnic near a lake. 0,5 điểm

Page 140: Tài liệu, học tập, trắc nghiệm, tiếng anh, văn bản, biểu ...s1.vndoc.com/.../06/40-de-thi-hoc-ky...6-co-dap-an.pdf · "/ 0. 1 2 %$ 3 ( 0 4 5 6 2 3 ( 5789 578: 2

VnDoc - Tải tài liệu, văn bản pháp luật, biểu mẫu miễn phí

HƯỚNG DẪN CHẤM BÀI KIỂM TRA CHẤT LƯỢNG HỌC KÌ II

NĂM HỌC: 2015 -2016MÔN: TIẾNG ANH - LỚP: 6

MÃ ĐỀ 02

CÂU NỘI DUNG ĐIỂM

A5,0 điểm

Câu 11,0 điểm

1. D 2. A 3. D 4. BMỗi câu 0,25 điểm

Câu 22,0 điểm

2. A 2. C 3. B 4. B

5. C 6. C 7. C 8. BMỗi câu 0,25 điểm

Câu 3

2,0 điểm

1. jogs 2. am going to go

3. are waiting 4. to haveMỗi câu 0,5 điểm

B2,5 điểm

Câu 42,5 điểm

1. He lives with her family in a small house in

London.0,5 điểm

2. He goes there by car. 0,5 điểm

3. He can eat a lot of fruit there. 0,5 điểm

4. He often swims in a beautiful small river near

the farm and he sometimes goes fishing with his

uncle.

0,5 điểm

5. He is going to stay there for a month this

summer vacation.0,5 điểm

C2,5 điểm

Câu 52,5 điểm

1. She usually goes to work on foot. 0,5 điểm

2. It is a big school. 0,5 điểm

3. What is his favorite food? 0,5 điểm

4. How much is an apple? 0,5 điểm

5. Nga is going to go Nha Trang on this summer

vacation.0,5 điểm

Page 141: Tài liệu, học tập, trắc nghiệm, tiếng anh, văn bản, biểu ...s1.vndoc.com/.../06/40-de-thi-hoc-ky...6-co-dap-an.pdf · "/ 0. 1 2 %$ 3 ( 0 4 5 6 2 3 ( 5789 578: 2

VnDoc - Tải tài liệu, văn bản pháp luật, biểu mẫu miễn phí

………………………………………………………………………………………...........................

Lưu ý : Học sinh không được sử dụng các loại tài liệu,từ điển

Học sinh làm bài trực tiếp vào tờ đề thi

Điểm Giám thị 1 Giám thị 2 Mã đề Số phách

01

I. Choose the word that has the underlined part pronounced differently from the others(1m)

1. A. books B. pencils C. rulers D. bags

2. A. read B. teacher C. eat D. ahead

3. A. tenth B. math C. brother D. theater

4. A. has B. name C. family D. lamp

Answer:

1. 2. 3. 4.

II. Choose the best answer to complete these sentences.(2m)

1. Hung is a .................... He’s strong and heavy.

A. gymnast B. weight-lifter C. driver D. doctor

2. I like hot ....................

A. season B. weather C. activity D. class

3. What .................... your father doing now? He is playing badminton.

A. is B. are C. do D. does

4. .................... does your brother go fishing? He goes once a week.

PHÒNG GD&ĐT TP ĐÔNG HÀ----------------------

KỲ THI KSCL HỌC KỲ II NĂM HỌC 2015-2016Môn: Tiếng Anh - Lớp 6

Thời gian làm bài: 45 phútName : ……………………. ...

Class: 6....

Identification number :……. Số Phách:………

Page 142: Tài liệu, học tập, trắc nghiệm, tiếng anh, văn bản, biểu ...s1.vndoc.com/.../06/40-de-thi-hoc-ky...6-co-dap-an.pdf · "/ 0. 1 2 %$ 3 ( 0 4 5 6 2 3 ( 5789 578: 2

VnDoc - Tải tài liệu, văn bản pháp luật, biểu mẫu miễn phí

A. How old B. How often C. How D. What

5. He .................... goes fishing in the winter. It’s too cold.

A. often B. always C. usually D. never

6. What about .................... by bike?

A. travel B. to travel C. traveling D. travels

7. .................... place are Phuong and Mai going to visit first?

A. Where B. Which C. How D. How long

8. My house is near a market so it’s very __________.

A. big B. noisy C. quiet D. small

Answer:

1. 2. 3. 4. 5. 6. 7. 8.

III. Put the verbs into the correct form.(1,5m)

1. My school (have)……………………two floors.

2. Look! The plane (fly) ..............to the airport, it (land).......................

3.What about (visit)…………………………………..our parents?

4. We shouldn’t (throw)………trash on the street.

5. They (not go)………….swimming in the winter.

Answer:

1……………. 2……………. ……………. 3………….. 4…………….. 5…………….

IV. Make questions for the underlined parts or rearrange to complete these sentences(1m)

1. We usually play basketball on the weekends.___________________________________

2. They are going to visit Hue next summer. _______________________________________

3. tallest / Tuan / our / is / the / class/ in.__________________________________________

4. the / spring / in / What / weather / is / like / the?_______________________________________

V. Read the passage carefully then choose the correct words to fill in the blanks.(1,5m)

favorite does plays or but everyday

Page 143: Tài liệu, học tập, trắc nghiệm, tiếng anh, văn bản, biểu ...s1.vndoc.com/.../06/40-de-thi-hoc-ky...6-co-dap-an.pdf · "/ 0. 1 2 %$ 3 ( 0 4 5 6 2 3 ( 5789 578: 2

VnDoc - Tải tài liệu, văn bản pháp luật, biểu mẫu miễn phí

Miss Loan is our teacher. She teaches us English. She is tall and thin (1)………..she is not weak.

She (2)…………….a lot of sports. Her (3)…………...sport is aerobics. She (4)…………aerobics three

times a week. She likes jogging, too. She jogs in a park near her house (5)……………In her free time

she listens to music (6)…………..watches TV.

Answer:

1. 2. 3. 4. 5. 6.

VI. Write the second sentence so that it has a similar meaning to the first.(2m)

1. There are a lot of flowers in my garden.

=>My garden …………………………………………..

2. Ha Tinh is smaller than Nghe An.

=> Nghe An ………………………………………..

3.Let’s go to Ha Long Bay next summer vacation.

=>What about ………………………………………………..?

4. I want some milk.

=>I’d ………………………………………………………….

VII.Write a passage to describe your close friend.(1m)

------THE END------

Page 144: Tài liệu, học tập, trắc nghiệm, tiếng anh, văn bản, biểu ...s1.vndoc.com/.../06/40-de-thi-hoc-ky...6-co-dap-an.pdf · "/ 0. 1 2 %$ 3 ( 0 4 5 6 2 3 ( 5789 578: 2

VnDoc - Tải tài liệu, văn bản pháp luật, biểu mẫu miễn phí

………………………………………………………………………………………..........................

Lưu ý : Học sinh không được sử dụng các loại tài liệu,từ điển

Học sinh làm bài trực tiếp vào tờ đề thi

Điểm Giám thị 1 Giám thị 2 Mã đề Số phách

02

I. Choose the word that has the underlined part pronounced differently from the others(1m)

1. A. does B. watches C. finishes D. brushes

2. A. fine B. kind C. nine D. tin

3. A. slow B. go C. nobody D. do

4. A. amazing B. apartment C. address D. armchair

Answer:

1. 2. 3. 4.

II. Choose the best answer to complete these sentences.(2m)

1. What .................... your father doing now? He is playing badminton.

A. is B. are C. do D. does

2. He .................... goes fishing in the winter. It’s too cold.

A. often B. always C. usually D. never

3. Long is a .................... He’s strong and heavy.

A. gymnast B. weight-lifter C. driver D. doctor

4. .................... place are Phuong and Mai going to visit first?

PHÒNG GD&ĐT TP ĐÔNG HÀ----------------------

KỲ THI KSCL HỌC KỲ II NĂM HỌC 2015-2016Môn: Tiếng Anh - Lớp 6

Thời gian làm bài: 45 phútName : ……………………. ...

Class: 6....

Identification number :……. Số Phách:………

Page 145: Tài liệu, học tập, trắc nghiệm, tiếng anh, văn bản, biểu ...s1.vndoc.com/.../06/40-de-thi-hoc-ky...6-co-dap-an.pdf · "/ 0. 1 2 %$ 3 ( 0 4 5 6 2 3 ( 5789 578: 2

VnDoc - Tải tài liệu, văn bản pháp luật, biểu mẫu miễn phí

A. Where B. Which C. How D. How long

5. My house is near a market so it’s very __________.

A. big B. noisy C. quiet D. small

6. She likes hot ....................

A. season B. weather C. activity D. class

7. .................... does your brother go fishing? He goes once a week.

A. How old B. How often C. How D. What

8. What about .................... by bike?

A. travel B. to travel C. traveling D. travels

Answer:

1. 2. 3. 4. 5. 6. 7. 8.

III. Put the verbs into the correct form.(1,5m)

1. My family (have)……………………five people.

2. Look! The plane (fly) ..............to the airport, it (land).......................

3.How about (visit)…………………………………..our parents?

4. We shouldn’t (throw)………trash on the street.

5. Nam (not go)………….swimming in the winter.

Answer:

1……………. 2……………. ……………. 3………….. 4…………….. 5…………….

IV. Make questions for the underlined parts or rearrange to complete these sentences(1m)

1. My brother is traveling to school by car. _____________________________________

2. They are going to visit Hue next summer. _______________________________________

3. tallest / Tuan / our / is / the / class/ in.__________________________________________

4. the / spring / in / What / weather / is / like / the?_______________________________________

V. Read the passage carefully then choose the correct words to fill in the blanks.(1,5)

At , teacher , eleven , leaves , in , goes

Page 146: Tài liệu, học tập, trắc nghiệm, tiếng anh, văn bản, biểu ...s1.vndoc.com/.../06/40-de-thi-hoc-ky...6-co-dap-an.pdf · "/ 0. 1 2 %$ 3 ( 0 4 5 6 2 3 ( 5789 578: 2

VnDoc - Tải tài liệu, văn bản pháp luật, biểu mẫu miễn phí

This is my sister, Trang. She is a (1)…………She teaches at a small school(2)………..the

village. She gets up (3)…………..six. Every morning she (4)………..at the quarter to seven. The

school is not near her house, so she (5)…………to work by motorbike. Her class starts at seven and

ends at(6)……….My sister rides home and has lunch at quarter to twelve.

Answer:

1. 2. 3. 4. 5. 6.

VI. Write the second sentence so that it has a similar meaning to the first.(2m)

1. There are 400 students in my school.

=>My school …………………………………………..

2. Tuan is shorter than his brother.

=> Tuan’s brother ………………………………………..

3.What about going to Cua Tung beach?

=>Let’s………………………………………………..

4. I want some milk.

=>I’d ………………………………………………………….

VII.Write about your house in the future about 50-60 words.(1m)

------THE END------

Page 147: Tài liệu, học tập, trắc nghiệm, tiếng anh, văn bản, biểu ...s1.vndoc.com/.../06/40-de-thi-hoc-ky...6-co-dap-an.pdf · "/ 0. 1 2 %$ 3 ( 0 4 5 6 2 3 ( 5789 578: 2

VnDoc - Tải tài liệu, văn bản pháp luật, biểu mẫu miễn phí

PHÒNG GIÁO DỤC VÀ ĐÀO TẠOCHÂU THÀNH

----------------ĐỀ CHÍNH THỨC

ĐỀ KIỂM TRA HKII, NĂM HỌC 2015- 2016MÔN: TIẾNG ANH - LỚP 6

Thời gian: 60 phút (không kể phát đề)

Điểmbằng số

Điểm bằngchữ

Giám khảo 1 Lời phê STT Số tờ

Giám khảo 2 Số phách

Thí sinh làm bài trực tiếp vào đề thi. Đề thi gồm có 02 trang.

I. Listen: (2,0 pts)

A. Exercise 1: Listen to the passage then fill in the missing words. You will listen three times.There are four ……………(1) in a year. They are spring, summer, fall and winter. In the spring

the weather is usually …………..(2). Sometimes it is cold but not very cold. There are

many ……………….(3) in the spring. In the summer, the day is long and the night is short. People often

go swimming in this season. The fall is the season of fruits. In the winter, it is usually very cold. The day

is ………………..(4) and the night is long.

B. Exercise 2: Listen to the passage then answer True or False. You will listen three times.

T / F

1. He goes jogging with his mother …….. ………

2. He goes swimming in a small river. …….. ………

3. He is going to stay in a big hotel. …….. ………

4. He is going to buy souvenirs for his friends. …….. ………

II. Choose the best answer ( A, B, C or D): (2,0pts)Question 1. “….are you going to stay here ?”. “For two weeks”.

A. How often B. How far C. How long D. How much

Question 2. I’m going to visit my uncle…..Saturday morning.

A. in B. about C. at D. on

Page 148: Tài liệu, học tập, trắc nghiệm, tiếng anh, văn bản, biểu ...s1.vndoc.com/.../06/40-de-thi-hoc-ky...6-co-dap-an.pdf · "/ 0. 1 2 %$ 3 ( 0 4 5 6 2 3 ( 5789 578: 2

VnDoc - Tải tài liệu, văn bản pháp luật, biểu mẫu miễn phí

Question 3. What about…..a cup of coffee?

A. have B. you have C. having D. do you

Question 4. Let’s …the movie.

A. go B. go to C. going D. is going

Question 5. What do you do ….your free time?

A. at B. to C. on D. in

Question 6. ……..does she go jogging? Once a week.

A. How long B. How often C. How about D. How

Question 7. He …….to the movie.

A. goes often B. is often C. go often D. often goes

Question 8.What’s the weather ….in the summer? It’s hot.

A. like B. like s C. to like D. is like

III. Read: (2,0pts)

A. Read the text then. Choose A, B, C, or DIt’s 7.15 now. Hai, Lan and their parents are in the dining room. They are having breakfast. They

usually have breakfast at 7 o’clock. What are they going to do after breakfast?

Hai’s mother is going to take the dishes into the kitchen and wash them. Hai is going to clean the

table. Lan is going to sweep the floor. Hai’s father is going to have tea and read the newspaper. Then at 8

o’clock , Lan is going shopping with her mother . Hai is going to do his homework. His father is going to

help him. At 10 o’clock, they are going to watch TV in the living room.

1. Where is Hai’s family?

A. It is at home. B. It is breakfast time.

C. They are eating. D. They are in the dining room.

2. What time do they usually have breakfast?

A. 7 o’clock. B. 7.15 C. 6 o’clock. D. 6.30.

3. What is Hai’s mother going to do at 8 o’clock?

A. She is going to sweep the floor. B. She is going to go shopping with Lan.

C. She is going shopping with Hai. D. She is going to wash the dishes.

4. Where are they going to watch TV? In the ______ .

A. dining room B. kitchen C. bedroom D. living room

Page 149: Tài liệu, học tập, trắc nghiệm, tiếng anh, văn bản, biểu ...s1.vndoc.com/.../06/40-de-thi-hoc-ky...6-co-dap-an.pdf · "/ 0. 1 2 %$ 3 ( 0 4 5 6 2 3 ( 5789 578: 2

VnDoc - Tải tài liệu, văn bản pháp luật, biểu mẫu miễn phí

B. Read the text then answer these questions.Mr. Tan is a teacher. He is teaching in a secondary school in a town. He is tall and thin. He has an

oval face and black eyes. His lips are thin. He has short black hair and a small nose.

Mr. Tan has breakfast at seven. He has some noodles and soda for breakfast. He has lunch at home.

He has rice, chicken, cabbage and some apples. Apple is his favorite fruit.

1. Where is Mr. Tan working? ……………………………………………………………

2. What does he have for breakfast? ………………………………………………………

3. Does he have short black hair? …………………………………………………………

4. What kind of fruit does he like? ………………………………………………………..

IV. Write: (2,0pts)A. Choose the best answer ( A, B, C OR D)

Question 1. There / be / lot of / tree / park /.

A. There is a lot of trees in this park. B. There are a lot of trees in this park.

C. There be a lot of trees in this park.D. There are lot of trees in this park.

Question 2. What / you / do / tomorrow /?

A. What do you do tomorrow? B. What are you do tomorrow?

C. What are you going to do tomorrow? D. What do you doing tomorrow?

Question 3. They / often / have / lunch / 12.15 /.

A. They have often lunch at 12.15. B. They often have lunch at 12.15.

C . They have lunch at 12.15 often. D. They have lunch at often 12.15.

Question 4. She / usually / play / badminton / summer /.

A. She usually play badminton in the summer.

B. She usually plays badminton in the summer.

C. She’s usually playing badminton in the summer.

D. She’s usually plays badminton in the summer.

B. Complete these sentences.

1. The museum is to the left of my house.

My house ............................................................................................................................

2. He goes to school at six forty-five.

Page 150: Tài liệu, học tập, trắc nghiệm, tiếng anh, văn bản, biểu ...s1.vndoc.com/.../06/40-de-thi-hoc-ky...6-co-dap-an.pdf · "/ 0. 1 2 %$ 3 ( 0 4 5 6 2 3 ( 5789 578: 2

VnDoc - Tải tài liệu, văn bản pháp luật, biểu mẫu miễn phí

He goes to school at a ...........................................................................................................

3. Huong walks to school every morning .

Huong goes ...........................................................................................................................

4.There are many flowers in our garden .

Our garden ............................................................................................................................

V. Speaking: (2,0pts)- THE END -

Page 151: Tài liệu, học tập, trắc nghiệm, tiếng anh, văn bản, biểu ...s1.vndoc.com/.../06/40-de-thi-hoc-ky...6-co-dap-an.pdf · "/ 0. 1 2 %$ 3 ( 0 4 5 6 2 3 ( 5789 578: 2

VnDoc - Tải tài liệu, văn bản pháp luật, biểu mẫu miễn phí

ANSWER KEYSI. Listen: (2,0pts)A. Exercise 1: Listen then fill in the missing words. You will listen three times

(1) seasons (2) warm (3) flowers (4) short.

B. Exercise 2: Listen to the passage then write True or False. You will listen three times.

(5) F (6) T (7) F (8) T

II. Choose the best answer ( A, B, C OR D): (2,0pts)Question 1. C Question 2. D

Question 3. C Question 4. A

Question 5. D Question 6. B

Question 7. D Question 8. A

III. Read: (2,0pts)A. Read the text then . Choose A, B, C, or D

1. D 2. A 3. B 4. D

B. Read the text then answer these questions.5. Mr. Tan is working / teaching in a secondary school in a town.

6. He has some noodles and soda for breakfast.

7. Yes, he does.

8. He lkes apple.

IV. Write: (2,0pts)A. Choose the best answer ( A, B, C OR D)

Question 1. B Question 2. C

Question 3. B Question 4. B

B. Complete these sentences.1. My house is to the right of the museum.

2. He goes to school at a quarter to seven

3. Huong goes to school on foot.

Page 152: Tài liệu, học tập, trắc nghiệm, tiếng anh, văn bản, biểu ...s1.vndoc.com/.../06/40-de-thi-hoc-ky...6-co-dap-an.pdf · "/ 0. 1 2 %$ 3 ( 0 4 5 6 2 3 ( 5789 578: 2

VnDoc - Tải tài liệu, văn bản pháp luật, biểu mẫu miễn phí

4. Our garden has many flowers.

V. Speaking: (2,0pts)

TAPE TRANSCRIPTPart 1:

There are four seasons in a year. They are spring, summer, fall and winter. In the spring the

weather is usually warm. Sometimes it is cold but not very cold. There are many flowers in the spring. In

the summer, the day is long and the night is short. People often go swimming in this season. The fall is the

season of fruits. In the winter, it is usually very cold. The day is short and the night is long.

Part 2:Phong likes jogging very much. Every morning, he jogs with his father.When it is warm, Phong

goes swimming in a small river with his uncle. Phong travels a lot this summer. He is going to visit Ha

Long Bay with his family. He is going to stay in a small hotel for two weeks. He is going to buy some

souvenirs for his friends. He hopes the trip will be interesting.

SPEAKINGPart 1: Talk about yourselves (0,5 pt)

(Giáo viên hỏi học sinh 2 câu hỏi về bản thân)

Part 2: Topics (1,0 pt)(Học sinh bốc thăm một trong ba chủ đề, chuẩn bị trong 1 phút, sau đó trình bày ít nhất 4 ý)

Topic 1: Activities and seasonsTopic 2: Free time fun

Topic 3: Food and drinks

Part III: Interview (0.5 pt)Giáo viên hỏi HS thêm ít nhất 2 câu hỏi về chủ đề các em vừa trình bày.

* Some suggested questionsPart 1: Talk about yourselves (0,5 pt)(Học sinh tự nói ít nhất 4 câu về bản thân mình)

Page 153: Tài liệu, học tập, trắc nghiệm, tiếng anh, văn bản, biểu ...s1.vndoc.com/.../06/40-de-thi-hoc-ky...6-co-dap-an.pdf · "/ 0. 1 2 %$ 3 ( 0 4 5 6 2 3 ( 5789 578: 2

VnDoc - Tải tài liệu, văn bản pháp luật, biểu mẫu miễn phí

- Name / middle name / family name.

- Age.

- Job.

- Class / grade.

- School

- Home address.

Part 2: Topics (1,0 pt)(Học sinh bốc thăm một trong ba chủ đề, chuẩn bị trong 1 phút, sau đó trình bày ít nhất 4 ý)

Topic 1: Activities and seasons1. How many seasons are there in Vietnam?

2. What are they?

3. What is the weather like in the spring / summer / fall/ winter?

4. What weather do you like ?

5. When it is cold / warm / cool / hot, what do you do?

6. Do you like cold weather?

7. What do you often do in the spring?

8. What do you often do in the fall?

Topic 2: Free time fun

1. What do you do in your free time?

2. Do you like to sports?

3. What kinds of sports do you play ?

4. When do you sports?

5. How often do you play sports?

6. How many hour a day do you play sports?

7. Who do you often play sports with?

8. Are you good at playing sports?

9. …

Topic 3: Food and drinks1. How do you feel ?

2. Do you like cold drinks?

Page 154: Tài liệu, học tập, trắc nghiệm, tiếng anh, văn bản, biểu ...s1.vndoc.com/.../06/40-de-thi-hoc-ky...6-co-dap-an.pdf · "/ 0. 1 2 %$ 3 ( 0 4 5 6 2 3 ( 5789 578: 2

VnDoc - Tải tài liệu, văn bản pháp luật, biểu mẫu miễn phí

3. What kinds of drinks do you like ?

4. How often do you drink milk?

5. Is milk good for your health?

6. What kinds of food do you like ?

7. What do you often have for breakfast?

8. What do you often have for lunch ?

9. What do you often have for dinner?

10. What kinds of vegetables do you like ?

11. Give some vegetables you know?

Part III: Interview (0.5 pt)Giáo viên hỏi HS thêm ít nhất 2 câu hỏi về chủ đề các em vừa trình bày.

- THE END -

Page 155: Tài liệu, học tập, trắc nghiệm, tiếng anh, văn bản, biểu ...s1.vndoc.com/.../06/40-de-thi-hoc-ky...6-co-dap-an.pdf · "/ 0. 1 2 %$ 3 ( 0 4 5 6 2 3 ( 5789 578: 2

VnDoc - Tải tài liệu, văn bản pháp luật, biểu mẫu miễn phí

PHÒNG GIÁO DỤC VÀ ĐÀO TẠOQUẬN BÌNH THỦY

----------------TRƯỜNG THCS BÌNH THỦY

ĐỀ THI HỌC KỲ IINĂM HỌC 2014 - 2015

MÔN: ANH VĂN KHỐI 6Thời gian làm bài: 60 phút (không kể phát đề)

Listen to a passage and write True (T) or False (F) . You will hear it three times. (1.5ms)

1.__________ Miss Nga teaches English at a big school.

2.__________ She is busy every day.

3. __________ She leaves her house for school at 6.45.

4. __________The school is far from her house.

5.__________ She goes to work by motorbike.

6. __________She goes shopping in the afternoon.

PART II. LEXICO GRAMMAR:

Choose the most suitable answers to these questions by circling A ,B ,C or D (3.5 ms)7. Lan is very hungry. She wants some___________.

A.ice- cream B. coffee C. lemonade D. noodles

8. We are going ______________ a movie tonight.

A. go B. to go C. see D.to see

9. What’s the _________________like in the spring? – It’s warm.

A.weather B. summer C. season C. flower

10. She doesn’t like watching T.V. She_____________ watches T.V.

A.usually B. often C. always D. never

11. I have two arms and two ______________

A. heads B. eyes C. noses D. chests

12. What ______________ your parents going to do this summer?

A. is B. are C. do D. does

13. I’m Vietnamese. Vietnam is my______________ .

A. city B. name C. country D. village

14. What do you do_________________ your free time?

A. on B. at C. in D. of

15. _________________ does Mai skip?- four times a week.

A.How often B. How long C. How many D. How

16. I am going ___________________ Ha Long Bay.

Page 156: Tài liệu, học tập, trắc nghiệm, tiếng anh, văn bản, biểu ...s1.vndoc.com/.../06/40-de-thi-hoc-ky...6-co-dap-an.pdf · "/ 0. 1 2 %$ 3 ( 0 4 5 6 2 3 ( 5789 578: 2

VnDoc - Tải tài liệu, văn bản pháp luật, biểu mẫu miễn phí

A.visiting B. visit C. visited D.to visit

17. What are you going to do this weekend?

A. I’m going to do B. I am going to go camping

C. I’m going to swimming D. I’m going visit some of my friends

18. Choose the word which has the underlined part pronounced differently from the rest.

A. fly B. family C. country D. forty

19. They go often to the movies on the weekend.

A B C D

20. How much rice do you need? -__________________ , please.

A. A kilo half B. Half a kilo C. A half kilo D. Half of kilo

PART III. READING COMPREHENSION. (2,5ms)

I. Read the passage carefully. Check True ( T) or False ( F) statements.(1m)There are four seasons in a year. They are spring, fall, summer and winter. In the spring, the

weather is usually warm. Sometimes it is cold but not very cold. There are many flowers in the

spring. In the summer, the day is long and the night is short. People often go swimming in this

season. The fall is the season of fruits. In the winter, it’s usually very cold. The day is short and the

night is long.

* True / False21.__________ The day is long and the night is short in the winter.

22. __________ The weather is usually warm in the spring.

23. __________ The season of fruits is in the winter.

24. __________ People often swim in the summer.

II. Read the passage carefully. Then answer the questions (1,5ms)Hoang likes sports very much. Every morning he goes jogging with his father in the park near

his house. In the afternoon, he often plays soccer with his friends. Sometimes they play badminton.

When it is hot, he goes swimming in a small pool with his brother. Hoang also likes traveling a lot.

He often goes somewhere for his holidays. This summer, he is going to visit Hue with his family.

He is going to stay at his uncle’s house for two weeks.

25. How often does Hoang go jogging?

___________________________________________________________________

26. What does he often do in the afternoon?

___________________________________________________________________

Page 157: Tài liệu, học tập, trắc nghiệm, tiếng anh, văn bản, biểu ...s1.vndoc.com/.../06/40-de-thi-hoc-ky...6-co-dap-an.pdf · "/ 0. 1 2 %$ 3 ( 0 4 5 6 2 3 ( 5789 578: 2

VnDoc - Tải tài liệu, văn bản pháp luật, biểu mẫu miễn phí

27. Does he play badminton?

___________________________________________________________________

28. Who does he go swimming with?

__________________________________________________________________

29. What is he going to do this summer?

___________________________________________________________________

30. How long is he going to stay in Hue?

___________________________________________________________________

PART IV. WRITING: (2.5ms)

I. Make questions with the underlined words. (0.75m)31. I’m fromVietnam.

_________________________________________________________________?32. This ice-cream is five thousand dong.

_________________________________________________________________?

33.I’d like some noodles and some milk.

_________________________________________________________________?

II. Rewrite the sentences with the same meanings. (0.75m)34. How about going to the library?

Let’s_____________________________________________________________!

35. I ride my bike to school every day.

I _______________________________________________________________

36. She likes noodles.

Her favorite_______________________________________________________

III. Write the complete sentences from the given words. (1m)37. Lan / listen /music / everyday.

__________________________________________________________________

38. The children /play / park / the moment.

__________________________________________________________________

39.I / visit / Nha Trang / this / Summer vacation.

__________________________________________________________________

40. There / not / be / any / milk / bottle.

Page 158: Tài liệu, học tập, trắc nghiệm, tiếng anh, văn bản, biểu ...s1.vndoc.com/.../06/40-de-thi-hoc-ky...6-co-dap-an.pdf · "/ 0. 1 2 %$ 3 ( 0 4 5 6 2 3 ( 5789 578: 2

VnDoc - Tải tài liệu, văn bản pháp luật, biểu mẫu miễn phí

__________________________________________________________________

More; Verbs

1. What time ……………… you (go) ………………….. to school?

2. Lan (watch) ……………………….. T.V three times a week.

3. Nam & Ba often (go) ………………………. fishing on Sunday.

4. We (not go) ………………………… to school on Sunday.

5. My brother like jogging. He (jog) ……………………… every morning. At the moment, he

(jog)……………………….. in the park.

6. What is she doing ? -She (cook )………………………… lunch.

7. Where is Ba? He (be) ………………. in the yard. He (play)…………………………

badminton.

8. Hoa (not read)………………………….. book now, he (do)…………………….. his

homework.

9. They (do) …………………………………….. their homework tonight.

10. Nam (visit) ………………………………… Hue this summer vacation.

--The end--

Page 159: Tài liệu, học tập, trắc nghiệm, tiếng anh, văn bản, biểu ...s1.vndoc.com/.../06/40-de-thi-hoc-ky...6-co-dap-an.pdf · "/ 0. 1 2 %$ 3 ( 0 4 5 6 2 3 ( 5789 578: 2

VnDoc - Tải tài liệu, văn bản pháp luật, biểu mẫu miễn phí

ANSWER KEYSPART 1: LISTENINGThis is Miss Nga. She is a teacher. She teaches English at a small school in the village. She is

busy every day. She gets up at 6.00 and leaves her house for school at 6.45. The school is not very

far from her house, so she goes to work by bicycle. Her class starts at 7.00 and ends at 11.00. In the

afternoon, she goes shopping and cooks dinner for her family.

Listen to a passage and write True (T) or False (F) . You will hear it three times. (1.5ms)

1. F. 2. T 3. T 4. F 5. F 6. T

PART II. LEXICO GRAMMAR :Choose and circle the best word or phrase. (3.5ms)

7.D 8. B 9. A 10. D 11. A 12. B

13. C 14. C 15. A 16. D 17. B 18.A 19.A 20.B

PART III. READING COMPREHENSION. (2,5ms)

I. Read the passage carefully. Check True ( T) or False ( F) statements.(1m)21. F 22. T 23. F 24.T

II. Read the passage carefully. Then answer the questions (1,5ms)

25. How often does Hoang go jogging?

He always goes jogging.

26. What does he often do in the afternoon?

He often plays soccer.

27. Does he play badminton?

Yes, he does

28. Who does he go swimming with?

He goes swimming in a small pool with his brother.

29. What is he going to do this summer?

He is going to visit Hue with his family.30. How long is he going to stay in Hue?

He is going to stay for two weeks.

PART IV. WRITING: (2.5ms)

Page 160: Tài liệu, học tập, trắc nghiệm, tiếng anh, văn bản, biểu ...s1.vndoc.com/.../06/40-de-thi-hoc-ky...6-co-dap-an.pdf · "/ 0. 1 2 %$ 3 ( 0 4 5 6 2 3 ( 5789 578: 2

VnDoc - Tải tài liệu, văn bản pháp luật, biểu mẫu miễn phí

I. Make questions with the underlined words. (0.75m)31. I’m from Vietnam.

Where are you from?32. This ice-cream is five thousand dong.

How much is this ice – cream ?33. I’d like some noodles and some milk.

What would you like?

II. Rewrite the sentences with the same meanings. (0.75m)34. How about going to the library?

Let’s go to the library !35. I ride my bike to school every day.

I (always) go to school by bike.36. She likes noodles.

Her favorite food is noodles.

III. Write the complete sentences from the given words. (1m)37. Lan / listen /music / everyday.

Lan listen to music everyday.38. The children /play / park / the moment.

. The children are playing in the park at the moment.39.I / visit / Nha Trang / this / Summer vacation.

. I am going to visit Nha Trang this Summer vacation.40. There / not / be / any / milk / bottle.

There isn’t ( is not) any milk in the bottle.

Page 161: Tài liệu, học tập, trắc nghiệm, tiếng anh, văn bản, biểu ...s1.vndoc.com/.../06/40-de-thi-hoc-ky...6-co-dap-an.pdf · "/ 0. 1 2 %$ 3 ( 0 4 5 6 2 3 ( 5789 578: 2

VnDoc - Tải tài liệu, văn bản pháp luật, biểu mẫu miễn phí

PHÒNG GD&ĐT THÁI THỤY-----------------------

ĐỀ KIỂM TRA HỌC KỲ IINĂM HỌC 2015 - 2016

Môn: Tiếng Anh 6 - Chương trình Thí điểmThời gian làm bài: 45 phút

I. Listen carefully and choose the best answer (A, B or C) to complete each sentence.

1. Our class begins at . . . . . . A. 4:05 B. 4:15 C. 4:50

2. My mother left this morning at . . . . . . A. 9:03 B. 9:13 C. 9:30

3. I'm going to catch my bus at . . . . . . A. 3:40 B. 4:20 C. 12:04

4. Let's get together at . . . . . . A. 12:05 B. 5:12 C. 5:22

5. The store closes today at . . . . . . A. 4:06 B. 5:45 C. 6:15

6. The movie starts at . . . . . . A. 7:04 B. 7:14 C. 7:40

7. She called this morning at . . . . . . A. 10:00 B. 10:05 C. 10:10

8. Their plane arrives at . . . . . . A. 11:05 B. 4:12 C. 11:45

9. Please come to my office at . . . . . . A. 8:05 B. 8:15 C. 8:25

10. I have to work at . . . . . . A. 3:03 B. 3:13 C. 3:30

II. Find the word with the different underlined sound.

11. A. come B. someone C. son D. now

12. A. classes B. misses C. glasses D. houses

13. A. looked B. watched C. washed D. needed

III. Choose the correct option A, B, C or D to complete the sentences.14. There . . . . . a big sofa and four in the living room.

A. is B. are C. am D. does

15. One of the two girls, Mai is the . . . . .

A. tallest B. tall C. taller D. more tall

16. My mother often cooks . . . . . at Tet.

A. peach blossoms B. Chung cake C. spring rolls D. special food

17. I . . . . . to Singapore three times.

A. have gone B. have been C. went D. visited

18. My brother can’t swim . . . . . he’s afraid of water.

A. because B. but C. and D. so

19. When he was young, he . . . . . swim very well. He won medals and championships.

Page 162: Tài liệu, học tập, trắc nghiệm, tiếng anh, văn bản, biểu ...s1.vndoc.com/.../06/40-de-thi-hoc-ky...6-co-dap-an.pdf · "/ 0. 1 2 %$ 3 ( 0 4 5 6 2 3 ( 5789 578: 2

VnDoc - Tải tài liệu, văn bản pháp luật, biểu mẫu miễn phí

A. can B. could C. was able D. will be able to

20. - A: “. . . . . is your favourite tennis player?” - B: I don’t like tennis.

A. What B. Which C. Who D. Where

21. - A: “I’ve just received a scholarship.” - B: . . . . .

A. Really B. Thank you C. Congratulations D. Well

22. That is . . . . . book I’ve ever read.

A. the borest B. the boring C. the most bored D. the most boring

23. It’s getting light. Shall I turn . . . . . the lights to save electricity?

A. off B. up C. on D. in

24. In the future, robots will help us to surf the . . . . .

A. websites B. Internet C. channels D. TV programmes

25. What does 3Rs stand . . . . . ?. It’s Reduce, Reuse, Recycle.

A. at B. for C. on D. by

26. A robot can do . . . . . different things from looking after a baby to building a house.

A. much B. many C. few D. little

27. Look! Nam is . . . . . judo in the playground.

A. dancing B. doing C. playing D. making

28. I can’t come right now. I . . . . . when I finish my homework.

A. are coming B. come C. will come D. came

29. If you read more, your vocabulary will get . . . . . every day.

A. good B. well C. better D. best

30. It is wrong of you to do this. This . . . . . never happen again!

A. does B. would C. will D. must

IV. Read the following passage and put a word in each of the numbered blanks.Fansipan is the highest pick of Vietnam, Laos and Cambodia, so it is called the “Roof of

Indochina” while the local (31) . . . . . call it Huasipan. Huasipan means large tottering rock.

Fansipan is 3,143 metres (32) . . . . .. It is in Lao Cai province, nine kilometres southwest of Sa Pa

Town (33) . . . . . the Hoang Lien Son mountain range.

Fansipan is now one of the very few ecotourist spots of Vietnam, with about 2,024 varieties of

plants and 327 types of animal and insect. Many tourists (34) . . . . . the mountain every year. If you

climb the mountain from the end of February to Match, you (35) . . . . . have a chance to admire the

beautiful blooming flowers in many different colours.

Page 163: Tài liệu, học tập, trắc nghiệm, tiếng anh, văn bản, biểu ...s1.vndoc.com/.../06/40-de-thi-hoc-ky...6-co-dap-an.pdf · "/ 0. 1 2 %$ 3 ( 0 4 5 6 2 3 ( 5789 578: 2

VnDoc - Tải tài liệu, văn bản pháp luật, biểu mẫu miễn phí

V. Rewrite the sentences so that it stays the same meaning.36. It’s not good to stay up so late to watch TV.

You should …………………………………………………………………………….

37. Nobody is older than my grandmother in my family.

My grandmother …………………………………………………………………….…

38. Chau is the best at English in her class.

Nobody in Chau’s class……………………………..………………………………….

VI. Rearrange the words to make correct sentences.39. time/ We’re/ great/ here/having/ a/ in Cambodia!/

………………………………………………………………………………………….

40. The/ lovely./ weather/ been/ has/

………………………………………………………………………………………….

- THE END -

Họ và tên thí sinh: ……………………………………………… Số BD: ……………

Page 164: Tài liệu, học tập, trắc nghiệm, tiếng anh, văn bản, biểu ...s1.vndoc.com/.../06/40-de-thi-hoc-ky...6-co-dap-an.pdf · "/ 0. 1 2 %$ 3 ( 0 4 5 6 2 3 ( 5789 578: 2

VnDoc - Tải tài liệu, văn bản pháp luật, biểu mẫu miễn phí

PHÒNG GD&ĐT TAM ĐẢO---------------

Đề này gồm 02 trang

ĐỀ KIỂM TRA HỌC KỲ II NĂM HỌC 2015-2016PHẦN LISTENING, VOCABULARY, GRAMMAR, READING,

WRITING

MÔN: TIẾNG ANH - LỚP 6Thời gian: 45 phút (không kể thời gian giao đề)

A. LISTENING

Question 1-4. Listen to the dialogue and decide if the statements are true or alse. Put a tick in theboxes. (1.0/8)

Statements True False

1. There is some meat and some noodles for lunch.

2. There are some oranges and some bananas.

3. There is some water.

4. There isn’t any milk.

Question 5-6. Listen again and then answer the questions. (1.0/8)5. Do they have any noodles for lunch?

=>……………………………………………………………………...

6. What do they drink?

=>……………………………………………………………………...

B. VOCABULARY AND GRAMMAR:I. Choose the word whose underlined part is pronounced differently from the rest. (1.0/8)

7. A. kite B. twice C. pastime D. swim

8. A. movie B. coffee C. jogging D. soccer

9. A. boots B. floor C. toothpaste D. food

10. A. often B. once C. sometimes D. come

II. Choose the best option to complete the sentences. (1.5/8)11. Let's ................. camping.

A. go B. going C. to go D. goes

12. What ................. going by bike?

A. about B. for C. on D. of

13. What ………………. he like?- He’d like a cake.

Page 165: Tài liệu, học tập, trắc nghiệm, tiếng anh, văn bản, biểu ...s1.vndoc.com/.../06/40-de-thi-hoc-ky...6-co-dap-an.pdf · "/ 0. 1 2 %$ 3 ( 0 4 5 6 2 3 ( 5789 578: 2

VnDoc - Tải tài liệu, văn bản pháp luật, biểu mẫu miễn phí

A. do B. does C. would D. is

14. We want ............. to school by bike.

A. to go B. going C. goes D. go

15. ................. are we going to travel? --> By minibus.

A. Where B. When C. Why D. How

16. The Mekong River is ………….. than the Red river.

A. long B. longer C. longest D. longger

III. Choose the underlined part that needs correcting in each sentence. (1.0/8)

17. What do you do when it hot?

A B C D

18. Nga likes the fall because she likes weather cool.

A B C D

19. We go to school seven day a week.

A B C D

20. Why the weather like in the spring? - It is warm.

A B C D

C. READING

Choose the best options to complete the text. (1.0/8)Miss. Lan …(21)… doing aerobics in her free time. She goes to the gym …(22)… a week, on

Monday and Friday. She always goes there …(23)… bus. She never cycles because she doesn’t have a

bike. She usually feels hot and thirsty after doing aerobics, so she always takes a bottle of …(24)… with

her when he goes to the gym.

21. A. to like B. like C. likes D. is liking

22. A. once B. twice C. three times D. four times

23. A. by B. with C. for D. on

24. cooking oil B. rice C. chocolate D. water

D. WRITING

I. Put the words in the correct order to make meaningful sentences. (0.5/8)25. mother / my / tall / is / and / thin.

…………………………………………………………………………………..

26. has / Hoa / oval / an / face.

Page 166: Tài liệu, học tập, trắc nghiệm, tiếng anh, văn bản, biểu ...s1.vndoc.com/.../06/40-de-thi-hoc-ky...6-co-dap-an.pdf · "/ 0. 1 2 %$ 3 ( 0 4 5 6 2 3 ( 5789 578: 2

VnDoc - Tải tài liệu, văn bản pháp luật, biểu mẫu miễn phí

………………………………………………………………….………………..

II. Use the cues to make sentences. (1.0/10)

27. Huy/often/play/soccer/free/time.

……………………………………………………

28. The girls/skip/moment

……………………………………………………

- The end –

Page 167: Tài liệu, học tập, trắc nghiệm, tiếng anh, văn bản, biểu ...s1.vndoc.com/.../06/40-de-thi-hoc-ky...6-co-dap-an.pdf · "/ 0. 1 2 %$ 3 ( 0 4 5 6 2 3 ( 5789 578: 2

VnDoc - Tải tài liệu, văn bản pháp luật, biểu mẫu miễn phí

PHÒNG GIÁO DỤC VÀ ĐÀO TẠOCHÂU THÀNH

-------------------------ĐỀ CHÍNH THỨC

ĐỀ KIỂM TRA HKII, NĂM HỌC 2014 - 2015MÔN: TIẾNG ANH - LỚP 6

Thời gian: 45 phút (không kể phát đề)

Điểmbằng số

Điểm bằngchữ

Giám khảo 1 Lời phê STT Số tờ

Giám khảo 2 Số phách

I. Listening: (2,0pts) (Học sinh nghe và làm bài phần nghe khoảng 10 phút)

Part 1: Listen and complete the passage. (Nghe và hoàn thành đoạn văn)Miss Chi is tall and thin. She has a / an (1) __________ face. She has long (2) __________ hair. She has

brown (3) __________. She has a small nose. She has full lips and small white (4) __________.

Part 2: Listen to the dialogue and check True (T) or False (F). (Nghe ghi True - Đúng / False - Sai)

T/F

1. Lan is hot and thirsty.

2. Lan would like some oranges.

3. Nam feels hungry and he wants some noodles.

4. Ba is tired and he’d like a drink.

II. Reading: (2,0pts)

Part 1: Read the text then choose the right answer for each sentence.Hi! My name’s Thu. I’m a student. My sister and I eat our breakfast at school. She eats some

noodles and a hot drink. I have some bread and some milk. We don’t have lunch at school. We have lunch

at home. We have rice, chicken, cabbage and some apples for lunch. Apple is my favorite fruit.

1. Thu and her sister have breakfast _____________.

A. at home B. at school C. at the restaurant D. at the food store

2. They _____________ at home.

A. have breakfast B. have lunch C. not have breakfast D. don’t have lunch

Page 168: Tài liệu, học tập, trắc nghiệm, tiếng anh, văn bản, biểu ...s1.vndoc.com/.../06/40-de-thi-hoc-ky...6-co-dap-an.pdf · "/ 0. 1 2 %$ 3 ( 0 4 5 6 2 3 ( 5789 578: 2

VnDoc - Tải tài liệu, văn bản pháp luật, biểu mẫu miễn phí

3. For lunch, they have _____________.

A. rice, chicken, cabbage and some oranges B. rice, chicken, lettuce and some apples

C. rice, fish, cabbage and some apples D. rice, meat, vegetables and some fruit

4. Her favorite fruit is _____________.

A. banana B. orange C. apple D. pineapple

Part 2: Read the text then answer the questions: (1pt)Minh and Quang are good friends. They are in grade 6. They study at Quang Trung Secondary

School. They often play soccer in their free time. In the spring, they often play table tennis. They usually

play badminton in the fall. In the winter, they sometimes go fishing. They always go swimming in the

summer. In the summer, the weather is very hot but they like it.

1. Which school do Minh and Quang go to?

_____________________________________________________________________

2. What do they do in their free time?

_____________________________________________________________________

3. How often do they play badminton in the fall?

_____________________________________________________________________

4. Do they like summer?

_____________________________________________________________________

III. Writing: (2,0pts)Part 1: Rewrite the sentences, beginning with the words given. (1pt) (Viết lại các câu sau bắt đầu

bằng những từ đã cho.)1.What about taking some photos?

Why _________________________________________________________________

2. His hair is not long. (Using opposite word)

It is__________________________________________________________________

3. She goes to the movies once a week. (Make question)

How often_____________________________________________________________

4. They like cool weather. (Make question)

What ________________________________________________________________

Page 169: Tài liệu, học tập, trắc nghiệm, tiếng anh, văn bản, biểu ...s1.vndoc.com/.../06/40-de-thi-hoc-ky...6-co-dap-an.pdf · "/ 0. 1 2 %$ 3 ( 0 4 5 6 2 3 ( 5789 578: 2

VnDoc - Tải tài liệu, văn bản pháp luật, biểu mẫu miễn phí

Part 2: Write a letter to your friend about your next summer vacation, using these cues. (1pt) (Viết1 lá thư cho bạn nói về kế hoạch kì nghỉ hè sắp tới)

- Place you are going to visit.

- Where you are going to stay.

- How long you are going to stay.

- What you are going to do.

Dear Lan,

Next summer vacation, _______________________________________________________________

_____________________________________________________________________________________

_____________________________________________________________________________________

____________________________________________________________________________

Love,

Hoa

IV. Use of English: (2,0pts)Choose the best option (from A, B, C or D) to complete each sentence or passage. (Chọn ý đúng nhất

từ A, B, C hoặc D để hoàn thành mỗi câu sau hoặc đoạn văn).1. The boy ______________ books in the library at present.

A. read B. reads C. reading D. is reading

2.We are from Australia. We speak ______________ .

A. Australian B. English C. Canada D. France

3. Are there any ________ on the table?

A. meat B. carrot C. lettuce D. onions

4. ___________ is a bottle of cooking oil? – It’s 25,000 dong.

A. How many B. What C. How old D. How much

Miss. Huong is our (5) __________. She teaches us history. She is tall and thin (6) __________ she is not

weak. She plays sports a lot. Her favorite sport is aerobics. In the evening, she (7) __________ to music or

watches television. This summer vacation, she (8) __________ visit Ha Long Bay.

5. A. worker B. teacher C. father D. doctor

6. A. but B. and C. or D. so

7. A. listen B. listens C. is listening D. listening

Page 170: Tài liệu, học tập, trắc nghiệm, tiếng anh, văn bản, biểu ...s1.vndoc.com/.../06/40-de-thi-hoc-ky...6-co-dap-an.pdf · "/ 0. 1 2 %$ 3 ( 0 4 5 6 2 3 ( 5789 578: 2

VnDoc - Tải tài liệu, văn bản pháp luật, biểu mẫu miễn phí

8. A. is B. going to C. is going to D. is go to

V. Speaking: (2,0pts)

Giáo viên tổ chức cho học sinh thực hiện phần nói bắt đầu lúc 14 giờ 15 phút ngày 23/4/2015.

- THE END –

Page 171: Tài liệu, học tập, trắc nghiệm, tiếng anh, văn bản, biểu ...s1.vndoc.com/.../06/40-de-thi-hoc-ky...6-co-dap-an.pdf · "/ 0. 1 2 %$ 3 ( 0 4 5 6 2 3 ( 5789 578: 2

VnDoc - Tải tài liệu, văn bản pháp luật, biểu mẫu miễn phí

ANSWER KEYSI. Listening: (2,0pts)Part 1:

1. round 2. black 3. eyes 4. teeth

Part 2:

1. T 2. F 3. T 4. F

II. Reading: (2,0pts)

Part 1:1. B 2. B 3. D 4. C

Part 2:1. They go to Quang Trung Secondary School.

2. They often play soccer in their free time.

3. They usually play badminton in the fall.

4. Yes, they do.

III. Writing: (2,0pts)

Part 1:1. Why don’t we take some photos?

2. It is short.

3. How often does she go to the movies?

4. What weather do they like?

Part 2:

Dear Lan,

Next summer vacation, I’m going to visit ______________. I’m going to stay ______________. I’m

going to stay for ______________. I’m going to ______________.

Love,

Hoa

IV. Use of English: (2,0pts)

1. D 2. B 3. D 4. D

5. B 6. A 7. B 8. C

V. Speaking: (2,0pts)

Page 172: Tài liệu, học tập, trắc nghiệm, tiếng anh, văn bản, biểu ...s1.vndoc.com/.../06/40-de-thi-hoc-ky...6-co-dap-an.pdf · "/ 0. 1 2 %$ 3 ( 0 4 5 6 2 3 ( 5789 578: 2

VnDoc - Tải tài liệu, văn bản pháp luật, biểu mẫu miễn phí

TAPE TRANSCRIPTPart 1:

Miss Chi is tall and thin. She has a round face. She has long black hair. She has brown eyes. She has a

small nose. She has full lips and small white teeth.

Part 2:Nam: How do you feel, Lan? Nam: I’m hungry. I’d like some noodles.

Lan : I’m hot and I am thirsty. What about you, Ba? Would you like noodles?

Nam: What would you like? Ba: No, I wouldn’t. I’m tired. I’d like to sit down.

Lan : I’d like some orange juice. Nam: Would you like noodles, Lan?

What about you? Lan : No, I wouldn’t. I’m full. But I am thirsty. I’d like a

drink.

SPEAKINGPhần A:

Học sinh bốc thăm chọn 1 trong 3 chủ đề sau và chuẩn bị theo gợi ý (khoảng 5 phút).

Topic 1:Weather

1. What / weather / like / season?2. What / you / do / in / season?

3. What weather / you / like?

4. What / you / do / when it’s _____ ?

Topic 2: Future plan1. Which place / you / going / visit?

2. Where / you / going / stay?

3. How long / you / going / stay?

4. What / you / going / do?

Topic 3: Sports and pastime

1. Which sports / you / play?

2. How often / you / play ______?

3. What / you / do / free time?

4. Do you like _________?

Phần B:

Page 173: Tài liệu, học tập, trắc nghiệm, tiếng anh, văn bản, biểu ...s1.vndoc.com/.../06/40-de-thi-hoc-ky...6-co-dap-an.pdf · "/ 0. 1 2 %$ 3 ( 0 4 5 6 2 3 ( 5789 578: 2

VnDoc - Tải tài liệu, văn bản pháp luật, biểu mẫu miễn phí

Giáo viên hỏi và đánh giá kết quả phần trả lời của học sinh.

Topic 1:Weather

1. What’s the weather like in the Spring / Summer / Fall / Winter?2. What do you do in the Spring / Summer / Fall / Winter?

3. What weather do you like?

4. What do you do when it’s __________? (Weather that Student has just answered in question 3)

Topic 2: Future plan1. Which place are you going to visit?

2. Where are you going to stay?

3. How long are you going to stay?

4. What are you going to do?

Topic 3: Sports and pastime

1. Which sports do you play?

2. How often do you play ______? (Sport that Student has just answered in question 1)

3. What do you do in your free time?

4. Do you like _________.

- THE END -

Page 174: Tài liệu, học tập, trắc nghiệm, tiếng anh, văn bản, biểu ...s1.vndoc.com/.../06/40-de-thi-hoc-ky...6-co-dap-an.pdf · "/ 0. 1 2 %$ 3 ( 0 4 5 6 2 3 ( 5789 578: 2

TRƯỜNG THCS HỒ XUÂN HƯƠNG-----------------------

ĐỀ KIỂM TRA HỌC KỲ II NĂM HỌC 2014 - 2015MÔN: TIẾNG ANH 6

Thời gian: 45 phút (không kể thời gian phát đề)

I. Listen and fill one words in the blanket.

Minh likes walking. On …(1)….weekend, he often …(2)….….walking in the mountains.He …(3)….……goes with two friends. Minh and his friendsalways ……(4)….……strong boot and warm clothes. They always take……(5)….…andwater and a camping stove. Sometimes, they (6)…. overnight.

II. Choose the word whose underlined part has different pronunciation from those ofthe others. Write the answer (A, B, C or D) in the box.

1. A. Times B. Games C. Watches D. Goes

2. A. Year B. Wear C. Near D. Hear

3. A. With B. Fifth C. Math D. Thank

4. A. Fishing B. Skip C. Picnic D. Twice

5. A. Apartment B. Pastime C. Camping D. Badminton

III. Write the best option ( A, B, C or D ) in the answer box to complete each of thesentences.1. We usually go swimming …………………weekends.A. to B. with C. on D. for2. Ho Chi Minh city has a ………….of 3.5 millionA. Population B.capital C. city D. town3. She never goes………………on Saturday sundays.A. school B. swimming C. movie theater D. video games4. We are going to visit our grandparents ………. a weekA. How B. Which C. that D. For5. Bruce is from Australia. He speaks…………A. Austratian B. English C. French D. Chinese6. Nam ………………soccer with his friends at the moment.A. doing B. playing C. is doing D. is playing7. He always gets up late and……………………………………………A. never he does morning exercises B. he never does morning exercisesC. he does morning exercises never D. he never do morning exercises8. Mr. Tuan is loading the truck ………………vegetablesA. to B. for C. with D. of

Page 175: Tài liệu, học tập, trắc nghiệm, tiếng anh, văn bản, biểu ...s1.vndoc.com/.../06/40-de-thi-hoc-ky...6-co-dap-an.pdf · "/ 0. 1 2 %$ 3 ( 0 4 5 6 2 3 ( 5789 578: 2

IV. Supply the correct forms of the verbs:1. I ( play) __________ soccer with my friends at the moment.2. We shouldn’t ( cut ) ___________down the trees3. What about ( play) ___________ soccer this afternoon ?4. When the lights are red. We must (stop)………….right now5. Nam usually ( go ) ________fishing in his free time

V. Write (A,B,C or D) to show the mistake in each sentence, and the correction in theanswer box. :

1. Why don’t we travel by coach to Huong pagoda? That’s an good idea

A B C D

2. What do you go to school ? – I go to school by bike.

A B C D

3. Does she sometimes plays volley ball with her friends after school?

A B C D

4. The Great Wall of China is over 6.000km thick.

A B C D

5. Peter and Jack is playing volleyball in the yard now.

A B C D

VI. Read the passage then answer the questions:

Miss Van is a doctor. She works in a big hospital in Ha Noi. She is a beautiful womanwith an oval face and round eyes. She has long black hair and a small nose. She likesgoing shopping with her friends in her freetime. She often buy books in the bookshopnear her office,too. But the thing she likes most is listening to music in her own room.Questions:

1. Does she work in a small hospital in Ha Noi ?……………………………………………………………………………2. What color is her hair?

……………………………………………………………………………3. Who does she go shopping with?……………………………………………………………………………4. What does she like most?……………………………………………………………………………

Page 176: Tài liệu, học tập, trắc nghiệm, tiếng anh, văn bản, biểu ...s1.vndoc.com/.../06/40-de-thi-hoc-ky...6-co-dap-an.pdf · "/ 0. 1 2 %$ 3 ( 0 4 5 6 2 3 ( 5789 578: 2

VII. Rewrite the following sentences that keep the same meaning:1. How many people does your family have?.

-> How ………………………………………………………?

2. I want two kilos of carrots

-> I would......................................................................................................

3. My favorite food are chiken and beef.

-> I like…………………………………………………

4. Lan walks to school every day

-> Lan goes................................................................

--------- The end --------

Page 177: Tài liệu, học tập, trắc nghiệm, tiếng anh, văn bản, biểu ...s1.vndoc.com/.../06/40-de-thi-hoc-ky...6-co-dap-an.pdf · "/ 0. 1 2 %$ 3 ( 0 4 5 6 2 3 ( 5789 578: 2

HƯỚNG DẪN CHẤM

I. Listening (1,5 diểm với 0,3 điểm cho mỗi câu đúng)GV mở phần C6 unit 12 sgk lớp 6 trang 132 - 133 cho hs nghe 3 lần

1. the 2. goes 3. often 4. wear 5. food 6. campII. 1,5 điểm ( 0,3 điểm cho mỗi câu đúng)

Câu 1 2 3 4 5

Đáp án C B A D A

III. 2 điểm (0,25 điểm cho mỗi câu đúng)

Câu 1 2 3 4 5 6 7 8

Đáp án C A A D B D B C

IV. 1,5 điểm, (0,3 điểm cho mỗi câu đúng)1. ís playing 2. cut3. playing 4. stop 5. goesV. 1,5diểm (0,3 điểm cho mỗi câu đúng)

Câu 1 2 3 4 5

Đáp án D A B A A

VI. 1 điểm (0,25 điểm cho mỗi câu đúng)1. No, She doesn’t2. It’s black3. She goes shopping with her friends4. She likes listening to music most

VII. 1 điểm (0,25 điểm cho mỗi câu đúng)1. How many people are there in your family?2. .… like two kilos of carrots3. I like chicken and beef4…goes to school on foot….

………The end…………

Page 178: Tài liệu, học tập, trắc nghiệm, tiếng anh, văn bản, biểu ...s1.vndoc.com/.../06/40-de-thi-hoc-ky...6-co-dap-an.pdf · "/ 0. 1 2 %$ 3 ( 0 4 5 6 2 3 ( 5789 578: 2

VnDoc - Tải tài liệu, văn bản pháp luật, biểu mẫu miễn phí

………………………………………………………………………………………...........................Lưu ý : Học sinh không được sử dụng các loại tài liệu,từ điển

Học sinh làm bài trực tiếp vào tờ đề thi

Điểm Giám thị 1 Giám thị 2 Mã đề Số phách

01

I. Choose the word that has the underlined part pronounced differently from the others. (1m)

1. A. books B. pencils C. rulers D. bags

2. A. read B. teacher C. eat D. ahead

3. A. tenth B. math C. brother D. theater

4. A. has B. name C. family D. lamp

Answer:

1. 2. 3. 4.

II. Choose the best answer to complete these sentences. (2m)1. Hung is a .................... He’s strong and heavy.

A. gymnast B. weight-lifter C. driver D. doctor

2. I like hot ....................

A. season B. weather C. activity D. class

3. What .................... your father doing now? He is playing badminton.

A. is B. are C. do D. does

4. .................... does your brother go fishing? He goes once a week.

A. How old B. How often C. How D. What

5. He .................... goes fishing in the winter. It’s too cold.

A. often B. always C. usually D. never

PHÒNG GD&ĐT TP HÀ TĨNHTRƯỜNG THCS LÊ BÌNH

KỲ THI KSCL HỌC KỲ II NĂM HỌC 2015-2016Môn: Tiếng Anh - Lớp 6 - Thời gian: 45 phút

Họ và tên : ……………………. ... Lớp: 6…… Số báo danh:……. Số Phách:………

Page 179: Tài liệu, học tập, trắc nghiệm, tiếng anh, văn bản, biểu ...s1.vndoc.com/.../06/40-de-thi-hoc-ky...6-co-dap-an.pdf · "/ 0. 1 2 %$ 3 ( 0 4 5 6 2 3 ( 5789 578: 2

VnDoc - Tải tài liệu, văn bản pháp luật, biểu mẫu miễn phí

6. What about .................... by bike?

A. travel B. to travel C. traveling D. travels

7. .................... place are Phuong and Mai going to visit first?

A. Where B. Which C. How D. How long

8. My house is near a market so it’s very __________.

A. big B. noisy C. quiet D. small

Answer:

1. 2. 3. 4. 5. 6. 7. 8.

III. Put the verbs into the correct form. (1,5m)1. My school ( have )……………………two floors.

2. Look! The plane (fly) ..............to the airport, it (land).......................

3.What about ( visit )…………………………………..our parents ?

4. We shouldn’t (throw)………trash on the street.

5. They (not go)………….swimming in the winter.

Answer:

1……………. 2……………. ……………. 3………….. 4…………….. 5…………….

IV. Make questions for the underlined parts or rearrange to complete these sentences. (1m)

1. We usually play basketball on the weekends.___________________________________

2. They are going to visit Hue next summer. _______________________________________

3. tallest / Tuan / our / is / the / class/ in.__________________________________________

4. the / spring / in / What / weather / is / like / the?_______________________________________

V. Read the passage carefully then choose the correct words to fill in the blanks. (1,5m)

favorite does plays or but everyday

Miss Loan is our teacher. She teaches us English. She is tall and thin (1)………..she is not weak. She

(2)…………….a lot of sports. Her (3)…………...sport is aerobics. She (4)…………aerobics three times a

Page 180: Tài liệu, học tập, trắc nghiệm, tiếng anh, văn bản, biểu ...s1.vndoc.com/.../06/40-de-thi-hoc-ky...6-co-dap-an.pdf · "/ 0. 1 2 %$ 3 ( 0 4 5 6 2 3 ( 5789 578: 2

VnDoc - Tải tài liệu, văn bản pháp luật, biểu mẫu miễn phí

week. She likes jogging, too. She jogs in a park near her house (5)……………In her free time she listens to

music (6)…………..watches TV.

Answer:

1. 2. 3. 4. 5. 6.

VI. Write the second sentence so that it has a similar meaning to the first.(2m)1. There are a lot of flowers in my garden.

=> My garden …………………………………………..

2. Ha Tinh is smaller than Nghe An.

=> Nghe An ………………………………………..

3.Let’s go to Ha Long Bay next summer vacation.

=> What about ………………………………………………..?

4. I want some milk.

=> I’d ………………………………………………………….

VII.Write a passage to describe your close friend.(1m)

------THE END------

Page 181: Tài liệu, học tập, trắc nghiệm, tiếng anh, văn bản, biểu ...s1.vndoc.com/.../06/40-de-thi-hoc-ky...6-co-dap-an.pdf · "/ 0. 1 2 %$ 3 ( 0 4 5 6 2 3 ( 5789 578: 2

VnDoc - Tải tài liệu, văn bản pháp luật, biểu mẫu miễn phí

PHÒNG GD&ĐT TP HÀ TĨNHTRƯỜNG THCS LÊ BÌNH

KỲ THI KSCL HỌC KỲ II NĂM HỌC 2015-2016Môn: Tiếng Anh - Lớp 6 - Thời gian: 45 phút

Họ và tên : ……………………. ... Lớp: 6…… Số báo danh:……. Số Phách:………

………………………………………………………………………………………..........................

Lưu ý : Học sinh không được sử dụng các loại tài liệu,từ điển

Học sinh làm bài trực tiếp vào tờ đề thi

Điểm Giám thị 1 Giám thị 2 Mã đề Số phách

02

I. Choose the word that has the underlined part pronounced differently from the others. (1m)

1. A. does B. watches C. finishes D. brushes

2. A. fine B. kind C. nine D. tin

3. A. slow B. go C. nobody D. do

4. A. amazing B. apartment C. address D. armchair

Answer:

1. 2. 3. 4.

II. Choose the best answer to complete these sentences. (2m)1. What .................... your father doing now? He is playing badminton.

A. is B. are C. do D. does

2. He .................... goes fishing in the winter. It’s too cold.

A. often B. always C. usually D. never

3. Long is a .................... He’s strong and heavy.

A. gymnast B. weight-lifter C. driver D. doctor

4. .................... place are Phuong and Mai going to visit first?

A. Where B. Which C. How D. How long

5. My house is near a market so it’s very __________.

A. big B. noisy C. quiet D. small

Page 182: Tài liệu, học tập, trắc nghiệm, tiếng anh, văn bản, biểu ...s1.vndoc.com/.../06/40-de-thi-hoc-ky...6-co-dap-an.pdf · "/ 0. 1 2 %$ 3 ( 0 4 5 6 2 3 ( 5789 578: 2

VnDoc - Tải tài liệu, văn bản pháp luật, biểu mẫu miễn phí

6. She likes hot ....................

A. season B. weather C. activity D. class

7. .................... does your brother go fishing? He goes once a week.

A. How old B. How often C. How D. What

8. What about .................... by bike?

A. travel B. to travel C. traveling D. travels

Answer:

1. 2. 3. 4. 5. 6. 7. 8.

III. Put the verbs into the correct form. (1,5m)1. My family (have)……………………five people.

2. Look! The plane (fly) ..............to the airport, it (land).......................

3.How about ( visit )…………………………………..our parents ?

4. We shouldn’t (throw)………trash on the street.

5. Nam (not go)………….swimming in the winter.

Answer:

1……………. 2……………. ……………. 3………….. 4…………….. 5…………….

IV. Make questions for the underlined parts or rearrange to complete these sentences(1m)

1. My brother is traveling to school by car. _____________________________________

2. They are going to visit Hue next summer. _______________________________________

3. tallest / Tuan / our / is / the / class/ in.__________________________________________

4. the / spring / in / What / weather / is / like / the?_______________________________________

V. Read the passage carefully then choose the correct words to fill in the blanks.(1,5)

At , teacher , eleven , leaves , in , goes

This is my sister, Trang. She is a (1)…………She teaches at a small school(2)………..the village.

She gets up (3)…………..six. Every morning she (4)………..at the quarter to seven. The school is not near

Page 183: Tài liệu, học tập, trắc nghiệm, tiếng anh, văn bản, biểu ...s1.vndoc.com/.../06/40-de-thi-hoc-ky...6-co-dap-an.pdf · "/ 0. 1 2 %$ 3 ( 0 4 5 6 2 3 ( 5789 578: 2

VnDoc - Tải tài liệu, văn bản pháp luật, biểu mẫu miễn phí

her house, so she (5)…………to work by bicycle. Her class starts at seven and ends at(6)……….My sister

rides home and has lunch at quarter to twelve.

Answer:

1. 2. 3. 4. 5. 6.

VI. Write the second sentence so that it has a similar meaning to the first.(2m)1. There are 400 students in my school.

=> My school …………………………………………..

2. Tuan is shorter than his brother.

=> Tuan’s brother ………………………………………..

3.What about going to Thien Cam beach?

=> Let’s………………………………………………..

4. I want some milk.

=> I’d ………………………………………………………….

VII.Write a passage to describe your close friend.(1m)

------THE END------

Page 184: Tài liệu, học tập, trắc nghiệm, tiếng anh, văn bản, biểu ...s1.vndoc.com/.../06/40-de-thi-hoc-ky...6-co-dap-an.pdf · "/ 0. 1 2 %$ 3 ( 0 4 5 6 2 3 ( 5789 578: 2

PHÒNG GD&ĐT THÁI THỤY

ĐỀ KIỂM TRA HỌC KỲ II NĂM HỌC 2015 - 2016

Môn : Tiếng Anh 6 - Chương trình đại trà (Thời gian làm bài 45 phút)

I. NGHE HIỂU

Nghe và chọn đáp án đúng nhất trong số A, B hoặc C để hoàn thành câu.

1. Our class begins at . . . . . . A. 4:05 B. 4:15 C. 4:50 2. My mother left this morning at . . . . . . A. 9:03 B. 9:13 C. 9:30 3. I'm going to catch my bus at . . . . . . A. 3:40 B. 4:20 C. 12:04 4. Let's get together at . . . . . . A. 12:05 B. 5:12 C. 5:22 5. The store closes today at . . . . . . A. 4:06 B. 5:45 C. 6:15 6. The movie starts at . . . . . . A. 7:04 B. 7:14 C. 7:40 7. She called this morning at . . . . . . A. 10:00 B. 10:05 C. 10:10 8. Their plane arrives at . . . . . . A. 11:05 B. 4:12 C. 11:45 9. Please come to my office at . . . . . . A. 8:05 B. 8:15 C. 8:25 10. I have to work at . . . . . . A. 3:03 B. 3:13 C. 3:30

II. NGỮ ÂM - TỪ VỰNG - NGỮ PHÁP

Bài 1. Chọn 1 từ trong số A, B, C hoặc D mà phần gạch chân có cách đọc khác so với các từ còn lại

ở mỗi dòng.

11. A. nine B. time C. ride D. children 12. A. water B. walk C. talk D. work 13. A. go B. home C. photo D. hot Bài 2. Chọn đáp án đúng nhất trong số A, B, C hoặc D để hoàn thành câu 14. It is twelve o’clock now. We . . . . . to have lunch. A. want B. to want C. wanting D. are wanting 15. Do you like pop music ? - . . . . . . A. I would. B. Yes, a lot. C. No, I like it . D. Yes, we are. 16. Nam usually goes . . . . . after school. A. to fishing B. to home C. fishing D. home back 17. - A: I am very tired. - B: . . . . . . A. Me too. B. For me the same. C. Also me. D. I also. 18. - A: “ . . . . . do the Hoang Ha buses run?” - B: “Every twenty minutes every day” A. How far B. What time C. How often D. How much 19. - A: “How big is Malaysia ?” - B: “It’s much . . . . . than Singapore.” A. big B. smaller C. smallest D. bigger 20. What time . . . . . on television? A. is the news B. are the news C. is news D. are news 21. My father is a good . . . . . He works in a big restaurant. A. doctor B. worker C. cooker D. cook 22. . . . . . . do you get there? – We walk, of course. A. Why B. What C. How by D. How 23. Excuse me, I’d like to . . . . . . this letter to the USA. A. send B. mail C. post D. All A,B,C 24. My head hurts. I have an awful . . . . . . . A. toothache B. headache C. earache D. backache 25. There . . . . . a lot of rain in summer in our country.

Page 185: Tài liệu, học tập, trắc nghiệm, tiếng anh, văn bản, biểu ...s1.vndoc.com/.../06/40-de-thi-hoc-ky...6-co-dap-an.pdf · "/ 0. 1 2 %$ 3 ( 0 4 5 6 2 3 ( 5789 578: 2

A. are B. be C. aren’t D. is 26. These are my notebooks, and those are . . . . . A. she B. her C. hers D. your’s

Bài 3. Tìm một lỗi sai trong số A, B, C hoặc D trong mỗi câu sau

27. What do Ba do when it is hot? A B C D 28. How many rooms there are in the new house? A B C D 29. I’m very thirty. I would like some apple juice. A B C D 30. We go to the library every Monday afternoons. A B C D III. ĐỌC HIỂU - VIẾT Bài 1. Đọc kỹ đoạn văn sau và điền vào mỗi chỗ trống bằng một từ thích hợp. I have a very close friend. Her name is Mai. She is eleven years old and she is in (31) . . . . . six. She can speak English very well. She has English (32) . . . . . Tuesday, Thursday and Friday. She (33) . . . . . also play the piano. In her room, there is a piano and some books. Every day she gets up at six. She has breakfast at six thirty and goes to school at a quarter to seven. She has her (34) . . . . . from seven o’clock to eleven thirty. Then she goes home and has (35) . . . . . at twelve o’clock. It’s three o’clock in the afternoon now and Nga is in her room. She is doing her homework.

Bài 2. Viết lại câu sao cho giữ nguyên nghĩa với câu cho trước

36. She always cycles to work? She always goes . . . . . . . . . . . . . . . . . . . . . . . . . . . . . . . . . . . 37. She likes noodles very much. Her favorite . . . . . . . . . . . . . . . . . . . . . . . . . . . . . . . . . . . . . 38. What is the price of these apples? How much . . . . . . . . . . . . . . . . . . . . . . . . . . . . . . . . . . . . . . 39. The car is blue. It is . . . . . . . . . . . . . . . . . . . . . . . . . . . . . . . . . . . . . . . . . . 40. She is taller than her friend. Of the two girls, she is . . . . . . . . . . . . . . . . . . . . . . . . . . . . . . . .

- The end –

Họ và tên thí sinh: ………………………………………. Số báo danh: ……………

Page 186: Tài liệu, học tập, trắc nghiệm, tiếng anh, văn bản, biểu ...s1.vndoc.com/.../06/40-de-thi-hoc-ky...6-co-dap-an.pdf · "/ 0. 1 2 %$ 3 ( 0 4 5 6 2 3 ( 5789 578: 2

PHÒNG GD&ĐT THÁI THỤY

HƯỚNG DẪN CHẤM KIỂM TRA HỌC KỲ II NĂM HỌC 2015 - 2016

Môn : Tiếng Anh 6 - Chương trình đại trà

I. ĐÁP ÁN

1. B 6. C 11. D 16. C 21. D 26. C 2. B 7. C 12. D 17. A 22. D 27. A 3. A 8. C 13. D 18. C 23. D 28. B 4. A 9. A 14. A 19. D 24. B 29. B 5. B 10. C 15. B 20. A 25. D 30. D 31. grade 32. on 33. can 34. classes 35. lunch

36. She always cycles to work? She always goes to work by bike/ bicycle. 37. She likes noodles very much. Her favorite food is noodles. 38. What is the price of these apples? How much are these apples?/ How much do these apples cost? 39. The car is blue. It is a blue car. 40. She is taller than her friend. Of the two girls, she is the taller.

II. HƯỚNG DẪN CHẤM - Điểm toàn bài 10,0 điểm = 40 câu. Mỗi câu đúng đạt 0,25 điểm - Làm tròn: + 5,25 làm tròn thành 5,5 + 5,5 giữ nguyên + 5,75 làm tròn thành 6,0

- The end –

Page 187: Tài liệu, học tập, trắc nghiệm, tiếng anh, văn bản, biểu ...s1.vndoc.com/.../06/40-de-thi-hoc-ky...6-co-dap-an.pdf · "/ 0. 1 2 %$ 3 ( 0 4 5 6 2 3 ( 5789 578: 2

VnDoc - Tải tài liệu, văn bản pháp luật, biểu mẫu miễn phí

PHÒNG GIÁO DỤC VÀ ĐÀO TẠOCHÂU THÀNH

---------------------------ĐỀ CHÍNH THỨC

ĐỀ KIỂM TRA HKII, NĂM HỌC 2012 - 2013MÔN: TIẾNG ANH - LỚP 6

Thời gian: 45 phút (không kể phát đề)

Điểm bằng số Điểm bằng chữ Giám khảo 1 Số thứ tự

Giám khảo 2 Số phách

I. Choose the best option (from A, B, C or D) to complete each sentence: (5,0pts) (Chọn ý đúng nhất

từ A, B, C hoặc D để hoàn thành mỗi câu sau).1. What about ______________ soccer this weekend, Lan?

A. play B. plays C. to play D. playing

2.Mom: Are there any onions on the table? Phuong: ______________.

A. No, there are B. No, there isn’t C. Yes, there are D. Yes, there

aren’t

3. How much ______________ does he want? – One kilo, please.

A. cabbages B. potatoes C. beef D. carrot

4. Mai is ______________ her homework at the moment.

A. doing B. playing C. going D. visiting

5. Nam is ______________. He’d like to sit down.

A. tired B. cold C. hungry D. full

6. ______________ is his hair? – It’s brown.

A. What B. What color C. Where D. Which

7. What’s your ______________? – I’m Vietnamese.

A. language B. country C. nationality D. name

8. A ______________ of noodles is 12.000 dong.

A. bar B. bottle C. box D. bowl

9. Nga doesn’t want ______________ to Huong Pagoda by bike because it’s too hot.

A. to going B. go C. going D. to go

Page 188: Tài liệu, học tập, trắc nghiệm, tiếng anh, văn bản, biểu ...s1.vndoc.com/.../06/40-de-thi-hoc-ky...6-co-dap-an.pdf · "/ 0. 1 2 %$ 3 ( 0 4 5 6 2 3 ( 5789 578: 2

VnDoc - Tải tài liệu, văn bản pháp luật, biểu mẫu miễn phí

10.Mai: ______________ go to Huong Pagoda? Ba: That’s a good idea.

A. Why don’t we B. Do you C. What aboutD. Let’s

11. What’s ______________ like in the spring? – It’s warm.

A. weather B. winter C. the weatherD. the winter

12. Miss Chi is very beautiful. She has ______________.

A. black long hair B. long black hair C. hair long black D. black hair

long

13. I am thirsty. I would like ______________ can of soda.

A. a B. an C. some D. any

14. He isn’t fat. He is ______________.

A. short B. heavy C. tall D. thin

15. My father is going to travel to Hanoi ______________.

A. at present B. every day C. tomorrow D. now

16. Do ______________ like cold weather?

A. they B. he C. she D. your brother

17. Mexico is ______________ city in the world.

A. the bigger B. the biggest C. biggest D. bigger than

* Choose the word that is not correct: (Chọn từ không đúng trong câu)

18. We sometimes make a picnic in the park.

A B C D

* Choose the word that has the underlined part pronounced differently from the others. (Chọn từ cóphần gạch chân được phát âm khác với những từ còn lại.)

19. A. tennis B. tent C. soccer D. never

* Odd one out: (Chọn 1 từ không cùng nhóm với những từ còn lại)

20. A. hand B. red C. orange D. blue

II. Circle the best options to complete the passage (1pt): (Khoanh tròn ý đúng nhất để hoàn thànhđoạn văn)

Minh likes (21) ____________. On the (22) ____________, he often goes walking in the mountains. He

usually goes with two friends. Minh and his friends always (23) ____________ strong boots and warm

Page 189: Tài liệu, học tập, trắc nghiệm, tiếng anh, văn bản, biểu ...s1.vndoc.com/.../06/40-de-thi-hoc-ky...6-co-dap-an.pdf · "/ 0. 1 2 %$ 3 ( 0 4 5 6 2 3 ( 5789 578: 2

VnDoc - Tải tài liệu, văn bản pháp luật, biểu mẫu miễn phí

clothes. (24) ____________ always take food, water and a camping stove. Sometimes they camp

overnight.

(21) A. walk B. walking C. walks D. to walking

(22) A. summer B. weekend C. week D. morning

(23) A. are going to wear B. are wearing C. wear D. wears

(24) A. They B. He C. She D. We

III. Give the correct form of the verbs in the brackets (1pt): (Cho dạng đúng của động từ trong

ngoặc)25. Lan sometimes (visit) _________________ her old friend in the vacation.

26. I (see) _________________ a new film tonight.

27. They (watch) _________________ television now.

28. My sister can (go) _________________ to the store for mom.

IV. Write sentences from the words given (1pt): (Viết câu từ các từ đã cho.)29. We / often / volleyball / summer.

……………………………………………………………………………………………………….

30. Hoa / walk / school / every day.

……………………………………………………………………………………………………….

31. My mother/ have / full lips.

……………………………………………………………………………………………………….

32. The students / be / the canteen.

……………………………………………………………………………………………………….

V. Read the text carefully, then answer the questions (2pts): (Đọc đoạn văn cẩn thận, trả lời câu hỏi)Miss Huong is a teacher. She teaches us Math. She is tall and thin but she is not weak. She plays sports

every day. Her favorite sport is aerobics. She does aerobics three times a week. She likes jogging, too. She

jogs in the park near her house every morning. In her free time, she reads and helps her mom.

33. What does Miss Huong do?

……………………………………………………………………………………………………….

34. Is she weak?

Page 190: Tài liệu, học tập, trắc nghiệm, tiếng anh, văn bản, biểu ...s1.vndoc.com/.../06/40-de-thi-hoc-ky...6-co-dap-an.pdf · "/ 0. 1 2 %$ 3 ( 0 4 5 6 2 3 ( 5789 578: 2

VnDoc - Tải tài liệu, văn bản pháp luật, biểu mẫu miễn phí

……………………………………………………………………………………………………….

35. Does she play sports?

……………………………………………………………………………………………………….

36. What is her favorite sport?

……………………………………………………………………………………………………….

37. How often does she jog?

……………………………………………………………………………………………………….

38. What does she do in her free time?

……………………………………………………………………………………………………….

* About you:

39. What do you do in your free time?

……………………………………………………………………………………………………….

40. How often do you help your mom?

……………………………………………………………………………………………………….

-THE END-

Page 191: Tài liệu, học tập, trắc nghiệm, tiếng anh, văn bản, biểu ...s1.vndoc.com/.../06/40-de-thi-hoc-ky...6-co-dap-an.pdf · "/ 0. 1 2 %$ 3 ( 0 4 5 6 2 3 ( 5789 578: 2

VnDoc - Tải tài liệu, văn bản pháp luật, biểu mẫu miễn phí

ANSWER KEYSI/ 5,0 đ - Mỗi lựa chọn đúng 0,25 đ (0,25đ x 20c = 5,0đ)

1. D 5. A 9. D 13. A 17. B

2. C 6. B 10. A 14. D 18. B

3. C 7. C 11. C 15. C 19. C

4. A 8. D 12. B 16. A 20. A

II/ 1,0 đ - Mỗi câu điền đúng 0,25 đ (0,25 đ x 4c = 1,0 đ)21. B

22. B

23. C

24. A

III/ 1,0 đ - Mỗi động từ chia đúng 0,25 đ (0,25 đ x 4c = 1,0 đ)25. visits

26. am going to see

27. are watching

28. go

IV/ Mỗi câu viết lại đúng 0,25 đ (0,25 đ x 4c = 1,0 đ)29. We often play volleyball in the summer.

30. Hoa walks to school every day.

31. My mother has full lips.

32. The students are at the canteen.

V/ 2,0 đ. Mỗi câu trả lời đúng 0,25 đ (0,25đ x 8c = 2,0đ)33. Miss Huong / She is a teacher.

34. No, she isn’t.

35. Yes, she does.

36. Her favorite sport is aerobics.

37. She jogs every morning.

Page 192: Tài liệu, học tập, trắc nghiệm, tiếng anh, văn bản, biểu ...s1.vndoc.com/.../06/40-de-thi-hoc-ky...6-co-dap-an.pdf · "/ 0. 1 2 %$ 3 ( 0 4 5 6 2 3 ( 5789 578: 2

VnDoc - Tải tài liệu, văn bản pháp luật, biểu mẫu miễn phí

38. In her free time, she reads and helps her mom.

39. (Students’ answers)

40.

- THE END –

Page 193: Tài liệu, học tập, trắc nghiệm, tiếng anh, văn bản, biểu ...s1.vndoc.com/.../06/40-de-thi-hoc-ky...6-co-dap-an.pdf · "/ 0. 1 2 %$ 3 ( 0 4 5 6 2 3 ( 5789 578: 2

PHÒNG GD&ĐT BẢO LỘC

LÂM ĐỒNG

ĐỀ KIỂM TRA HỌC KỲ II

MÔN TIẾNG ANH, LỚP 6

Thời gian làm bài: 45 phút

I. Vocabulary and Grammar

Circle the best option A, B, or C to complete each of the following sentences. (2.5

points)

1. I'm ____. I'd like some noodles.

A. thirsty B. hungry C. empty

2. There is____ water in the bottle.

A. any B. many C. some

3. What's he doing? He's ____.

A. swims B. swimming C. swim

4. We go to the movies____ Sunday evening.

A. to B. at C. on

5. It's very ____ in the winter.

A. cold B. hot C. cool

6. They are doing ____ homework at the moment.

A. them B. theirs C. their

7. He sometimes____ to pop music.

A. listens B. reads C. watches

8. - How ____ is a sandwich? - It's 2,000 dong.

A. many B. much C. old

9. - How____ does she go jogging? - Every evening.

A. many B. much C. often

10. ____ is the weather like in the summer?

A. What B. Which C. How

II. Reading

A. Fill in the blanks with the words from the box (2.5 points)

Page 194: Tài liệu, học tập, trắc nghiệm, tiếng anh, văn bản, biểu ...s1.vndoc.com/.../06/40-de-thi-hoc-ky...6-co-dap-an.pdf · "/ 0. 1 2 %$ 3 ( 0 4 5 6 2 3 ( 5789 578: 2

much to natural picnic

go they tent sometimes

Huong and Hoa are close friends. They often (1)____ to the movie theater at the

weekend. They (2) _______ go to the zoo. They also have a (3) ______, but not

always. They go (4) ____ the countryside. They always take a (5) ____, some water

and a camping stove. Sometimes, (6) ____ camp overnight. They like camping very (7)

____. They can enjoy the fresh air and the (8) ____ beauty.

B. Read the passage and do the tasks that follow.

My uncle is an engineer. His name is Hung. He is going on vacation this summer. First,

he is going to visit Ha Long Bay. He is going to stay in a small hotel near the bay for

two days. Then he is going to visit Da Lat for three days. Finally, he is going to visit

some friends in Ho Chi Minh City. He is going to stay there for one day. He is going to

walk along Saigon River. He is going to fly home.

Decide if these statements are True (T) or False (F) (l.0point)

T F

1. Mr. Hung is going to visit three cities this summer.

2. He is going to visit Ha Long Bay first, then Ho Chi

Minh City, and finally Da Lat.

3. He is going to stay in Da Lat for three days.

4. He is going to travel home by train.

Answer the following questions (1.0point)

1. Where is Mr. Hung going to stay in Ha Long Bay?

………………………………………………………………

2. Is he going to visit some friends in Ho Chi Minh City ?

………………………………………………………………

Page 195: Tài liệu, học tập, trắc nghiệm, tiếng anh, văn bản, biểu ...s1.vndoc.com/.../06/40-de-thi-hoc-ky...6-co-dap-an.pdf · "/ 0. 1 2 %$ 3 ( 0 4 5 6 2 3 ( 5789 578: 2

III. Writing

A. Match a sentence in column A with a sentence in column B (2.0 points)

Your answers: 1- ….. 2 - ….. 3 - ….. 4 - …..

B. Rearrange these sets of words to make complete sentences (l.0 point)

1. very/ my/ is/ beautiful/ country

2. smaller/ is/ house/ my/ than / Lan's house

----------- THE END -----------

A

1. How do you feel?

2. What's your favorite sport?

3. How much meat do you need?

4. Is Ha Not smaller than Ho Chi

Minh City?

B

a. Two kilos.

b. I feel happy.

c. Yes, it is.

d. It is soccer.

Page 196: Tài liệu, học tập, trắc nghiệm, tiếng anh, văn bản, biểu ...s1.vndoc.com/.../06/40-de-thi-hoc-ky...6-co-dap-an.pdf · "/ 0. 1 2 %$ 3 ( 0 4 5 6 2 3 ( 5789 578: 2

VnDoc - Tải tài liệu, văn bản pháp luật, biểu mẫu miễn phí

1

THE SECOND SEMESTER TESTGRADE :6

I. Choose the word which is pronounced differently from the others. (1m)1. A. clothes B. watches C. benches D. classes2. A. read B. teacher C. eat D. ahead3. A. pens B. books C. rulers D. erasers4. A. kite B. twice C . swim D. pastimesII. Choose the best answer : (3ms)1. I’d like……………..bananas, please!

a. any b. some c. many d. much2. Lemonade, apple juice and……………..are cold drinks.

a. vegetables b. noodles c. iced tea d. beans3. She isn’t heavy. She is……………...

a. thin b. fat c. strong d. light4. Lan……………..aerobic everyday.

a. does b. plays c. goes d. dances5. ……………..sports do you play?

a. What’s b. Who c. Which d. How6. We need a tent to go……………..

a. fishing b. swimming c. camping d. jogging7. They are going to stay……………..their uncle and aunt this summer vacation.

a. in b. at c. by d. with8. What is your nationality? I am……………..

a. Vietnamese b. Vietnam c.England9. There are only two……………..in my country.

a. weather b. seasons c. citadel d. cities10.……………..do you go to school? Every afternoon.

a. How much b. How often c. How many d. How11.What do you often do……………..it is hot?

a. What b. Which c. How d. When12. What about……………… tennis this afternoon?

a. play b. playing c. to play d. playsIII . Supply the correct form of the verbs : (1m)1. Mai……………..orange juice at the moment. (drink)2. I……………..thirsty now. (be)3. My father is going……………..Ha Long Bay. (visit)4. They……………..in the kitchen now. (be)IV. Read1.Complete the dialogue with the words from the box: (2ms)

have - want - why - camping - by - travel - do - going

Thanh : What are we going to (1)……………..in the vacation?Mai : Let’s go (2)……………...Thanh : We don’t (3)…………….. a tent. What about (4)……………..to Hue?

Page 197: Tài liệu, học tập, trắc nghiệm, tiếng anh, văn bản, biểu ...s1.vndoc.com/.../06/40-de-thi-hoc-ky...6-co-dap-an.pdf · "/ 0. 1 2 %$ 3 ( 0 4 5 6 2 3 ( 5789 578: 2

VnDoc - Tải tài liệu, văn bản pháp luật, biểu mẫu miễn phí

2

Mai : I don’t(5)……………..to go to Hue.(6)…………….. don’t we go to Huong pagoda?

Thanh : That’s a good idea.How are we going to (7)……………..?

Mai : Let’s go (8)…………….. minibus.Thanh : Yes, Good idea.2. Answer the questions: (1m)a. What are Mai and Thanh going to do in the vacation?…………………………………………………………………………….b. How are they going to travel?………………………………………………………………………………………V. Arrange the words into the sentences (2ms)

1. want/ Tom/ Mary/ and/ glasses/ two/ of/ water.…………………………………………………………..2. sister/ my/ going/ is/ to/ live/ country/ the/ in.………………………………………………………..3. going/ dinner/ tonight/ is/ have/ to/ where/ Lan?…………………………………………………………..4. the/ spring/ in/ what/ weather/ is/ like/ the?……………………………………………………………….5. building/ tallest/ the/ is/ this/ city/ the/ in/ world.…………………………………………………………………6. Lan’s house/ smaller/ is/ house/ my/ than.…………………………………………………………..7. she/ never/ with/ camping/ goes/ her/ friends.……………………………………………………………8. often/ she/ go/ does/ how/ cinema/ to/ the?…………………………………………………………..

Page 198: Tài liệu, học tập, trắc nghiệm, tiếng anh, văn bản, biểu ...s1.vndoc.com/.../06/40-de-thi-hoc-ky...6-co-dap-an.pdf · "/ 0. 1 2 %$ 3 ( 0 4 5 6 2 3 ( 5789 578: 2

VnDoc - Tải tài liệu, văn bản pháp luật, biểu mẫu miễn phí

3

KEYS:I. Choose the word which is pronounced differently from the others(1 correct = 0.25 x 4 = 1m)1. A 2. D 3. B 4. CII. Choose the correct answer: (1 correct = 0.25 x 12 = 3ms)1. b 2. c 3. d 4.a 5.c 6.c7.d 8.a 9.b 10.b 11.d 12.bIII. Supply the correct form of verbs: (1 correct = 0.25 x 4 = 1m)

1. is drinking2. am3. to visit4. are

IV. Read1. Complete the dialouge: (1 correct = 0.25 x 8 = 2ms)

1. do 2. camping 3.have 4. going 5. want 6.why 7. travel 8.by2. Answer (1 correct = 0.25 x 4 = 1m)a They are going to Huong pagoda.b. They are going to travel by minibusV. Arrange the words into the sentences (1 correct = 0.25 x 8 = 2ms)

1. Tom and Mary want two glasses of water.2. My sister is going to live in the country.3. Where is Lan going to have dinner tonight?4. What is the weather like in the spring?5. This city is the tallest building in the world.6. My house is smaller than Lan’s house.7. She never goes camping with her friends.8. How often does she go to the cinema?

PHÒNG GD- ĐT HUYỆN………….. THI KIỂM TRA CHẤT LƯỢNG HKIITRƯỜNG THCS…………………. MÔN TIẾNG ANH LỚP 6

Thời gian : 45 phút (không kể phát đề)Họ và tên…………………………Lớp

Điểm Số báo danh Giám thị 1:

………………………

Giám khảo 1:

……………………..

Page 199: Tài liệu, học tập, trắc nghiệm, tiếng anh, văn bản, biểu ...s1.vndoc.com/.../06/40-de-thi-hoc-ky...6-co-dap-an.pdf · "/ 0. 1 2 %$ 3 ( 0 4 5 6 2 3 ( 5789 578: 2

VnDoc - Tải tài liệu, văn bản pháp luật, biểu mẫu miễn phí

4

Giám thị 2 :

……………………..

Giám khảo 2 :

…………………….I Vocabulary and GrammarA Choose the word odd one out (1 point) Chọn từ không cùng loại1 a chicken b. beef c. tomato d. meat2 a. fish b. orange- juice c. lemonade d. coffee3 a. Is b. Are c. hello d. Am4.a carrot b. rice c. onion d. lettuceB Circle the best option A, B, or C to complete each of the following sentences. (2.5 points)Hoàn thành các câu sau bằng cách khoanh tròn chữ cái A, B, C và D (2.5 điểm)1. I'm ____. I'd like some noodles.

A. thirsty B. hungry C. empty2. There is____ water in the bottle.

A. any B. many C. some3. What's he doing? He's ____.

A. swims B. swimming C. swim4. We go to the movies____ Sunday evening.

A. to B. at C. on5. It's very ____ in the winter.

A. cold B. hot C. cool6. They are doing ____ homework at the moment.

A. them B. theirs C. their7. He sometimes____ to pop music.

A. listens B. reads C. watches8. - How ____ is a sandwich? - It's 2,000 dong.

A. many B. much C. old9. - How____ does she go jogging? - Every evening.

A. many B. much C. often10. ____ is the weather like in the summer?

A. What B. Which C. HowII. Reading* Read the passage carefullyMy uncle is an engineer. His name is Hung. He is going on vacation this summer. First, he isgoing to visit Ha Long Bay. He is going to stay in a small hotel near the bay for two days.Then he is going to visit Da Lat for three days. Finally, he is going to visit some friends inHo Chi Minh City. He is going to stay there for one day. He is going to walk along Saigonriver. He is going to fly homeA True (T) or False (F) (2.0point)1. Mr. Hung is going to visit three cities this summer………..2. He is going to visit Ha Long Bay first, then Ho Chi Minh City, and finally in Da Lat.…….3. He is going to stay in Da Lat for three days.4. He is going to travel home by train.B Answer the following questions (1.0point)1. Where is Mr. Hung going to stay in Ha Long Bay?

Page 200: Tài liệu, học tập, trắc nghiệm, tiếng anh, văn bản, biểu ...s1.vndoc.com/.../06/40-de-thi-hoc-ky...6-co-dap-an.pdf · "/ 0. 1 2 %$ 3 ( 0 4 5 6 2 3 ( 5789 578: 2

VnDoc - Tải tài liệu, văn bản pháp luật, biểu mẫu miễn phí

5

………………………………………………………………2. Is he going to visit some friends in Ho Chi Minh City?………………………………………………………………III. WritingA Rearrange these sets of words to make complete sentences (l.5 point) hoàn thành cáccâu sau với từ cho sẵn.1. Lan/ tall/ is/ and/ thin.___________________________________________________2. What/ you/ are/ to do/ going/ this summer vacation/?/___________________________________________________________3. I’d like/ meat/ and/ some/ vegetables .____________________________________________________________B Match a sentence in column A with a sentence in column B (2.0 points)Nối câu hỏi ở cột A tương ứng với câu trả lời ở cột B

A B C1. How do you feel? A Two kilos 1+2. What's your favorite sport? B I feel happy 2 +3. How much meat do you need? C Yes, it is 3 +4. Is Ha Not smaller than Ho Chi D I like soccer 4 +Minh City?

ĐÁP ÁN MÔN TIẾNG ANH 6I/ (3 điểm)

A .Mỗi câu trả lời đúng dược 0,251 d, 2 a, 3 c, 4 b

B Mỗi câu trả lời đúng dược 0,251B 2C 3B 4C 5A 6C 7A 8B 9C 10A

II/ READINGA .Mỗi câu trả lời đúng dược 0,5

1 F 2 F 3 T 4 FB Mỗi câu trả lời đúng dược 0,5

1 He is going to stay in a small hotel near the Bay2 Yes, he is

III WRITINGA Mỗi câu trả lời đúng dược 0,5

1/ Lan is tall and thin2/ What are you going to do this summer vacation?3/ I’d like some meat and vegetables.

B 1B 2 D 3A 4C

MA TRẬN ĐỀ KIỂM TRA HỌC KÌ II - TIẾNG ANH 6 (2013-2014)

Tên chủ đề Nhận biết Thông hiểu Vận dụngCộng

TNKQ TL TNKQ TL TNKQ TL

Page 201: Tài liệu, học tập, trắc nghiệm, tiếng anh, văn bản, biểu ...s1.vndoc.com/.../06/40-de-thi-hoc-ky...6-co-dap-an.pdf · "/ 0. 1 2 %$ 3 ( 0 4 5 6 2 3 ( 5789 578: 2

VnDoc - Tải tài liệu, văn bản pháp luật, biểu mẫu miễn phí

6

Grammar &vocabulary

4C1đ

10c2.5đ

14c3.5đ

Reading4c2đ

2c1đ

6c3.0đ

Writing1c0.5đ

2c1đ

3c1.5đ

Tổng số câuTổng số điểmTỉ lệ%

4c1đ

15c5đ

4c2đ

23 c10đ

Page 202: Tài liệu, học tập, trắc nghiệm, tiếng anh, văn bản, biểu ...s1.vndoc.com/.../06/40-de-thi-hoc-ky...6-co-dap-an.pdf · "/ 0. 1 2 %$ 3 ( 0 4 5 6 2 3 ( 5789 578: 2

VnDoc - Tải tài liệu, văn bản pháp luật, biểu mẫu miễn phí

7

THE SECOND SEMESTER EXAMINATIONGRADE 6

I- Choose the best answer : (3pts)1. Lan ___________aerobic everyday.

a. does b. plays c. goes d. dances2. ______________sports do you play?

a. What’s b. Who c. Which d. How3. We need a tent to go __________________.

a. fishing b. swimming c. camping d. jogging4. They are going to stay _____________their uncle and aunt this summer vacation.

a. in b. at c. by d. with5. ___________ do you go to school? Every afternoon.

a. How much b. How often c. How many d. How6. What do you often do ___________it is hot?

a. What b. Which c. How d. When.7. What about _____________ soccer this afternoon?

a. play b. playing c. to play d. plays7. What____________the weather like in the summer?

a. does b. is c. are d. be8. Let’s________________camping.

a. to go b. go c. going d. goes9. _______________are you going to stay there? - For two days.

a. How long b. How often c. How much d. How many10._______________does he want to travel? - By car.

a. What b. How c. Who d. Which11.I’m going to visit my grandmother ______________a week.

a. at b. on c. for d. in12. Which word is different?

a. thirsty b. fly c. hungry d. usuallyII- Put the adverds in their correct positions: (1pt)1. I am late for school. (never).............................................................................................................................2. We go to class on time. (always).............................................................................................................................3. What do you do in the summer? (often)............................................................................................................................4. He jogs in the morning. (usually)............................................................................................................................

III . Supply the correct form of the verbs : (2pts)

Page 203: Tài liệu, học tập, trắc nghiệm, tiếng anh, văn bản, biểu ...s1.vndoc.com/.../06/40-de-thi-hoc-ky...6-co-dap-an.pdf · "/ 0. 1 2 %$ 3 ( 0 4 5 6 2 3 ( 5789 578: 2

VnDoc - Tải tài liệu, văn bản pháp luật, biểu mẫu miễn phí

8

1. Mai ____________orange juice at the moment. (drink)2. I ______________thirsty now. (be)3. My father is going ___________Ha Long Bay. (visit)4. They _____________in the kitchen now. (be)

1. is drinking 2. am 3. to visit 4. are

IV. Complete the dialogue with the words from the box: (2pts)

have - want - why - camping - by - travel - do -going

Thanh : What are we going to ________in the vacation?Mai : Let’s go ___________________.Thanh : We don’t _________ a tent. What about ________to Hue?Mai : I don’t ____________to go to Hue.

_______________ don’t we go to Huong pagoda?Thanh : That’s a good idea.

How are we going to ______________?Mai : Let’s go _______________ minibus.Thanh : Yes, Good idea.V. Read the passage. Then answer the questions: (2pts)

Mai is going on vacation this summer. First she is going to visit Hue for a week. She is goingto stay with her uncle and aunt. Then she is going to visit Nha Trang for three days. There she isgoing to visit the beach and swim.1. True (T) or False (F)? (1pt)

True(T) False (F)

a. Mai is going to visit Hue first.b. She is going to visit Nha Trang for a week

2. Answer the questions: (1pt)

a. How long is Mai going to visit Hue?………………………………………………………………………………………………………………………………………………………………b. What is she going to do in Nha Trang?………………………………………………………………………………………………………………………………………………………………

Page 204: Tài liệu, học tập, trắc nghiệm, tiếng anh, văn bản, biểu ...s1.vndoc.com/.../06/40-de-thi-hoc-ky...6-co-dap-an.pdf · "/ 0. 1 2 %$ 3 ( 0 4 5 6 2 3 ( 5789 578: 2

VnDoc - Tải tài liệu, văn bản pháp luật, biểu mẫu miễn phí

9

TEST OF ENGLISH AS A FOREIGN LANGUAGETIME: 45 MINUTESGRADE: ENGLISH 6

THE FOURTH FORTY FIVE MINUTE TEST

I. Find the word whose underlined part is pronounced differently from the others.(1m)

II. Choose the best answer (3 ms).1. Tam is going to bring his ............... to take photos.

a. tent b. stove c. camera2. He wants …………………. to the zoo.

a. goes b. go c. to go3. What about …….. to the park?

a. to go b. go c. going.4. It is warm in the ………

a. winter b. fall c. spring5. What’s the weather ………… in the summer?

a. likes b. like c. to like6. He ………fishing in his free time.

a .goes always b. always goes c. alway goes7. ……do you go to the zoo? - once a year.

a. How much b. How often c. How many8. There are four …………. in a year.

a. summer b. winter c. seasons9. They are going to stay ………… their uncle and aunt.

a. at b. with c. in10 Nam wants to go ………vacation in Da Lat.

a. on b. at c. to11. ……… go to Ha Noi.

a. Let’s b. Why don’t we c. What about

1/ A. lamp B. table C. family D. breakfast2/ A. warm B. park C. farm D. car3/ A. teaches B. washes C. goes D. brushes4/ A. parks B. travels C. tents D. stops

Full name: __________________Class : _____________________Testing date: ________________Number: ___________________

Page 205: Tài liệu, học tập, trắc nghiệm, tiếng anh, văn bản, biểu ...s1.vndoc.com/.../06/40-de-thi-hoc-ky...6-co-dap-an.pdf · "/ 0. 1 2 %$ 3 ( 0 4 5 6 2 3 ( 5789 578: 2

VnDoc - Tải tài liệu, văn bản pháp luật, biểu mẫu miễn phí

10

12.I am doing my homework …………a. now b. next week c. twice a week.

III. Supply the correct verb form. (1.5 ms)1. What about (go)……………….…..to the movies?

2. At the moment my sisters (have)...................................breakfast

3. Why don’t we (have)………………….noodles?

4. Nam and Minh (swim) …………………… now.

5. His sister does not want (go) …………………… camping.

6. ………..your brother (visit) …………………….your parents tomorrow?

1. going 2. is having 3. have 4. are swimming 5. to go 6. Will - visit

IV. Arrange these words in the correct sentence (2 ms)1. is/ What/ the weather like/ in the spring?

...............................................................................................................................................2. are going to/They/ visit/Hue/ next week.

...............................................................................................................................................3. all over the/ I am going/ to travel/ world

...............................................................................................................................................4. often/ does she/How/ go dancing?

...............................................................................................................................................

V. Read. Then do the tasks below. (2 ms)Nam likes hot weather. He usually goes swimming in the summer. In the fall, he

sometimes goes fishing with his friends. He doesn’t often go jogging in the winter. When it’scold, he stays at home and reads books. In the spring, the weather is warm, he goes walkingor camping twice a week. He is going to go to Ha Noi for 2 weeks this summer* T/F exercise

Statements T F1. Nam always goes swimming in the summer2. He often goes jogging in the winter3. It’s warm in the spring4. He never goes fishing with his friends

Page 206: Tài liệu, học tập, trắc nghiệm, tiếng anh, văn bản, biểu ...s1.vndoc.com/.../06/40-de-thi-hoc-ky...6-co-dap-an.pdf · "/ 0. 1 2 %$ 3 ( 0 4 5 6 2 3 ( 5789 578: 2

VnDoc - Tải tài liệu, văn bản pháp luật, biểu mẫu miễn phí

11

* Question1. How long is Nam going to go to Ha Noi?....................................................... ............................................................................................................................................... ........................................................................................2. How often does he go camping?...................................................................................................................................................................................................... ........................................................................................3. What does he do in the fall?...................................................................................................................................................................................................... ........................................................................................4. Does he go jogging when it’s cold?.................................................................................................................VI. Each sentence has one mistake find out and correct it. (0.5 m)

1. Mai ’d like walking in the mountain in her free time. …………. …………2. We are going to fly home on next month. …………. …………

TEST FOR THE SECOND TERM(6th form)

Name: …………….………………………………….Class: 6A ….

I. Circle the correct letter for each sentence below: (4 points)1, What is he going to do …………………? - He is going to Nha Trang.

A, this summer B, in this summer C, for this summer D, to this summer2, It is not very cold. Why ………………… go swimming?

A, not we B, let us not C, don’t we D, do we3, Hung is from Vietnam. He is …………………

A, a Vietnamese B, Vietnamese C, Vietnamese girl D, Vietnam4, Thuy’s 18 years old. Hoa’s 12 years old. Thuy is …………………

A, older than Hoa B, the oldest C, older than D, old5, It’s always very hot in the ………………….

A. winter B. summer C. fall D, cool6, ………… are you from? – I am from Japan.

A. Why B. How C. Who D, Where7, Why don’t we …………….. to the museum?

Page 207: Tài liệu, học tập, trắc nghiệm, tiếng anh, văn bản, biểu ...s1.vndoc.com/.../06/40-de-thi-hoc-ky...6-co-dap-an.pdf · "/ 0. 1 2 %$ 3 ( 0 4 5 6 2 3 ( 5789 578: 2

VnDoc - Tải tài liệu, văn bản pháp luật, biểu mẫu miễn phí

12

A. going B. go C. to go D, goes8, Mai is the …………………. in our class.

A. Short B. shorter C. Shortter D, shortest

II. Put in the gap with the right tense of verb given (2 points)1) We (not watch)…………………………….TV every night.2) She (read)……………………………… a book now.3) He (visit) ………………………her grandparents tomorrow.4) Let’s (play) …..………………… table tennis.

III. Read. Then answer the questions (2 points)Mrs. Mai is a nurse. She works in a hospital. She is short and thin but she is not weak. She plays lotsof sports. Her favorite sport is badminton. She likes jogging, too. She jogs twice a week. She jogs inthe park near her house. In her free time she watches TV or listens to music.1, What does Mrs. Mai do?=>....................................................................................................................................................................................................2, What is her favorite sport?=>....................................................................................................................................................................................................3, How often does she jog?=>....................................................................................................................................................................................................4.What does she do in her free time?=>....................................................................................................................................................................................................IV. Complete a passage about Long by using the words given. (2 points)His name/ Long. He/ be/ 12 years old. He/ be/ Viet Nam. He/ be/ student/ grade 6. He/ speak/Vietnamese. He/ like/ soccer/ very much. He/ usually/ play soccer/ after school. Next summervacation/ he/ visit/ Ha Long Bay. He/ stay/ a hotel/ 2 days.Beginning withHis name is Long. He .........................................................................................................................................................

............................................................................................................................................................................................................

............................................................................................................................................................................................................

............................................................................................................................................................................................................

............................................................................................................................................................................................................

Page 208: Tài liệu, học tập, trắc nghiệm, tiếng anh, văn bản, biểu ...s1.vndoc.com/.../06/40-de-thi-hoc-ky...6-co-dap-an.pdf · "/ 0. 1 2 %$ 3 ( 0 4 5 6 2 3 ( 5789 578: 2

VnDoc - Tải tài liệu, văn bản pháp luật, biểu mẫu miễn phí

13

Answer keys and points(6th form)

I. Circle the correct letter for each sentence below (4 points)Mçi c©u ®óng 0,25 ®iÓm

1, B 2, C 3, B 4, A 5, B 6, D 7, B 8, DII. Put in the gap with the right tense of verb given (2 points)

Mçi c©u ®óng 0,5 ®iÓm1, don’t watch2, is reading3, is going to visit4, playIII. Read. Then answer the questions (2 points)

Mçi c©u ®óng 1 ®iÓm1, Mrs. Mai is a nurse2, Her favorite sport is badminton3, She jogs twice a week4, She watches TV or listens to music.IV. Complete a passage about Long by using the words given. (2 points)

Mçi c©u ®óng 0,25 ®iÓmHis name is Long. He is 12 years old. He is Vietnamese. He is a student in grade 6.

He speaks Vietnamese. He likes soccer very much. He usually plays soccer after school.Next summer vacation he is going to visit Ha Long Bay. He is going to stay in a hotel for 2days.